You are on page 1of 248

Cornell UntDetsitg iCibtatg

JItltaca,

Ntw f orb

BOUGHT WITH THE INCOME OF THE

SAGE ENDOWMENT FUND


THE GIFT OF

HENRY W. SAGE
1891

Cornell University Library

arV19569
Elementary calculus

3 1924 031 220 811

The
tine

original of

tliis

book

is in

Cornell University Library.

There are no known copyright

restrictions in
text.

the United States on the use of the

http://www.archive.org/details/cu31924031220811

ELEMENTARY CALCULUS

THE MACMILLAN COMPANY


NEW YORK
BOSTON CHICAGO DALLAS ATLANTA SAN FRANCISCO

MACMILLAN &
LONDON

CO., Limited

BOMBAY CALCUTTA MELBOURNE


CO. OF TORONTO

THE MACMILLAN

CANADA,

Ltd.

ELEMENTARY CALCULUS
BY

WILLIAM

F.

OSGOOD,

Ph.D.,

LL.D

PEKKIN8 PK0FBS80K OF MATHEMATICS IN HABVAKD UNIVERSITY

Wetogorft

THE MACMILLAN COMPANY


1921
AU HgMe
5
reeemeel

COPTBIGHT, 1921,

bt the macmillan company.


Set

up and

electrotyped.

Published January, 1931.

J. S.

Berwick & Smith Co. Gushing Co. Norwood, Mass., U.S.A.

PREFACE
The
object of this book
is

to present the elements of the

Differential Calculus in a

form easily accessible for the undergraduate. It is possible, from the very beginning, to illustrate the ideas and methods of the Calculus by means of applications to physics and geometry, which the student can readily grasp, and which will seem to him of interest and value. To do this, the stress in the illustrative examples worked in the text must be laid first of all on the thought which underlies the method of solution, in distinction from the exposition of a process, reduced in the worst teaching to rules, whereby the answer can be obtained. The 'treatment of maxima and minima. Chapter III, 2, 3, and curve tracing, Chapter III, 5 and Chapter VII, 10, will serve to show what is here meant.
It
is,

however, also essential that the student receive thorough

training in the formal processes and the technique of the Calthis side has been treated with care and completeNote, for example, the differentiation of composite functions in Chapter II, 8, and the exposition of the use of differentials in differentiating in Chapter IV, 4, 5.

culus,
ness.

and

An

important application of the graphical inethods, with


is

which the Calculus


solving

so intimately associated,

is

that of

approximately numerical equations which do not come under the standard rules of algebra and trigonometry. Hitherto, however, little attempt has been made to present
is, in any systematic and elementary In Chapter VII the common methods in use by physicists and others who apply the Calculus are set forth

this subject, simple as it

manner.

and

illustrated by simple examples. The book might have included a

brief treatment of curva-

ture

and evolutes, and the

cycloid.

But probably most

VI

PREFACE
up integration
and

teachers of the Calculus will prefer to take


next,

so the closing chapter is devoted to the last of the

elementary functions, the inverse trigonometric functions, with


special reference to their one great application in the elements

of mathematics, namely, their application to integration.

The book an abridged


first

is

so written that

it

can be adapted,

if desired, to

course, in which, after the fundamentals of the

three chapters have been covered, any of the remaining

topics can be treated briefly,

and thus a wide scope in subject


is

matter

is possible,

even when the time

short.

Cambridge, Massacbusbtts,
January, 1921.

CONTENTS
CHAPTER
1.

INTRODUCTION
PAGE
2.

Functions Continuation.

General Definition of a Function

10

CHAPTER

II

DIFFERENTIATION OF ALGEBRAIC FUNCTIONS.

GENERAL
13 16

THEOREMS
1.

Definition of the Derivative


Differentiation oi x"
Differentiation ot '^x
.

2. 3. 4. 5. 6. 7.

Differentiation of a Constant
.

20
21 22

8. 9.

Three Theorems about Limits. Infinity General Formulas of Differentiation General Formulas of Differentiation, Continued General Formulas of Differentiation, Concluded
Differentiation of Implicit Algebraic Functions

29 32 35 39

CHAPTER
Tangents and Normals

III

APPLICATIONS
1.

.46
49 53 60 64
.
.

2. 3. 4.
5.

Maxima and Minima


Continuation
:

Auxiliary Variables

Increasing and Decreasing Functions

Curve Tracing
Relative

...

6. 7.

Maxima and Minima.

Points of Inflection

67
71

8.

Necessary and Sufficient Conditions Velocity; Rates


vii

72

VUl

CONTENTS
CHAPTER
Infinitesimals

IV

INFINITESIMALS AND DIFFERENTIALS


1.

2. 3. 4. 5.

Continuation.
Differentials

Fundamental Theorem

... ....
.

PAGE

Technique of Differentiation
Continuation.
Differentiation of Composite Functions

81 87 91 95 100

CHAPTER V
TRIGONOMETRIC FUNCTIONS
1.

Radian Measure

....

2. 3. 4.
5.

Differentiation of sin x

Certain Limits Critique of the Foregoing Differentiation


Differentiation of cos x, tan x, etc

.... ....

105 110 112


116

117 118
121

6. 7. 8.
9.

Shop Work Maxima and Minima


.

Tangents

in Polar Coordinates

128 134 138

Differential of

Arc

10.

Rates and Velocities

CHAPTER
Logarithms
Differentiation of Logarithms
1

VI

LOGARITHMS AND EXPONENTIALS


1.

146
151

2.

3.

The Limit

hm

(1+0'
Interest

155

4. 5. 6.
7.

The Compound

Law

Differentiation of e^

156 167
to

Graph of the Function x" The Formulas of Differentiation

Date
VII

....

160
161

CHAPTER
The Problem
of Niunerical

APPLICATIONS
1.

Computation

2.

Solution of Equations.

Known Graphs

.... ....

166 166

CONTENTS
3. 4. 5. 6. 7. 8.

IX
PAGE

Interpolation

9.

Newton's Method Direct Use of the Tables Successive Approximations Arrangement of the Numerical Work in Tabular Form Algebraic Equations Continuation. Cubics and Biquadratics

...

....
. . .

170
172 176

180 184
187 189 195

10.

Curve Plotting

CHAPTER

VIII

THE INVERSE TRIGONOMETRIC FUNCTIONS


1. 2. 3.
4.
5.

Inverse Functions The Inverse Trigonometric Functions

...

...
.

Shop Work
Continuation.
Applications

Numerical Computation

....

206 209 215 218 221

CALCULUS
CHAPTER
The
was invented
I

INTRODUCTION
Calexilus

in the seventeenth century

the mathematician, astronomer, and physicist, Sir Isaac


in England, and the philosopher Leibniz in

by Newton Germany. The

reaction of the invention on geometry and mathematical physics

was most important. In fact, by far the greatest part of the mathematics and the physics of the present day owes its
existence to this invention.
1.

first

Functions. The word function, in mathematics, was applied to an expression involving one or more letters
;

which represent variable quantities


expressions
(a)
a?,

as,

for example, the

23?-Z'x
Va'^

+ l;
;

(6)

V,
x^

x^

,.

xy
x^

ax
Va;2

+ by

a
(d)

+x
sin
a;,

+ y^
logs;,

^yij^e2

tan^isc.

In the second example under (6), two letters enter but a thought of as chosen in advance and then held fast, x alone being variable. A quantity of this kind is called a constant.
;

is

Thus
is

ax

+o
6.
1

a function of x which depends on two constants, a and

'

CALCULUS

Such expressions are written in symbolic, or abbreviated, form as f(x), f(x, y) (read "/ of x," "f of x and y " etc.) other letters in common use being F, $, etc.* Thus the
:

<f>,

equation
(1)

f{x)

= 2a?-'&x + l =

defines the function /() in the present case to be 2a!' 3a;+l.

Again,
(2)
is
<j,{x,

y, e)

x"-

+ y'

-\-

an equation defining the function

<j>{x,

y, z) as x^

+ y^ +

z*.

We shall be concerned for the present


single variable, as illustrated

by

(1)

with functions of one above. Here, x is called

the independent variable, since


like.
is

we

The value

of the function, or

called the dependent variable,

it any value we more briefly, the function, and is often denoted by a

assign to

single letter, as

^ =/() y = 23? Zx + l.
Graphs.

or

function

of a single variable,

y =/(<), can
be
represented

geometrically

by

its

graph, and this representation is of great aid


in studying the properties

of

the

function.
vari-

The independent
able
is laid off

as the

as-coordinate,
scissa,

or

ab-

and the dependx"

ent variable, or funcfirst

* To distinguish between /(x) and F(x), read the and the second, " Uu^e Fotx."

" small/

of

INTRODUCTION
tion, as the y^oordinate, or ordinate.

Thus the graph of the

function
is

the curve

Illustrations from Geometry and Physics. The familiar formulas of geometry and physics afford simple examples of

functions.

Thus the

area,

A, of a

circle is

given by the

formula

A = jtr\

where r denotes the

radius, n being the fixed number 3.1416. Here, r is thought of as the independent variable, it may have any positive value whatever, and is the function, or

dependent variable.
Again, for the three round bodies, the volumes are
(a)
(6)
(c)

F=

firr^,

sphere
cylinder;
cone.

F=irr%, V='^r%,
(6)

In

and
;

(c),

the base

is

h denotes the altitude and r, the radius of here a function of the two independent

variables, r

and

h.

The
(a)
(/8)

surfaces of these bodies are given

by the formulas

(y)
I,

S = Attt^, S = 2Trrh, S=
tttI,

sphere
cylinder

cone

in the last formula, denotiag the slant height.

Thus we

have three further examples of functions of one or of two


variables.

The formula

for a freely falling


s

body

is

= \gt\
t

where

denotes the distance fallen and

the time

$>

is

constant, for it has just one value after the units of time

and

4
length liave been chosen.

CALCULUS
Here,
If,
t

is

and
for
f

8 is

the function.

however,
,

the independent variable we solve this equation

then s becomes the radependent variable and t, the function. Sometimes two variables are connected by an equation, as

pv

c,

where p denotes the pressure of a gas and v


temperature remaining constant.

its

volume, the

Here, either variable can

be chosen as the independent variable, and when the equation is solved for the other variable, the latter becomes the de-

pendent variable, or function.

Thus,
c

if

we

write

P
p
is

the independent variable, and v

is

expressed as a function

of p.

But

if

we

write
-, P=V

the rSles are reversed.


variable

The Independent Variable Bestricted. Often the independent is restricted to a certain interval, as in the case of the
y

function

= Vo^ x\
and a
:

Here, x must

lie

between

a<^ x<^a.
since other values of x

make

a^

x^

negative,

and the above

expression has no meaning.

This was also the case with the geometric examples above There, r, h, I were necessarily positive, since there is no such thing, for example, as a sphere of zero or negative
cited.

radius.

The independent

variable

may

also be restricted to being a

positive whole number, as in the case of the

sum

of the first

INTRODUCTION
terms of a geometric progression
s
:
'

= a + ar + ar^ +
1

+ ar'-K

Here,

a ar"
r
Suppose a

= 1, r = ^, the progression thus becoming


1

1+.. .+J-. + 1 + 22 ^ 2"-i


2

Then

-i.-^
2

and we have an example of a function with the iadependent


variable a natural number,
i.e. a positive integer. In the case of ihe functions treated in the calculus, the domain of the independent variable is a continuum, i.e., for functions of a single variable, an interval, as

a-^x<b,

or

<

as.

Ordinarily, the later letters of the alphabet, particularly


X, y, z,

are used to represent variables, the early letters denot-

ing constants.
pression as

Thus

it will
aad'

be understood, when such an ex-

+ OX + C
and x
is

is

written down, that

o, 6, c

are constants

the variable.

Multiple-Vahied Functions;

Principal Value.

The

expres-

sions above cited are all examples of single-valued functions


i.e. to each value of the independent variable x corresponds but one value of the function. A function may, however, be

muUiple-valued ; as in the case of the function y defined the equation


a,2

by

+ ^2 _ a\

Here
y=

Va^ x^,

6
and

CALCULUS

so is a double-valued function. This function is, however, completely represented by means of the two single-valued

functions,

= ^a^x^

and

y = Va* iC*.
Graph of

Oraphofy,when

INTRODUCTION
is

often called the principal value of the double- valued function

defined

by the equation

Since multiple-valued functions are studied


single-valued functions,
less
it -will
,

by means

of

be understood henceforth, un-

the contrary

is

explicitly stated, that the

word function

means

single-valued function.

Absolute Value.

It

is

frequently desirable to use merely

the numerical, or absolute value of a quantity, and to have a

notation for the same.

The notation

is:

|a!|,

read " absolute

value of X."

Thus

|-3| = 3

and

|3|=3.

We
stated

can

now

write in a single formula what was formerly


(1),

by the two equations

namely the

definition of the

radical sign,
(2)

V^ = |a|.
we
often

Again, by the difference of two numbers


value of the larger less the smaller.

mean

the

and 10

is

difference of

and the difference a and 6, in this sense, can be expressed

Thus the difference of 4 of 10 and 4 is also 6. The


as either

Continuous Functions. function, /(), is said to be continuous if a slight change in x produces but a slight change in
the value of the function.

shown
tions

to be continuous

cf.

Thus the polynomials are readUy Chap. II, 5, and all the func-

with which we

shall

have to deal are continuous, save at

exceptional points.

As an example
certain point

may

of a function which is discontinuous at a be cited the function (see Tig. 3)

/(.)=i.

CALCULUS
approaches the value
0,

When X

the function increases nuof the function has the

merically without limit.


axis of y as

The graph

an asymptote.

The

fractional rational functions are continuous except at

the points at which the denominator vanishes.

y=

1^

Thus the function

^'-'-^
is
a;

continuous except at the points

=1

function

and a; = becomes
the curve

1.

Here,

the
Its

infinite.

graph

is

x'^

+1
=l

y~{x-l)Qc + iy
which evidently has the lines x and a = 1 as asymptotes.

The function
Fig. 3
is

/(a;)

= tan

a;

continuous except

when x
^ X

is

an odd midtiple of
TT.

ir/2,

+l ^ 2w -r

EXERCISES
1.

If

f{x)
/(1)

=x^-4:X + 3,

show that

= 0,

/(2)=-l,

/(3)

= 0.

Compute /(0),/(4).
2.

Plot the graph of the function.


^(a;)

If

"

= 4a!'
2,x-Z
x

compute ^(2) and ^(V3).


3.

If

F(x):

+7

'

compute F{-\/2) correct to three

significant figures.

Ans. -.0204.

INTRODUCTION
4.

If

$()

(a;'

a;)siii x,

find all the values of x for -whicli

*(0)=0.
5.

If

^(a!)=a!^

a;"^,

findi/-(8).

6.

Solve the equation


3^

xy

-\-

= By
x.

for y, thus expressing ^ as a function of


7.

If

f(x)=ci^,
/(a')/(2/)

show tlmt
8.

=/( + y)^

If
a;

y=
as a function of
y.

+ ^,

express
9.

Draw

the graph of the function


/(a;)=cB2

+ 4a! + 3,

taking 1 em. as the unit.

Suggestion
10.

Write the function in the form, (x

+ l)(a; + 3).

Draw

the graph of the function


f[x)

= a^ 4

a;.

11.

Draw

the graph of the function

and hence
tion has.
12.

illustrate the

two

discontinuities

which

this func-

Draw

the graph of the function


f( ^^^'
X

1
a?

{x-lf

10
13.

CALCULUS
For what values of x are the following functions
dis-

continuous ?
(a) /()
(6)

= cot f(x) = sec


a;

= esc (d) /() = tan |(c)

f(x)

a;

14.

Express the double-valued function

defined

by the

equation
in terms of
15.

a?-f = -l
two single-valued functions.

Express the quadruple-valued function defined by the


y*

equation

- 2j/^ +

x''

=
n terms of the arithmetic

in terms of four single-valued functions.


16.

Express the sum

of the first

progression

a+{a + b)+(a + 2b)+


as a function of n.

+(a + n

lb)

Thus obtain the sum of the


function of n.
17.

first

n positive integers as a

If

dollars are put at simple interest for one year at

r per cent, (a) express the amount


as a function of

(principal

and

interest)

Express the amount A at the end of n years, the interest being compounded annually, as a function of P, r, and n. (c) Express the amount at the end of one year, if the interest is compounded m times in the year at equal intervals, as a function of P, r, m.
r.

P and

(b)

2.

Continuation.

ception of the function


cal formulas

General Definition of a Function. The conis broader than that of the mathemati-

the definition in

mentioned in the last paragraph. its most general form.


77ie variable

Let us state

Definition of a Function.
is given,

is

said to be a

function of the variable x if there eadsts a law whereby, when x

is determined.

INTRODUCTION

11

Consider, for example, a quantity of gas confined in a chamber,

for instance, the charge of the mixture of gasolene and


compressed in the cylinder of an automobLle.
this pressure varies

air as it is being

The charge

many
and

exerts at each instant a definite pressure, p, of so pounds per square inch on the walls of the chamber,

with the volume, v, occupied by the In the small fraction of a second under consideration, presumably but little heat is gained or lost through the walls of the chamber, and thus p is a function of v,
charge.

In this

ease, the function is

given approximately by the math-

ematical formula

where

C denotes a certain constant.

But that which

is

of first

importance for our conception is not the formula, but the fact that to each value of v there corresponds a definite value of p. In other words, there is a definite graph of the relation be-

tween V and p. The representation of the relation by a mathematical formula is, indeed, important; but what we must first see clearly is the fact that there is a definite relation to
express.

As another
pen of a
cal

illustration take the curve traced out

by the

self-registering

meteorological station.

thermometer of the kind used at a The instrument consists of a cylindri-

drum turned
at uniform

slowly by clock-

work
speed

about
of

vertical

axis,

sheet

a a paper

^la. 4

being

wound

firmly round the drum.

A pen is held against the paper, and pen above a certain level is proportional to the height of the
the height of the temperature above the temperature corre-

12

CALCULUS

when

sponding to that level. The apparatus is set in operation, and the drum has been turning for a day, the paper is taken off and spread out flat. Thus we have before us the graph of the temperature for the day ia question, the independent variable being the time (measured in hours from midnight) and the dependent variable being the temperature, represented by the other coordinate of a point on the curve. One more illustration, that of the resistance of the atmosphere to a rifle bullet. This resistance, measured in pounds, depends on the velocity of the bullet, and it is a matter of

physical experiment to determine the law.


is is

But that which


the fact that there

of first importance for our conceptions

is

law, whereby,

when

the velocity,

v, is

given an arbitrary

value withia the limits of the velocities considered, there cor-

responds to this V a definite value, B, of the resistance. is a, function of v and write say, then, that

We

B = fiv).
In
this connection, ef. the chapter

on Mechanics,

7,

Graph

of the Resistance, La the author's Differential

and

Integral

Gdkulus.

CHAPTER

II

DIFFERENTIATION OF ALGEBRAIC FUNCTIONS

GENERAL THEOREMS
1.

Definition of the Derivative.

The Calculus

deals with

varying quantity. If j/ is a function of x, then x is thought of, not as having one or another special value, but as flowing or growing, just as we think of time or of the expanding circular ripples

made by a

stone dropped into a placid pond.

And y
ing.

varies with x, sometimes increasing, sometimes decreas-

Now if we

consider the change in x for a short interval,


x',

say from

x=:Xotox=
y',

the corresponding change in y,a,sy


ratio of these changes is

goes from y^ to the change in x.

will be iu general almost proportional to

For the

x'

Xa
when
x' is

and

this

quantity changes only slightly


Xq.

nearly

equal to

Let us study this

last

statement minutely.

Fig. 5

The above

ratio has a simple geometric meaning, if


;

we draw

the graph of the function

for

PM=x'-Xa;
13

MP' = y'-y^,

14

CALCULUS

aj'-OTo

where

t'

denotes the angle which the secant


x.

PP

makes with

approach ojq as its limit. Then r' approaches as its limit the angle t which the tangent line of the graph at makes with the axis of x, and hence
the axis of
let x'

Now

lim y x'^ x'


(

~yo = tan r

!Cd

read

"limit, as x' approaches

Xg,

of ^
X'

~^

Xf,

J
of
its

The determination of

this limit

and

the discussion

mean-

ing is the fundamental problem of the Differential Calculus. Such are the concepts which underlie the idea of the derivative of

the definition.
(1)

a function. Let

We

turn

now

to

a precise formulation of

y=f(')
x,

be a given function of x. Let Xq be an arbitrary value of and let ^o ^^^ corresponding value of the function

(2)

2/0

=/(ai)).
;

Give to X an Lucrement,* Ax i.e. let x have a new value, and denote the change in x, namely, a/ aj^, by Aa;
a;'

x',

a!o

= Aa;,

a;'

= a^ + Aa;.

The
(3)

function, y, will thereby have changed to the value


2/'

=/('')

and hence have received an iucrement. Ay, where

y'-yo = Ay,

y'

= yo + %

The student must not think of this symbol as meaziing A times x. used a single letter, as h, to represent the difference in question x'=Xo + h; hut h would not have reminded us that it is the

We might have
:

increment of
is

x,

and not

of y, with

which we are concerned. The notation

read " delta x."

DIFFERENTIATION OF ALGEBRAIC FUNCTIONS


Equation
(4)
'

15

(3) is

equivalent to the following


yo

+ Ay =/(a'o + Aaj).
we
obtain

From
equation

equations (2) and (4)

by subtraction the

A2/=/(a!o

+ Aa;)-/(a!o),
Aaj

and hence
C5-V ^ ^

Ay^ /(a!o+Aa!)-/(a;o)
Ak
oi"

Definition
(5),

a Dbeivativb. Ax

The

limit

which the ratio

namely

approaches when

approaches zero

(6)

lim^
tix^Ax

or

li^/^^Mr^M,
AxiO

Ax

is called the derivative

of y with respect to x and is denoted by

Bjy or DJioi) (read


(J)

"D x oiy"):
ox^Ax

In

this definition

Ax may be

negative as well as positive, and

from the the limit (6) must be the same when Aa; approaches negative side as when it approaches from the positive side.

To

differentiate
is

a
y
in-

function

to find its

derivative.

The geometrical
terpretation
of

the
to

analytical process of
differentiation
is

find the slope of the

graph of the function.


Por,

tanr'=^

16
2.
it is

CALCULUS
Differentiation of x".

Suppose n has the value

3,

so that

required to diEEerentiate the function

(1)

a?.

by step. Begin, which is to be held fast during the, rest of the process, and compute from equation (1) the corresponding value yoofy:
then,

We

must follow the definition of by assigning to a; a particxilar

1 step

value,

a^,

(2)

2/0

= XgK

Next, give to x an arbitrary increment. Ax, denote the corresponding increment in y by Ay, and compute it. To this end we first write down the equation
(3)

ya

+ Ay={xo+As:y.
expanded by the
:*

The right-hand

side of this equation can be


(3)

binomial theorem, and hence


(4)

can be written in a new form


-(-

yo+Ay = 00o^ + 3 x^^Ax + 3 x^Ax^


:

Aa?.

Subtract equation (2) from equation (4)

Ay = Sxg^Ax

+ 3 XgAx'^ + Aa^.

Next, divide through by Ao;

Ax

^ = SV + 3xJ^x +
lim (3 a;2

Ax''.

We are now ready to let Ax approach


lim

as its limit

M=

+ 3 x^Ax + Ax^).
come

*It
to

is

at this point that the specific properties of the functional

into play.

Here,
will

it is

compute the
it

limit.

the'binomial theorem that enables us ultimately In the differentiations of later paragraphs and
h.e

chapters

always

some

characteristic property of the function in

hand which

will

make

possible

a transformation at

this point.

DIFFERENTIATION OF ALGEBRAIC FUNCTIONS


The

17

limit of the left-hand side is, by definition, D,y. On the right-hand side, each of the last two terms in the parenthesis

approaches the limit

0, and so their sum approaches 0, also. term does not change with Aw. Hence, the whole parenthesis approaches the limit Swo^. We have, then, as the

The

first

final result

0,2/

= 3 V-

The

subscript has
Xq is

now

remind us that
dropped.

served its purpose, which was, to not to vary with Ax, and it may be

Thus

d^^=3xK

that

The n

differentiation of the function x" in the general caseis

cisely the
(6)

any positive integer can be carried through in presame manner. As the result of the first step we have
'

2/0

aso"-

Next comes
(6)
y

+ Ay = (Xo + Aa;),
to the expression

and we now apply the binomial theorem the right-hand side. Thus
(7) yo

on

+ ^y = xo" + mV^Ak + '^^^~J'^ a;o-2Aa;2 H


we have
h
Aa;".

Aa;.

On

subtracting (6) from (7)

Ay = nXo"-^Ax

+ ^^"' ~J'^ x^^-^-Ax"- H 1 ^

Now divide

through by Ax
nxo"-^

^= Ax
and
let

+ '^^"'~'^\ --^Ax 1 '2


the limit zero

++

Ax--^

Ax approach

Arm Ax

lim

^ = lim
/^asiO

f a!o"-i

+ "^" ~ ^^ V'^Aw -H 1-2

-I-

Aa!-i\

J
is

Each term

of the parenthesis after the first

the product

of a constant factor

and a positive power of

Aa;.

This second

18
factor approaches zero

CALCULUS
when
Aa;
;

approaches zero consequently There is only a fixed number of these terms, and so the whole parenthesis approaches the limit waso""". Hence
the whole term approaches zero.

On
(8)

dropping the subscript we obtain the


Z),B

final result

raa!-i.

In particular,
(9)

if

= 1,

we have
D,x

= l.

EXERCISES
Differentiate the following seven functions, applying the

process of 1 step
1.

by

step.
,

DIFFERENTIATION OF ALGEBRAIC FUNCTIONS

19

Plot the curve accurately for values of x from a; = to x = 5, taking 1 cm. as the unit, and draw the secants* in each of the three foregoing cases.

What
{xq, yo)
9.

= (l> 4)?
In Ex.

appears to be the slope of the curve at the point Prove your guess to be correct.
7, let
a!o

= 1- H

Ax

is

given successively the

values .01 and


10.

.01,

compute Ay and

^
Ax

Complete the following table

Ax

20
3.

CALCULUS
Derivative of a Constant.

The function

where
lel

c denotes a constant, has for its


x.

graph a right
it is

line paralis

to the axis of

Since the derivative of a function

repre-

sented geometrically by the slope of its graph, the derivative of this function is zero

clear that

Dji
It
is

= 0.

instructive, however, to obtain this result analytically

by the

process of

1.

We have here
2/o=/(9i))=c,

yo + Ay =/(a!o4- Aa;)= c;

hence

Aw "
Aa; to
its

and
0.

2 =
Aa;

0.

Now allow
0,

approach
:

The value

of Ay/Aa;

is

always

and hence

limit * is

lim^=0,
Ax^ Aa;
*

or

Z),c

= 0.
" a variahle JT

We note here an error

frequently

made

in presenting the subject of

limits in school mathematics.

It is there often stated that

approaches a limit Ait comes indefinitely near to A, but never reaches A." This last requirement is not a part of the conception of a variable's

approaching a
tion, the ratio

limit.

It is true that

it is

often inexpedient to allow the

independent variable to reach


sion

its limit.

Thus, in differentiating a func-

by
is

value of

Ay/Ax ceases to have a meaning when As = 0, since diviThe problem of differentiation is not to find the AyI Ax, when Ax = such a question would be absurd. What
is

impossible.

we do

to allow Aa; to

that limit.
ario6ie.

We

its limit without ever reaching can do this for the reason that Az is the iTidepemdent

approach zero as

it is Ay or Ay/Ax that is under consideration, we have do with dependevi variables, and we have no control over them, as to whether they reach their limit or not. Thus in the case of the text both Ay and Ay/ Ax are constants (=0). When Aa; approaches 0, they always have one and the same value, and so, under the correct concei>tion of approach to a limit each approaches a limit, namely 0.

When, however,

to

DIFFERENTIATION OF ALGEBRAIC FUNCTIONS

21

We
4.

can state the result by saying

The derivative of a con-

stant is 0.

Differentiation of Va;.

Let us differentiate
y=-\/x.

Here,

2/0

= V0)

+ % = Va;o + Aa;, Ay _ Vgp + Aa; VajQ


2/o

Ax
when
Aa;

Aa;

We cannot as yet see what limit the right-hand


approaches
0, 0, for

side approaches both numerator and denominator

approach

and - has no meaning.

We

can, however, trans-

form the fraction by multiplying numerator and denominator by the sum of the radicals and recalling the formula of Elementary Algebra
=

_ 52 =( _ 6)( + j).
_

Thus

%
^a;

V^'o

+ Aa; Vajp
Aa;

Vxg +
Va;o

Aa;

-j-

Vxp

^^
and hence lim ^ = lim

+ Aa!+V% (xg + Aa;) 1 Vxo + AX + Vwo -Vxq + Aa; V^'

a^ Ax

i^ Va!o

^
-(-

a;o

-|-'

^=

^^

Aa; -H Va;o

2 VaJo

Dropping the subscript, we have

2Vm
EXERCISES
1.

Differentiate the function'^/

=
Vx

Ans. Dj)

=
2-\/a?

2.

If

2/=V2-3a;,
Z)j

show that

2V2-3a;

=^=r

22
3.

CALCULUS
Prove:

D,Vl x =
2)^Va

2Vl-a!
6

4.

Prove:

6a;

2 Va

+ bx

In the further 5. Three Theorems about Limits. Infinity.* treatment of differentiation the following theorems are needed.

the

Theobem I. The limit sum of their limits:


Urn

of the

sum of two

variables is equal to

{X+Y)= lim X + lim T.

variables,

In this theorem we think of and each of which approaches a limit

as

two dependent

limX=^,

limr=JB.

We

do hot care what the independent variable

may

be.

In

the applications of the theorem to computing derivatives, the

independent variable will always be Ax, and it will be allowed to approach 0, without ever reaching its limit. approaches A, it comes nearer and nearer to this Since

Let the difference between the variable and be denoted by then the limit of e is
value.
; :

its limit

(1)

DIFFERENTIATION OF ALGEBRAIC FUNCTIONS


It will be convenient to think of these

23

sented geometrically
t

A
and

by

points on the scale of numbers, thus

nt
Fig. 7

0.
r,.

numbers as repre:

r-te

Of

course,

may be

negative or
0.

and
(1)

rj

may

be

negative as well as positive, or even

Consider the variable

X+Y.

Its value

from

and

(2) is
,.

X + T=A + B + +
Hence
lim (X+Y)

^,
rj).

=lim(A

+B+ e+

But since lim e

and lim 17
is

= 0,

the limit of the right-hand

side of this equation

Consequently,

A + B, or lim{X + Y)=A + B. lim (X+Y)= lim X + lim P,


The
limit

q. e. d.

CoBOLLAKY.

of the

sum of any

fined

number of

variables is equal to the sum, of the limits of these variables :

lim (Xi

+ X2+-+X,)= lim Xi + lim Xj Suppose n = 3. Then Xi + X2 X3 = (Xi X2) -f X3.


-1-

-1-

lim X.

-I-

-f-

From Theorem

it

follows that

lim (Xi

-f-

X2 + X3)

= lim (Xi

-|-

X2) -1- lim Xs-

Applying the Theorem again, we have


lim (Xi

+ Xj) = lim Xj
is

-|-

lim Xj.
It can

Hence the
lished for
ji

corollary
;

true for

= 3.

how be

estab-

and so on. By the method of Mathematical Induction it can be proven generally. Or, the proof of the main theorem may be extended directly to the present
theorem.

=4

24

CALCULUS
II.
Tlie limit

Theorem

of the product of two variables

is

equal to the product of their limits

lim

(XF) = (limX)(lim
it

T).

From

equations (1) and (2)

follows that

or

XT= AB + B + Ari +
lim 'XT= lim
Since

erj.

Hence

(AB + B + A7i + eq).


each of the last three terms in 0, and so the limit of the

A and B are constants,


is

the parenthesis approaches the limit


parenthesis

AB.

Hence

lim{XY)= AB,
or

lim

{XT) = (lim :X)(lim

T),
is

q. e. d.

Corollary.
to the

The limit of the product of n variables product of the limits of these variables :
lim (XiXj
...

equal

X) = (lim Xi)(lim X^)

(limX).

The proof
Remark.

is

similar to that of the corollary under


particular case under

Theorem

I.

As a

Theorem II we have

lim(C'X) = C(limX),

where

O is

a constant.
III.

Theobbm
divisor is not

The

limit

of the quotient of two variables

is

equal to the quotient of their limits, provided that the limit of the
;

^
we have
t
7,
_

From

equations (1) and (2) above

X^ A +
Y B+

DIFFERENTIATION OF ALGEBRAIC FUNCTIONS


Subtract

25

AfB from

each side of this equation and reduce

X A_ A-^ A_ Bt Ar]
Y B B+
Hence
7]

B'

+ Br,'

26

CALCULUS

where

(7 is

a constant and lim

X^O.

as its limit, Remark. If the denominator approaches no general inference about the limit of the fraction can be drawn, as the following examples show. Let have the

values

F=i 10'
(1) If

J- -J100' 1000'
'

-^ 10"'

the corresponding values of

X are
'

102' 1002'

10002'

10""'

then

lim

X - = lim
T

(2)

If

X=

Vip' vioo' viooo'

111
'

=
1 10-

0.

^^'

then

X/F=10"/^
If

approaches no limit, but increases beyond

all limit.

(3)

X = ^, 10'
any

100' 1000'

'

10"'

'

where

c is

arbitrarily chosen fixed

number, then

lim
1

= Y
1

c.

fA\ Tf
*
''

"V^

J-

1
""10,000'

~io' ~ioo' iooo'

"'

then

X/ Y

assumes alternately the values

+1

and

1,

and

hence, although remaining finite, approaches no limit.

To sum
or it

up, then,
increase

we

see that

when

X and Y both approach


limit whatever, although remain-

as their limit, their ratio

may approach any


limit, or finally,

may

beyond all

DIFFERENTIATION OP ALGEBRAIC FUNCTIONS


ing finite,
i.e.

27

always lying between two fixed numbers, no matin value,

ter

how widely tlie latter may differ from each other it may jump about and so fail to approach a limit.

If limX=^:?t=0 and limF=0, then X/ Finbeyond all limit, or becomes infinite. A variable Z is said to become infinite when it ultimately becomes and remains greater numerically than any preassigned quantity, however large.* If it takes on only positive values, it becomes
Infinity.

creases

positively infinite; if only negative values, it becomes negatively


infinite.

We express its behavior by the notation or lim^ = -|-oo limZ = or limZ = oo.
oio

But

this notation does

not imply that infinity

is

a limit

the

variable in this case approaches no limit.

And

so the notation

should not be read " Z approaches infinity" or


infinity

"Z

equals

but " Z becomes infinite.'' Thus if the graph of a function has


;

its

tangent at a certain
shall

point parallel to the axis of ordinates,

we
,

have for that

point

lim
read
"

^ = 00

Ay/Ax becomes
writers find
it

infinite

when Ax approaches

0."

convenient to use the expression "a variable approaches a limit " to include the case that the variable becomes infinite.

Some

We

shall

pression, but shall understand the

not adopt this mode of exwords " approaches a limit

in their strict sense.


'*T!f

a function f(x) becomes

infinite

when x approaches a

cer-

tain value a, as for

example
/(a;)=for

= 0,

* Note that the statement sometimes made that " Z becomes greater than any assignable quantity" is absurd. There is no quantity that is greater than any assignable quantity.

28 we denote
this

CALCULUS
by writing
/(a)=oo

(or

/(a) =+00

or

= oo,

if this

happens to be the case

and we wish
It
is

to call attention to the fact).

in this sense that the equation

tan 90 =00
is

to be understood in Trigonometry.

mean
so.

The equation does not that 90 has a tangent and that the vcdue of the latter is It means that, as x approaches 90 as its limit, tana;

exceeds numerically any number one may name in advance, and stays above this number as x continues to approach 90

without ever reaching

its limit, 90.

Definition of a Continuous Function.


explicit the definition given iu

We can now make more


is

Chapter I by saying: f(x)

continuous at the poiut

a;

= a if

lim/(a!)=/(a).

From

Exercises 1-3 below

it

follows that the polynomials


x,

are continuous for all values of

and that the fractional

rational functions are continuous except


va.nishes.

when

the denominator

EXERCISES
1.

Show

that, if

is

any positive

integer,

lim(Z) = (limX).
2.

If

O (x) = Co + C^X + C2X^


lim
x=a

-\

+ cjif,
ofl,^

then

Q{x)= G(a)=

(^

+ CiU +

+ - + cjn".
if

3.

If Q{x)

and F(x) are any two polynomials and

F{a)^0,

then

lim^M = ^(^.

DIFFERENTIATION OF ALGEBRAIC FUNCTIONS


4.

29
show

If

X remains finite and


that

Y" approaches

as its limit,

^^"-^

lim(Xr)=0.
Show
^^^
a^

5.

x3a;2
Suggestion.

_1 + + 2a; 1~3"

Begin by dividing the numerator and the dex^.

nominator by

Evaluate the following limits


6.

^ + W-'^^, lim ^^''< ;=a!3-7a; + 3

7.

Km
.=
4:afi

12^!+5

+ 3x*-+7 x^-1
-^T

8.

lim?^^^'././. '''
x=a cx

+ da;"'
X

9.
*-^,

A\

lim^^^. ^^ x^ ex + dx~^
lim
="

10.

lim *-"

11.

^ VS + 5a;2-|- 4^
'

12.

lim

^^ + ^' + '^
X

13.

limX=o<,

VI + a*

6.

General Formulas of differentiation.


I.

Theorem
a function
tive
(I)
is

The derivative of

the product

of a constant and

equal to the product of the constant into the deriva-

of the function

30
The

CALCULUS
limit of the left-hand side is

D^.

On

approaches

D^

as its limit.
is

limit of the right-hand side

Hence by 5, Theorem cD.tt, and we have

the right, Aw/Aa; II, the

X>,(cm)=cD,m,

q.e.d.

equal to the

Theorem II. sum

Tim derivative of of their derivatives

the

sum of two

functions

is

(H)

DIFFERENTIATION OF ALGEBRAIC FUNCTIONS


Polynomials.

31

We

are

now

in a position to differentiate

any

polynomial.

For example:

= 7D,3^ - 5Dji? + 1 = 280)3 - 153? + 1.


EXERCISES
Differentiate the following functions
1.

2.

3.

4.

= 2x^ Zx + l. y=a + bx+ cx\ y = x^-3<jfi + x l. y = a + bx + CO? +


y

Ans. D^y Ans.

= ix 3.

D^ b + 2cx.
x'-

Ans.

D^ = ia^ 9 + 1.
6a!' 1.
ax

dsfi.

5.

y ^
jr^.

=
2

ax--^2hx

Ans.

3a:^

6.

+c

^^^

+b
h

2h
7.

n-!B*-3|a;2+V3.
Differentiate
(a) Vot
(6)
(c)

Ans. inafi-T^x.

8.

16^2 with, respect to a + bs+ cs^ with respect to s .Qlly^ 8.15m2/^ .'dim with respect to y.
t
;

9.

Find the slope of the curve


42/

a;^

Bk 1
Ans.

at the point (1,


10.

2).
^y

1.

At what angle does the curve

= AiX a?

cut the negative axis of

a;

32
11.

CALCULUS
At wtat
angles do the curves y

= 3? and y =

oi?

intersect ?
7'-

Ans. 0 and 8
12.

At what

angles do the curves y

intersect?

= a? Zx and y = x Ans. ges-BC and 38 40'.

7.

General Formulas of Differentiation, Continued.


III.

Theorem
formula :
(III)

The derivative of a product

is

given by the

D^uv) = uD^v
2/

+ vD^u.
r

Let

=^ Mv.

Then
2/0

2/o

= Mo^)
(''o

+ % = (mo + ^) + ^i>), Ay = UyAv + VqAu + AmAv,


Av = -^ Ax
Av
Wo
Aa;
,

Am
""o

1-

1-

Aa;

Am Au Ax
.

Am

and, by

Theorem

I,

lim^=
^lioAa;

lim(

Ax^\

?to

|+ Hmf )+ Ax Ax
-Wo

lim(
Axiio\

i^3:^\

Ax

J
0, since

By Theorem
lim Am

II,

5,

the last limit has the value

and lim (Av/Asc) = Dj). The first two limits have the values UgD^v and VgD^u respectively.* Hence, dropping
the subscripts,

we have
D^y = uD^v

+ vDji,
differentiated.
liere,

q. e. d.

By

a repeated application of this theorem the product of

any number of functions can be


* More
strictly,
:

When more
Theorem
I, II,

the notation sliould read


etc.

before the subscripts

are dropped

[DiO]js=5||,

Similarly in the proofs of

andV.

DIFFERENTIATION OF ALGEBRAIC FUNCTIONS


than two' factors are present, the formula
ten in the form
is

33

conveniently writ-

D^(uvw)
(1)

_Dji
^

D^v
^

Djjo

uvw

w
is

For a reason that will appear


rithmic derivative of uvw.

later, this

called the loga-

Theokem
formula :*
(IV)

IV.

The derivative of a quotient

is

given by the

34
Example
1.

CALCULUS
Let

_ 2 3a;
^~l-2a!'

Then

i}y- a-2x)DJ2-3x)-(2-3x)DAl-2x) '^


(1

- 2xy

^ (l-2a!)(-3)-(2-3a!)(-2) ^
(l-2a;)2

(l-2xy'

Example

2.

To prove

that the theorem


X>j.a;"

= ma;""' = m.
Here

is

true

when

n.

is

a negative integer, n

a!"

1
a;"
wia!"""i

TT

Hence

n Djc"

^- = a!"i)^l liJjB" =

= nuc__1 \
""

On
proof

replacing
is

m in this last expression by its value, n,


EXERCISES

the

complete.

Differentiate the following functions

-^.^
4-

'^ -v

"

+ x^
a^
^
a;

'^

^y^

_-

v+

r.

+ x^y 32 2' (1
0'

a!)2

5.

=
1

Ans. Dfi
t
' '

t^. +a

Ans
'

= ~ (i + ty ^^ + 208-0" f? + 2az-ira?'

DIFFERENTIATION OF ALGEBRAIC FUNCTIONS


2 ay
a2-2/2
9.

35

ax+b

:lslji..

10.

36
The
letters

CALCULUS
truth of the theorem does not depend on the particular

by

vrhich

the variables are denoted.

We may replace,

for example,

xhj t and yhj


DfU

x:

= D^u D,x.
we
thus

Dividing through by the second factor on the right, obtain the formula
(V")

^A = D,x
1.

Example

In

4 we differentiated the function Va;, and

we saw
manner.

that other radicals can be differentiated in a similar

But each new differentiation required the evaluation by working through the details of a limiting process. Theorem V enables us to avoid such computations, as the following example will show.
of lim Ay/Aa;

To

differentiate the function

u = Va^
Let
y

'.

= a^ x*.
u

Then
and the
if

= Vy,
comes directly under Theorem V,

differentiation thus

we

set

/(y)

^y>
D,u

^ (') =

a^

a;^-

Hence we have
(1)

= D^y/y D^a^ - x^).


jy

Now,

the formula

-^~_

does not

mean

that the independent variable must be denoted


If the independent variable
_
D,-s/~y
is y,

by the
reads

letter x.

the formula

= -^2V^

DIFFERENTIATION OF ALGEBRAIC FUNCTIONS 37


Consequently
(2) (1)

can be written in the form

D^ri

= -!-(- 2 x) =
2V2/

"^
-

V02-

We have, then,

as the final result


D,-\/a?

x^ =

Va^
Exam/pie
2.

x'^

To

differentiate the function

Let

Then

- x/ z=:l X. y = z~K
(1

To apply Theorem V in the present case, the letters u and y must be replaced respectively by y and z. Thus Theorem V
reads here:
or

= D^Dj^, = D^^D^a-x). Dj)


D^y

Since Formula (8) of


tegral values of

2 has been extended to negative in-

n by

7,

Ex.

2,

we have

Z)^==-3-*.
Hence
or

D,y

= -Zz-\-l)=-.
z

D,
=

(l-a!)8

(1-a;)*

x-'

EXERCISES

Diiferentia|e A^^lowing functions


.-

'

1.

(f'i

v=Va2 +
2/

a;
a;''.

^ras,

P>
2.*

Va2

a;2

=
1.

a;

Va2

.dns.
a;2

V(a2
shown

a;^)^

* Note that case

Formula
4,

(8) of 2 has also been

to hold for the

= ^

Ex.

38
3.

CALCULUS
2/=Vl + a! + ai2.
y
Ans.

2 VI
<
, .

+
"^

"^
iB

4-

A 4.

A Ans.

V3-2a; + 4a!2

2a; VS 2 + 4
1
a;

a;^

~
6.

x^ + 1 u
(1

a;)'

Ans.

.6
a;)3

(1

- x)* + 4a!

{2-3a!)2
^

(2-39!)'
^

(l+2a!)*
10.

(2

9.

y=(-^Y.

-(r^T
M

2a; +
(1

"^

ai2

11.*
12. 13.

= a;(l M = a;(a + te)". u = x\a +


a;)<.

Ans.
-4ns.

5a!)(l

a!)3.

[a

6a;)".

14.

+ (w + l)6a;](a + 6a;)"~'u = a;'(l x)*.


^ns.

15.

M u

= a;Va

a;.

~
2 Va

16.

= 7?^(^ ^.
"^

17.

M = a;Vl+a; + a!2. =
"^
-

18.

M=

Va" -

a;2

19.

Vl + +
a;

a;^

20.

=JM^.
^^c

+ d*

21.t

^*=^
M=

i (a2-2aa;)<

22.

M=r
(a;2

- 1)2
^

23.

24.

M=

a;* 36a;2+362a!-6 6

25.

+ + a+6 M=
1
a;

a;'

a;

{a-irhxY
i

* Use Theorem HI. t Do not use Theorem IV.

DIFFERENTIATION OF ALGEBRAIC FUNCTIONS 39


9.

Differentiation of Implicit Algebraic Functions.

Wlien

X and y are connected by such a relation as

or or
i.e. if

+ y2 = a% = 0, a^ 2xy + xysaiy = x + y log x,


a;2 2/^

is

given as a function of x by an equation,

F{x,

y)=0
we

or

$ (a;,

y)

=*

(a;,

y),

which must
function of

first
x.

be solved for

If

y, then y is said to be an implicit solve the equation for y, thus obtaining

the equation

y =/(),
2/

thereby becomes an

explicit

function of
a;

x.

By an
satisfies

algebraic function of

is

meant a function y which

an equation of the form

O{x,y)=0,
where G(x,
of
y) is an irreducible polynomial in x and y; i.e. a polynomial that cannot be factored and written as the product

two polynomials. Thus the polynomials are algebraic functions y

for if

= ao + aiX+

+ " = P{vi),

then y

satisfies

the algebraic equation

G{x,y)=y-P{x)=0.
Similarly, the fractions in x are algebraic functions
;

for if

P(xy

w^here
factor,

-P(a;)

and

Q(<c)

are polynomials having no

common

then y

satisfies

the algebraic equation

G{x,y) = Qix)y-Fix)=.0.

40

CALCULUS
fractions are also called rationed

The polynomials and the


functions. '

Thus,

ax
a;2

by +^ + y^
y.

is

a rational function of the two independent variables x and Agaia, all roots of polynomials, as

y
or such functions as

= Vl + X +
X

sfi,

^1
are algebraic, as
is

seen on freeing the equation from radicals


converse, however,

and transposing.
tions

The

namely, that every


of rational funcit is sufB.cient

algebraic function can be expressed

and

radicals,

by means

is

not true.

In order to differentiate an algebraic function,


to differentiate the equation as it stands.
(1) x^

Thus

if

+ y^ = a?,

we have
(2)
Djc''-

+ Dy = D,a?.
Theorem V,
8.

To
Thus

find the value of the second term, apply

Z>^2

= D^'D^ = 2yDj/.

find,

This last factor, D^, is precisely the derivative we wish to and it is given by completiug the differentiations indio
,

cated in (2)

t^

and solving

this equation for

Djy

The

final result
(1) for

is,
:

of course, the
,

same as

if

we had

solved

equation ^

y= Vo* - x\

DIFFERENTIATION OF ALGEBRAIC FUNCTIONS 41


and then differentiated

Va2
In the
(3)

a?

case,

however, of the equation

a?-2xy + y^ = 0,
solve for y

we cannot

and obtain an

explicit function expressed

in terms of radicals.

Nevertheless, the equation defines y as a perfectly definite function of x for, on giving to x any
;

special numerical value, as x

tion for

y,

here,

= 2, we

have an algebraic func-

and the roots of


of precision.

this equation can

be computed to any degree

To find

the derivative of this function, differentiate equation

(3) as it stands (4)

with respect to x

Dj>?-2DXxy)

+ D^y^ = 0.
by

The second term Theorem III of 7

in this last equation can be evaluated


:

I>,(xy)

= xD^y + y, V of

where Dj^ denotes the derivative we wish to find. To the evaluation of the third term in (4) Theorem
applies:

Dy = 5y^Dj,.
3a!2

^^^'^^

- 2 xDj^ -2y + 5y*D,y =


we have

0.

Solving this equation for Dj/,

as the final result

'^

5y*-2x
is

Thus, for example, the curve


(1, 1),

seen to go through the point

and

its

slope there

is

(-D^)a,i)=-i-

42

CALCULUS
differentiation of implicit functions as set forth in the

The

above examples

is based on the assumptions a) that the given equation defines y as a function of a; 6) that this function has a derivative. The proof of these assumptions belongs to a more advanced stage of analysis. In the case, however, of the
;

we meet in practice, for example, such equations the condicome from a problem ia geometry or physics, tions for the existence of a solution and of its derivative are fulfilled, and we shall take it for granted henceforth that this
equations
as

is

true of the implicit functions

we

meet.
are

Derivative of x", n Fractional. prove the theorem Djc" =


for the case that

We
,

now

in a position to

nx"''-

is

fra,ction.

Let

where p, q are whole numbers which are prime to each other. Let ,

Then

= '.., y = x".
y
x,

Differentiating each side of this equatiqn with respect to

we have
and
since,

^ D,r
by Theorem V,
8,

^ Djx",

D^ = Dj>Djj = qy-^Dj),
it

follows that

qy-Wjj=px!^\
X>^

or

= -
qy
1

This last denominator has the value

(a?Y-^

=x

'.

DIFFERENTIATION OF ALGEBRAIC FUNCTIONS 43


Hence

We

see,

44
4.

CALCULUS
Plot the curve
a^

+ ^ = 81,
Show
that this curve
is

taking 1 cm. as the unit.


axes.

cut orthog-

onally by the bisectors of the angles

made by the coordinate

Differentiate the following functions


5.

M = Vl
u

X.

Ans.

5 V(l
Ans. 3
5-Vc^

6.

= 'Va'' 2ax + x\
= Vc* 3c2a!-|-3ca;2a^.

- xy -2

3-s/a-x
Ans.

7.

2cx + x^
(x>?.

=\
=

"^

9.
a;

= x-i/a + bx +
Ans.

10.

..<l^^^.

3 Va!2(l

- xy
^

JJ

Va-gj+Va +

a;

^^^
'

g^

a!2

+ aVa^-a;" Va^
ofi

13.

r=Va6.
lO^a;!'

16.

= V2
a;

a;.

DIFFERENTIATION OF ALGEBRAIC FUNCTIONS 45


20.

"-'^
V2aa!-a!2

21.

-'''

^-=^
(1

22.
24.

= iB(a2 x^y.
is 2.

23.

+ )' M = (6 V& +
^
)

!Pind the slope of the curve y

= x^

in the point

whose

abscissa
25.
26.

Ans. tan t

= .116.

If pv^*

c,

find

D^p.
find
D^j/-

If2/V5

= l+a;,

^ns.

^-=42a;Va!

.27.

Differentiate

y in two ways, where


xy

+ A:y = Zx,
'

and show that the


28. 29.

results agree.

The same, when

2/^

= 2 mx.

Show

that the curves

32/

= 2a; + a;y,

2?/

+ Sx +

j/^

aj'j/,

intersect at right angles at the origin.


30.

Find the angle at which the curves

2x = x'^xy + a^,
intersect at the origin.

x'

+ y* + 5x = 7y,
Ans. tan
<^

1.4.

CHAPTER

III

APPLICATIONS
1.

Tangents and Normals.

By

the tangent line, or simply


its points,

the tangent, to a curve at any one of

P,

is

meant the

straight

line
is

through

whose slope
point.

the same as

that of the curve at that

X
Fro. 9

Let the coordinates of F be denoted by (xq, y^. Now, the equation of the straight

line

through P, whose slope


y

is \, is
ai))-

ya = K^

If

On the other we denote

hand, the slope of the curve at any point


the value of this slope at
A.

is

Djf.
this

(a!o,

^o)

^7 {P^\,

will

be the desired value of

Hence the equation

of the tangent to the curve or

y=f(x)
at the point
(1)
(aJo,

F{x,y) =

y^

is

y-yo= (Ps)aip - Xo).


is

Since the normal


X', is

perpendicular to the tangent,

its slope,

the negative reciprocal of the slope of that line, or

X'=46

APPLICATIONS
Hence the equation
(2)

47
(x^, y^) is

of the normal to the curve at or

2^-2^o=-Tp-y(aJ-!eo)

x-Xo+iDjy)o-

{y

-yo)=0.

Example
curve

1.

To

find the equation of the tangent to the

y-a?

in the point x

= ^,y \-

Here

Hence the equation

of the tangent or

is

y i = ^(x ^)

3a!

4?/ 1 = 0.

Example

2.

Let the curve be an

ellipse

t+t=l
a^

Differentiating the equation as it stands,

we

get

a^

b^

ah)

Hence the equation

of the tangent

is

This can be transformed as follows


a^o2/

- aW = - 6% + ^'^^, + ahf^y = oV + 6V = a^l^,

V%fB

48

CALCULUS
EXERCISES

1.

Find the equation of the tangent of the curve


y

= a? X
where
it

at the origin of
X.

at the poiat

crosses the positive axis

Ans. x

+ y = 0;

2x y 2 = Q.

2.

Find the equation of the tangent and the normal of the


a;2

circle

+ y2 = 4
answer.

at the point (1,


3.

VS) and check your

Show

that the equation of the tangent to the hyperbola

at the point

{xq, yg) is

a"
4.

Show

that the equation of the tangent of the parabola


y'^

= 2mx = m( +
ai))-

at the point

(xo, yg) is

VoV
5.

Show

that the equation of the tangent of the parabola


y^

= m? 2mx

at the poiat

(xg, yg) is

ygy
6.

= m? m{x +Xg).

Show

that the equation of the tangent of the equilateral

hyperbola

xy =ia^
at the point
(xg, yg) is

y^ + x^ = 2a''.
7.

Find the equation of the tangent


a? + y^ = a%x y)

to the curve

at the origin.

Ans. x

= y.

APPLICATIONS
8.

49
by the
coordi-

Show

that the area of the triangle formed

nate axes and the tangent of the hyperbola

at

any point
9.

is

constant.

Find the equation of the tangent and the normal of the


.

curve
in the point distinct

a?

= a'y'
.

from the origin in which

it is

cut

by the

bisector of the positive coordinate axes.


10.

Show

that the portion of the tangent of the curve

xi
at

+ yi = a^
is

any

point,

intercepted between the coordinate axes,

constant.
11.

The parabola

y^

= 2 ax

cuts the curve

a^

3 axy

+ ?/'==
point.

at the origin

and at one other

Write down the equa-

tion of the tangent of each curve in the latter point.


12.

Show that

in the second point at


2.

the curves of the preceding question intersect an angle of 32 12'.

ft.

square a box

Maxima and Minima. Problem. From a piece of tinis to be made by cutting out equal squares

from the four corners and


bending up the sides. Detei;-. mine the dimensions of the box of this description which will hold the most.

^ ^'
32a;
}-

Solution.

Let
the

be
of

the

length of

side

the the

32a;
Fio. 10

square

removed;

then

dimensions of the box are as indicated in the diagrams,


noting the cubical content of the box by
u,

De-

we have

50
1)

CALCULUS
u = x(3-.2xy,

2)

= 9x 12a;'' + 4*'.
which makes u from the nature of the

The problem

is,

then, to find the value of x

as large as possible, x being restricted case to being positive and less than :

3)

0<a!<f.
The problem can be
1).

treated graphically

curve

We wish

to find the highest point

by plotting the on this curve.

It appears to be the point

for

which x

= ^,

u=

2,

since other values

of x

which have been tried lead


to smaller values of u.

The foregoing method


has the advantage that
is direct,
it

for it assumes no

knowledge of mathematics beyond curve plotting. It


has the disadvantage that curve plotting, even in the
simplest cases, is labori-

ous

and, furthermore,

we

have not really proved Fig. 11 that x = ^ is the best value. We have merely failed to find a better one. The Calculus supplies a means of meeting both the difficulties mentioned, and yielding a solution with the greatest ease. The problem is to find the highest poiat on the curve. At
this point, the tangent of the curve is evidently parallel to

the axis of

x.

tan T

= D^u,

must have the value

Consequently, the slope of the tangent, here

i.e.

APPLICATIONS
All

51

we need do, therefore, is to compute D^u, most conveniently from equation 2), and set the result equal to
:

Bji

= 9 - 24a! + 12a;2 = 0.
x,

On

solving this quadratic equation for


''
.

we

find

two

roots,

"J*

T-

Only one of
values for

these, however, lies within the range 3) of possible

x,

namely, the value x =

^,

and hence

this is the

required value.

EXERCISES
1.

Work

the foregoing problem


ft.

for'

the case that the tin

is

a rectangle 1 by 2
determine in this

Plot accurately the graph, taking 10 cm. as the unit, and

way what appears


an
inch.

to be the best value for x,

correct to one eighth of

Solve the problem by the Calculus, and show that the best

value for x
2.

is

.21132

ft.,

or 2.5359

in.

A farmer

wishes to fence
fence.

a straight
river

river,

off a rectangular pasture along one side of the pasture being formed by the

and requiring no
ft.

build a fence 1000

long.

He has barbed wire enough to What is the area of the largest


having a given perimeter,

pasture of the above description which he can fence off ?


3.

Show

that, of all rectangles

the square has the largest area.


4.

Show
Each
8
ft.

that, of all rectangles

having a given area, the

square has the least perimeter.


5.

side of a shelter tent is a rectangle

How must 1;he

tent be pitched so as to
inside ?

afford the largest

amount of room

The

~^^

ends are to be open.


Ans.
angle.

The angle along the

ridge-pole

must be a right

52
6.

CALCULUS

of their squares

Divide the number 12 into two parts such that the sum may be as small as possible. (What is meant is such a division as this one part might
:

be

and then the other would be would here be 16 + 64 = 80.)


4,
7.

8.

The sum

of the squares

Divide the number 8 into two such parts that the sum

of the cube of one part and twice the cube of the other may-be
as small as possible.
8. Divide the number 9 into two such parts that the product of one part by the square of the other may be as largd^^

as possible.
9.

of one part
10.

Divide the number 8 into two such parts that the product by the cube of the other may be as large as possible.

At noon, one ship, which is steaming east at the rate 2^ mUes an hour, is due south of a second ship steaming south at 16 miles an hour, the distance between them being 82 miles. If both ships hold their courses, show that they
of
will be nearest to each other at 2 p.m.
11.
If,

in the preceding problem, the second ship lies to


till

from noon

course at 16 miles an hour,

one o'clock, and then proceeds on her southerly when will the ships be nearest to

each other?
12.

Find the

least value of the function

y
cyi3.

= x^ + 6x + 10.
y

Ans.

1.

What

is

the greatest value of the function

= 3x a^

for positive values of


-14.

a;

-For

what value of x does the function

12Vg
l attain its greatest value ?

+ 4a;
Ans. x

=^

APPLICATIONS
15.
tive,

53

At

-what point of the interval a

<

a;

<

6,

a being

posi-

does the function

{x
attain its least value ?
16.

a){h

x)
Ans. X

= Va6.
by

Find the most advantageous length for a means of which to raise a weight of 490 lb. (see Fig. 13), q
if

lever,

the distance of the weight


1

C*^

from the fulcrum is 1 ft. and the


lever weighs 6
i^^
3.
lb.

\qx

to the foot.

Fig.

13

Continuation: Auxiliary Variables.

fact, usually,

happens that

It frequently,
to

in

it is

more convenient

formu-

late a

problem if more variables are introduced at the outset than are ultimately needed. The following examples will serve to illustrate the method.

Example

1.

greatest area

Let it be required to find the rectangle of which can be inscribed in a given circle.
It is evident that the area of the

rectangle will be small

tude
is

is

small and also

when its altiwhen its base

short.

largest for

Hence the area will be some intermediate shape.


rec-

Let u denote the area of the


tangle.
(1)

Then
M

= 4 xy.
then the rectangle In fact,

But X and y cannot both be chosen


Fio. 14

arbitrarily, for

will not in general be inseriptible in the given circle.


it is

clear

from the Pythagorean Theorem that x and y must


x^

satisfy the relation


(2)

+ y^ = A
;

We could now eliminate y between equations (1) and (2), thus obtaining u in terms of x alone and it is, indeed, im-

54

CALCULUS
is

portant to think of this elimination as performed, for there

only one independent variable ia the problem. The graph of u, regarded as a function of x, starts at the origia, rises as x
increases, but finally

comes back to the axis of x again when x a. All this we read off, either from the meaning of u and x in the problem or from _* equations (1) and (2).
It is better, however, in practice not to elimi-

J.

respefct
(1),

nate
(2)

with

y, but to differentiate equa;tions (1) and ta x as they -Itand, and then set D,,u = 0.
"

Thus from
and from

")

D^u
(2),

= 4:(^+xD,y)=0, 2a; + 2 yD,y = 0.


we
see that

From

the second of these equations

y
Substituting for Dj) this value in the
first

equation,

we

get

y y

=0

or

y^

= a?.
it

Since x and y are both positive numbers,

follows that

= x.
is

Hence the maximum rectangle

a square.

<X

EXERCISES
1.

Work

the same problem for an ellipse, instead of a

circle.
2.

Work

the problem for the case of a variable rectangle

inscribed in a fixed equilateral triangle.

Example

2.

To

find the

most economical dimensions for a

tin dipper, to hold

pint.

APPLICATIONS

55

sible,

Here, the amount of tin required is to be as small as posthe content of the dipper being given. Let u denote the

surface,
a)

measured in square inches.


M

Then

= 2irrh +

irr^.

But r and h caimot both be chosen arbitrarily, for then the dipper would not in general hold a pint. Fig. If denotes the given volume, measured in cubic inches, then, since this volume can also be expressed as

16

jrr^,

we have
b)
irr^A

V.

Differentiating equation o) with respect to r


D,ii

and setting

= 0, we have

D,u = ir\2h
or
c)

+ 2rDJi + 2 r = 0,
j

h
DifEerentiating 6)

+ rD,h + r = 0.
get

we

d)

T\2rh
r cannot

+ rWM=0.
as well as

Now,

in this problem,
r,

last equation
e)

through by

and so we may divide by ^

this

2h
It remains to eliminate

+ rD,h, = 0.
D^h between equations
c)

and

e).

From

e),

Substituting this value of D,A in

c),

we
or

find

/)

h2h + r = 0,
of the dipper

= h.

Hence the depth

must

just equal its radius.

Just what have we done here ? The steps we Discussion. have taken are suggested clearly enough by the solution of

56
Example
h, as
1.

CALCULUS

We

have chosen one of the two variables, r and

the independent variable (here, r); differentiated the function u, which is to be made a minimum, with respect to r,

and
6),

set

D,m

= 0.

Then we

differentiated the second equation


r,

likewise with respect to


it all

eliminated D,u, and solved.


is

But what does


Just this
function of
:

mean ?

What

behind

it all.?
is

the quantity m,
r.

the nature of the case,

For,

when

to r is given

any positive value, a

Now,
i.e.

dipper can be constructed which will fulfill the requirements. if r is very large, we shall have a shallow pan, and evidently the amount of tin required to

make

it will

be large

u will also have a large value.


if

shall then have a high cylinder a pipe. Is it clear that m, the surface, will be large in this case, too? I fear not, for it' is purely a relative question as to how high such a pipe must be to hold a pint, and I see no way of guessing intelligently. By means of equation 6), however, we see that

But what

is

small ?

We

of minute cross sections,

i.e.

and
Ml

if

we

substitute this value


/

ma), we

get

F
Ttr^

2F
r

From this last formula it is clear that, when r is small, u actually is large. The graph of u, regarded as a function of
!^

r, is

therefore ia character as
figure.

O
and
also very high

shown by the accompanying


the axis of
r
is
r,

It is a continuous curve lying above

very high when r is small, It has, therefore, a lowest point, and for this value of r, the area u of the dipper But at this lowest point the slope of the curve, will be least. Thus we see, first, that we have a genui),M, has the value 0.

when

large.

APPLICATIONS
ine

57

minimum problem

there
c)

is

actually a dipper of small-

and d) must hold, and since from these equations it follows by elimination that r = ft, there is only one such dipper, and its radius is equal to its altitude. The problem is, then, completely solved.
est area.

Secondly, equations

We inquired merely for the shape

of the dipper.

If the size
b)

had been asked for, too, it could be found by solving for r and h, and expressiag Via cubic inches
:

and /)

F= ^^ = 28.875,
fl-r-s

= 28.87,

'

= 2.095.
its

It can

happen in practice that a function attaias


Usuneces-

greatest

or its least value at the end of the interval. derivative does not have to vanish.
ally,

In that case, the

the facts are patent, and so no special


is

investigation

needed.

But

it is

sary to assure oneself that a given problem

which looks like one of the above does not come under this head, and this is done, as in the cases discussed in the text, by showing that near the ends of the interval the values of the function are larger, for a

minimum

problem, than for values well

withia the interval.

EXERCISE
Discuss in a similar manner the best shape for a tomato can
is to hold a quart. Here, the tin for the top must also be figured in. Show that the height of such a can should be equal to the diameter of the base. As to the size of the can,

which

its

height should be 4.19 General Remark.

in.

It

might seem as

if

the method used in

the solution of the above problems were likely to be insecure,


since the graph of such a function u might, in the very next problem, look like the accompanying figure. In such a case, there would be several values of x, for each of which D^u 0,

58

CALCULUS
to take.

and we should not know which one

this case does not arise in practice,

at

least,

Curiously enough, I have never

come across a physical problem which In problems like the above, there must be at least and when one X for which Djt =
led to this difficulty.
;

we
Fio. 19

solve a given problem,

we actually
the con-

find only one


dition.

x which
is

fulfills

Thus there

no ambiguity.

EXERCISES
1.

A 300-gallon tank
and
is to

vertical sides,

is to be built with a square base and be lined with copper. Find the most

economical proportions.
Ans. The length and breadth must each be double the
height.
2.

Find the cylinder of greatest volume which


Its altitude is one-third that of the cone.

T~
ji
I

can be inscribed in a given cone of revolution.


Ans.
3.

What

is

the cylinder of greatest convex

I_
^"
Fio. 20

surface that can be inscribed in the

same cone ?

Ans. Its altitude


4.

is

half that of the cone.

Of

all

the cones which can be inscribed in a given sphere,

find the one

whose

lateral area is greatest.

Ans. Its altitude exceeds the radius of the sphere by 33^ %


of that radius.
5.

Find the volume of the greatest cone of revolution which

can be inscribed in a given sphere.


6. If the top and bottom of the tomato can considered in the Exercise of the text are cut from sheets of tin so that a regular hexagon is used up each time, the waste being a total

loss,

what wiD then be the most economical proportions

for

the can ?

APPLICATIONS
7.

59
its

If the strength, of a

beam

is

proportional to

breadth

and

to the square of its depth, find the shape of the strongest

beam

that can be cut from a circular log.

Ans. The ratio of depth to breadth

is

V2.

its

8. Assuming that the stiffness of a beam is proportional to breadth and to the cube of its depth, find the dimensions of the stiffiest beam that can be sawed from a log one foot in

diameter.
9.

What What

is

the shortest distance from the poiut (10, 0) to '


w2 a

the parabola
10.

= 4a;9 /

points of the curve

are nearest
11.

(4,

0) ?
is to be made of a long rectangular-shaped by bending up the edges so as to give a rec-

trough

piece of copper

tangular cross-section.
that
its

How

carrying capacity

may

deep should it be made, in order be as great as possible ?


of water to be given

12.

30 C.

Assuming the density by the formula


p

from

0 to

= po(l + ca + pt^ + yf),


t

where and

po denotes the density at freezing,

the temperature,

- 6.53 X 10-6, a=5.30xl0-^ /8 = show that the maximum density occurs at


13. E!

y= 1.4x10-8, = 4.08.
ellipse

Tangents are drawn to the arc of the

+^=1
off

which lies in the first quadrant. Which one of them cuts from that quadrant the triangle of smallest area ?
14.

Work

the same problem for the parabola


y^

= a^ iax.

60
15.

CALCULUS
Show that,
of all circular sectors having the same perim-

eter, that

two straight
4.

one has the largest area for which the sum of the sides is equal to the curved side.

Increasing

affords a simple

and Decreasing Functions. The Calculus means of determining whether a function is inSince the slope of the

creasrag or decreasing as the independent variable increases.

graph

is

^iven by

we
as

see that

D^, when D,y

is positive,

iacreases,
is

y iacreases but
negative,
as

o
creases.

o
"Fia. 21

when Dj/
y

decreases

iais

Figure 21 shows the graph ia general when

D^

positive.

In each figure both x and y have been taken as positive. is said above in the text is equally true when one or both of these variables are negative; for the words increase

But what

and decrease as here used mean algebraic, not numerical, iacrease or decrease. Thus if the temperature is ten degrees below zero (i.e. 10) and it changes to eight below ( 8), we say
the temperature has risen.
If

we measure the time t, in hours


from noon, then 10 a.m. will correspond to t = 2. Let u
t

= lZ

denote the temperature, measured iu degrees. Then a tem^o. 22 perature chart for 24 hours from midnight to midnight might look like the accompanying figure. At any instant, t t', for which the slope of the curve, D,u, is positive, the temperature is rising, no matter whether the thermometer ia above zero or below, and no matter whether

is

positive or negative

and

similarly,

when

Z>,m is negative,

the temperature

is falling.

Again, suppose the amount of business a department store

APPLICATIONS

61

does in a year, as represented by the net receipts each, day, be


plotted as a curve
(y

= receipts,

time, measured lq days), the curve being

measured in dollars x = smoothed ia the


;

usual way.
is

positive

ness of

D^ <
We
.the

Then a point of the curve at -which the derivative D^y > 0) indicates that, at that time, the busithe firm was increasing whereas a point at which means that the business was falling off.
(i.e.
;

proof of which

can state the result in the form of a general theorem, is given by inspection of the figure (Fig. 21)
figure,

and the other forms of the


discussion.

brought out in the above

Theorem
(a)
(b)

When x
'

increases, then

if if
'

Djf

> 0, D^y < 0,


its

y increases;

y decreases.

Application.

As an

application consider the condition that

a curve y =f{x) have


Fig. 23.

concave side turned upward, as ia


is

The slope
:

of the curve

a function of x

tan T
For,

= ^(w).
given, a point of

when x
curve,

is

the

and

hence

also

the
is

*-*

FlQ. 23

slope of the curve at this point,

Consider the tangent line at a variable point P. as tracing out the curve and carrying the tangent along with it, the tangent will turn in the counter
determined.
If

we think

of

clock-wise sense, the slope thus increasing algebraically as x

concave upward. And conx increases, the tangent will turn in the counter clock-wise sense and the curve will be concave upward. Now by the above theorem, when
increases,

whenever the curve

is

versely, if the slope increases as

Z>^tanT>0,
tauT increases as x increases. Hence the curve upward, when D, tan t is positive and conversely.
;

is

concave

62
The
of
y.

CALCULUS
derivative

D^tanr

is

This

is

called the second derivative of y,

the derivative of the derivative and is denoted


.
-r,

as follows

^
.

(read

"

D x second

of

").*

The

test for the curve's being concave

downward

is

obtained

in a similar manner,

and thus we are led

to the following

important theorem.

Test fob a Cukve's being Concave Upward, etc.


curve
,,
,

The

is

CONCAVE UPWARD CONCAVE DOWNWARD

whsn
when

D^ > D^ <
A

0.

point at which
changes

the curve

from being concave upward and becomes concave

downward
versa)
is

(or

vice

called

poirU
Fig. 24

of

inflection.

Since

D^y

changes

sign at such a point,


this

function will' necessarily,

if

continuous, vanish there.

Hence

A necessary condition for a point of inflection is that


BJ'y

= 0.

Example.

Consider the curve

= 3? 3x.
is

derivative

* The derivative of the second derivative, Dx^DJiy), and is written D^^, and so on.

called the third

APPLICATIONS
Its slope at

63

any point

is

given

.by

the equation

The second derivative


D/2/

of y with

respect to x has the value

= 6a;.
is

Thus we
values of

see that this curve

concave upward for


x,

all positive

ward

for all negative values.


it

character

and concave down^la. 25 In is as shown in the accompanying figure.


EXERCISES

For what values of x are the following functions increasing ? For what values decreasing ?
1.
2/

= 4 2a;2.
when x

2.

Ans.
3.

= x^ 2x + Z. Increasing, when > 1 decreasing,


y
a;
;

<

1.

2/

= 5 + 12x
as

a;2.

4.

y=3fi
Increasihg,
a;

21x + l.
and when
a;

Ans.
ing,

when
3.

> 3,

< 3

decreas-

when
y

3< <

^.

= 5 -\-&x 3?.
y

6.

= x x".

7.

ofi

^x^ + 12x l.

In what intervals are the following curves concave upward


in what,
8.

downward ?

Ans. Concave

= a^ 3a;2 + 7a; 5. upward, when > 1 concave


y
a;
;

downward,

when

a;

<

1.

64
9.
2/

CALCULUS

11.

13.
15.

= 15 + 8a; + 3a!= 10. y = a^ Qx^ x 1. = 3 9x + 24:x^ia^. 12. y = 2a>> X*. y 4.0^ 6x + 11. 14. y ==-121x + 7x^ x\ y= y = 13 + 23x-24:X'' + 12a^-x^.
05*.

as*

In the early work of plotting curves 5. Curve Tracing. from their equations the only way we had of finding out what the graph of a function looked like was by computiug a large

number
scarcely

of its points.

We

are

now

in possession of powerful

methods for determining the character of the graph with

any computation. I"or, first, we can find the slope of any point; and, secondly, we can determine in what intervals the curve is concave upward, in what concave downward.*
the curve at

Example.
(1)

Let

it

be required to plot the curve

3y
Determine
first its

= x^- 3x^ + 1.
slope at

a)
(2)

any point

3n^ = 3x^-6x,

Djj

= v? 2x.

* There are two great applications of the graphical representation of a One is quantitaMvie, the other, quaUtative. By the first I mean the use of the graph as a table, for actual computation. Thus in the use
function.

have a graph of the function y = logio a between 1 and 10 for by means of such a graph the student can read ofi the logarithms he is using, correct to two or three significant figures, and so obtain a check on his numerical vork. Therfe is, however, a second large and important class of problems, in which the character of a function is the important thing, a minute determination of its values being in general irrelevant. A case in point is the determination of the number of roots of an algeof logarithms it is desirable to

drawn accurately

for values of x

braic equation, e.g.

ss

_ ^2 - 4 x

-t-

= +

0,

Here,

we

plot the curve


it

y ^^.^

_xi -

4:x

and inquire where


adequate to

cuts the axis of x.

For

this

purpose

it is

altogether

know the

character of the curve, and for treating this problem

the methods of the present paragraph yield a powerful instrument.

APPLICATIONS
It is always useful to to the curve
is

65

know

the points at which the tangent

parallel to the axis of x.

These are obtaiued


(2)

hy

setting

D^y =

and

solving.
a;2

Thus we get from


0.

the

equation

2a; =
and

The

roots of this equation are

=2

Now determine accurately


plot them,

the points having these abscissas,

and draw the tangents there


y

We

do

not

yet
the
its

know whether
curve lies above

tangent in one of
these points, or be-

low its tangent it might even cross


;

its

tangent, for the


Fib. 26

point might be a point of inflection.


aid of the second derivative.
6)

These questions will


derivative

all

be answered by

Compute the second


DJh/

= 2a; - 2 = 2(!B - 1).


when x
is

We
tive

see that

it is

positive

greater than 1 and nega-

when X

is less

than 1

-D.V

>

66

CALCULUS

coordinates accurately, determine the slope of the curve there,

and draw accurately the tangent


yU=.
This
draw.
liue
is

there.

= -i;
is

-D^U=i

1.

the last of the important tangents which

Since the curve

X = 1, must be in character as indicated. We see, then, that the axis of x between and 1, and again ta the right poiut a; = 1 and it cuts that axis a third time to the
;

we need to concave upward to the right of the and concave downward to the left of that line, it
it

cuts

of the
left of

the origin.

These last two points can be located more accurately by computing the function for a few simple values of x.

hence the curve cuts the axis of x between

a;

= 2 and = 3.
a;

2/L=-i=-3;
hence the curve cuts the axis between a; = and a; = Incidentally we have shown that the cubic equation
a?
1.

3x^ + 1 =
we have
located each between two

has three real roots, and


successive integers.

EXERCISES
Discuss in a similar manner the following curves.
ticular
:

In par-

a)
if

such

Determine the points at which the tangent is horizontal, exist, and draw the tangent at each of these points

6) Determine the intervals in which the curve is concave upward, and those in which it is concave downward
c)

Determine the points' of

inflection, if

any

exist,

and draw

the tangent in each of these points

APPLICATIONS
d)

67

Draw

in the curve.*
it is

In most cases
1. 2.
3.

desirable to take 2 em, as the imit.

4.
5. 6.

= x^ + 3x^ 2. y = a^3x + l. y=iifl +3x + l. 6y = 2a^-3x^-12x + 6. 6y = 2d' + 3x^-12x y = a? + x^ + x + l.


y

4:.

Suggestion.

Show

that the derivative has no real roots and

hence, being continuous, never changes sign.


7. 8. 9.

12y = 4:a?-6x^
y

+ 12x 9.

10.

11.
12.

14. 16.
18.

19.

= 2x' x a^. 12y = 4a!8 + 18a!2 + 27a; + 12. = l 4a! + 6c(;2_3ffi3. y = l + 2x + x^ 6a!2 + 8. 13. y = a^ + 1. 42/ = 15. y = X + x^. y = x a^. 17. y=2xl*~xK y= + y = 3a!5 + 6a!' + 15a! + 2. 60y = 23^ + 16a^ + GOaj^ 30.
3/
0(?.

a!*

^ix"^

a!*

a;2.

6.

Relative

Maxima and Minima.


2/ =/(')

Points of Inflection.

function
(1)

inflection, the material

* Since a curve separates very slowly from its tangent near a point of graph of the curve must necessarily coincide with the material graph of the tangent for some little distance.

68
is

CALCULUS
maximum at a point x = Xoii its value at Xq is any other point in the neighborhood of a%. But
not represent the largest value of the

said to have a

larger than at

such a

maximum need

function in the complete interval a

^ ^ 6,
a;

as

is

shown

by-

Fig. 27,
it is

and for

this reason

called a relative maxi-

mum,
a

in distinction from
absolute

maximum maximorum,
an
A.

X
a;=a
x x^

or

maximum.
definition

x=b

holds for a minimum, the word " larger " merely being replaced by " smaller." It is obvious that a characteristic feature of a maximum is
that the tangent there is parallel to the axis of x, the curve being concave downward. Similarly for a minimum, the curve here beiag concave upward. Hence the foUowiag

^-

similar

Test fok a
(a)

Maximum ob a Minimum.
[Z)^]_=0,
a maximum for x

If

[i).^]_<0,

the function has


(6)
it

= Xo;

if

iDjy-]^=0,
condition
is sufficient,

[i).^]_>0,

has a minimum.

The

but not necessary

cf 7.
.

Example.

Here
and hence

3x'' + l. D^ = Gaj^- 6a; = 6a;(a!2- l)(a!= + 1), = 1, 0, 1. for D^y =


Let
y

!ic^

a;

Thus the necessary condition for a maximum or a minimum, Dj/ = 0, is satisfied at each of the points a; = 1, 0, 1. To complete the determination, if possible, compute the
second derivative,

j)^^^s0<^-6,
sign at each of these points

and determine

its

APPLICATIONS
[i)j^]j=_i
[D,^2/],_o

69

= 24 > 0,

.-.

= l gives a minimum;
gives a

= 6<0,
=
24

..x=
.-.

maximum
minimum.

[PJh/],,^

> 0,

x=

1 gives a

Points of Inflection. A point of inflection is characterized geometrically by the phenomenon that, as a point describes

the curve, the tangent at


rection and,

P
y

ceases rotating in the one di-

tumiag back,

be-

gins to rotate in the opposite


direction.

Hence the slope


t,

of the curve, tan

has either
at

maximum

or a

minimum

Fio. 28

a point of

inflection.
if

Conversely,

tan r has a
will

maximum
tion.

or a

minimum, the curve


is

have a point of

inflec-

For, suppose tan t

at a
Xq,

maximum when x
increases, tan t,

= Xg.

Then
is

as X, starting with the value

i.e.

the slope
con-

of the curve, decreases algebraically,

and so the curve

cave

downward

to the right of

a^.

On

the other hand, as x

and so the curve is concave upward to the left of X(,. Now, we have just obtained a theorem W'hich insures us a maximum or a minimum in the case of any function which
decreases, tan t also decreases,
satisfies

the conditions of the theorem.

If, then,

we

choose

as that function, tanr, the theorem tells us that tanr

wUl

surely be at a

maximum

or a

minimum if
DJ^

D,tanr = 0,
Hence, remembering that
tan T

tsmr^O.

= D^y,
If

we

obtain the following

Test fok a Point of Inflection.

the curve has

a point of inflection at

Xf,.

70
This
test, like

CALCULUS
the foregoLag for a maxionim or a
;

minimum,

is sufficient,

but not necessary

cf. 7.

Example.

Let

Then

= + 2a!S-12a;2 + 14a; 1. 27D,y = + 6x' 2ix + U, 27 DJh/ = 12a:'' + 12 - 24 = 12(a! - l)(a; + 2), 2rZ),52/ = 12(2a; + l).
.

272/

ffi*

4:afi

a!

Setting
S1I1C6

D/y

= 0,

we

get the points


0,

a;

= 1 and x = 2. And
36
=jfc

27[A'2/]^= 36

=ifc

27[X>,'2^]_.,

0,

we see The

that both of these points are points of inflection.


s,lope of

the curve in these points

is

given by the

equations

27[i?^]^i=0,

27[Z)^]_3=54.
to the axis of
is 2.

Hence the curve


points
;

is parallel

x at the

first

of these

at the second its slope

EXERCISES
Test the following curves for maxima, minima, and points
of inflectioil, and * determine the slope of the curve in each

point of inflection.
1.

y y

=:

4:0^ 15x^+12 x+1.!i^

3.

6y

afi

Ssc'+Sa^l.
.

Z.

+ a^ +

iifi.

4.

y=(x-iy{x+2y.
^-

y=-, 2

+ 3a?
test

6.

y=(l x^^
Deduce a

7.

for "distinguishing be-

tween two such points of


Fig. 29.

inflection as those indicated

in

APPLICATIONS

71

7. Necessary and Sufficient Conditions. In order to understand the nature of the tests obtained in the foregoing paragraph
it is

essential that the student

ing of a necessary condition and of a

have clearly in miad the meanLet sufficient condition.

us illustrate these ideas by means of some simple examples.


a)

A
A
its

necessary condition that a quadrilateral be a square

is

that its angles be right angles.

But the condition


it.

is

obviously

not sufficient
6)

all

rectangles also satisfy

sufficient

condition that a quadrilateral be a square

is

that

angles be right angles and each side be 4


is

in. long.

But the condition


be 6
c)
iu.

obviously not necessary

the sides might

long.

necessary and sufficient condition that a quadrilateral


its

be a square is that mutually equal.

angles be right angles and

its sides

be

As a further illustration consider the following. It is a well-known fact about whole numbers that if the sum of the digits of a whole number is divisible by 3, the number is divisAlso, if the sum of the digits of a ible by 3 and conversely. whole number is divisible by 9, the number is divisible by 9
;

and conversely.
i)

Hence we can say

A
is

necessary condition that a whole

by 9
ii)

that the
is

sum

of its digits be divisible

number be divisible by 3. But the number be divisible by 9. But the

condition

not

sufficient.

A
is

sufficient

condition that a whole

by 3

that the
is

sum

of its digits be divisible

condition
iii)

not necessary.

necessary

and

sufficient
is

condition that a whole


that the
t

numbe

ber be divisible by 3 (or 9) divisible by 3 (or 9).

sum

of

its digits

Turning now

to the considerations of 6,

we

see that a

necessary condition for a

miaimum

is

that

72

CALCULUS
a;

at the point in question,


sufficient.

= a^.

But

this condition is not

When
or
it

it

is

fulfilled,

the function

may have

maximum,
tangent.

may have a

point of inflection with horizontal

On

the other hand, the condition

is sufficient

for a

minimum.

But

it is

not necessary.

Thus

the function
(1)

y=x^
minimum when x
of course fulfilled

ohviously has a
tion,

0.

The necessary

condi-

Dj/

= 0, is

But here
is

Z>,=2/

= 12

a;^,

and

[D^^']^

not positive
is

it is 0.

Again, as was shown in


of inflection

4,

a necessary condition for a point

that

DJhf
at that point.

=
Thus
in

But

this condition is not sufficient.


(1)

the case of the curve


origin.

this

condition

is

fulfilled at the

But the
It

origin is not a point of inflection.


to the student that such tests are

Remark.

may seem

unsatisfactory since they do not apply to all cases and thus

appear to be incomplete. But their very strength lies in the fact that they do not tell the truth in too much detail. They single out the big thing in the cases which arise in practice and yield criteria which can be applied with ease to the great
majority of these cases.
8.

Velocity; Bates.

By

the average velocity with which a


t is

point moves for a given length of time


8 traversed

meant the distance

divided by the time

average velocity

=-

APPLICATIONS
Thus a railroad
of 15/-|^= 30
train

73

stations 16 miles apart in half

which covers the distance between two an hour has an average speed

fast

in question is moving sometimes and sometimes slowly, we can describe its speed approximately at any given instant by considering a short interval of time immediately succeeding the instant t^ in question, and

mUes an hour. When, however, the point

taking the average velocity for this short interval.

For example, a stone dropped from


the law
To-find
(1)
.

rest falls according to

how fast

it is

going after the lapse of


So

to

seconds.

Here

= 16o'.
t'

A
(2)

little later,
fall,

at the end of

seconds from the beginning of

the

s'

= 16'2
t'

and the average velocity for the interval of


(3)

to

seconds

is

^j^^
t

ft.

per second.

to

Let us consider this average velocity, in particular, after the


lapse of 1 second
to

= 1,
^
t'

So

= 16.
.

Let the interval of time,


s'

to,

be -^

sec.

Then

= 16 X = 19.36, _ 3^ ^ 336 a second. s;^-So .1


1.1==

ft.

t'

to

Thus the average velocity


second.

for one-tenth of a second immediis

ately succeeding the end of the first second of fall

33.6

ft.

Next, let the interval of time be j^ sec. Then a similar computation gives, to three significant figures

^^^^ = 32.2
t'

ft.

a second.

to

74

CALCULUS
the interval
32.0
ft.

And when
velocity
is

is

taken as x^^nr

^^-'

*^ average

a second.

These numerical results iadicate that we can get at the speed of the stone at any desired instant to any desired degree of accuracy by direct computation we need only to reckon
;

out the average velocity for a sufB.eiently short interval of

time succeeding the iastant in question. We can proceed in a similar manner when a point moves according to any given law. Can we not, however, by the aid

and at the same time make precise exactly what is meant by the velocity If we regard the interval of the point at a given instant? of time- 1' io as an increment of the variable t and write ' = Ai, then s' So = As will represent the corresponding <o increment in the function, and thus we have
of the Calculus avoid the labor of the computations

average velocity

Then the average


this limit

Now allow

At to approach

as its limit.

velocity will in general approach a limit,

and

we

take

as the definition of the

velocity, v, at the instant

lim (average velocity from t=tgto

= actual velocity * at instant


or

= f) =
t
to,

v
it

= lim = D,s.
At^ At

Hence

derivative

appears that the velocity of a point is the timeof the space it has traveled. In the ease of a

freely falling

body

this velocity is

= D,s = 32

1.

In the foregoing
actually traversed
let s

definition, s has

been taken as the distance


point, P.

by the moving

More

generally,

is denote the length of the arc of the curve on which moving, s being measured from an arbitrarily chosen fixed

* Sometimes called the instantarieous velocity.

APPLICATIONS
point of
tliat curve.

75

Either direction along the curve


s.

may

be

chosen as the positive sense for

Thus, in the case of a

freely falling body, s might be taken as the distance of the body above the ground. If h denotes the initial distance, then

at

where s' denotes the distance actually traversed by any given instant. Hence
D,s
or

P
,

a'

+ D/ = 0,
= D,s'.

^s
Fig. 30
is

D,s

Here

D^ gives

numerically the value of the velocity, but D^s

a negative quantity.

We

will, accordingly,

extend the conception of velocity,

defining the velocity v of the point as D,s V

= DfS.
will

Thus the numerical value of v or D,s


case s increases with the time, DfS
is

speed, or the value of the velocity in the earlier sense.

always give the In

positive

and represents
is

the speed.
tive,

If,

however,

s decreases

with the time, D,s

nega-

and the

velocity, v, is therefore here negative, the speed

now being given by

or X),s. Speed=|u| =

In

all eases,

\D,s\.

Example.
that

velocity of 96

Let a body be projected upward with an initial Assuming from Physics the law ft. a second.
s

= 96f-162,
end of 2
sec.

find its velocity

a) at the

6) at the end of 5 sec.

Solution.

By

definition, the velocity at

any instant

is

Hence
a)
6)

= D,s = 96 -32t.
v\^,=^. ^V
l^ = -64.

76

CALCULUS
of these results
is that,

The meaning

at each of the two


ft.

instants, the speed is the same, namely, 64

the height above the ground


s

is

also seen to be the

a second (and same,


is
t

128 ft.). But when t 2, DiS is positive ; hence s creasing with the time and the body is rising. When
is

in-

= 5,

D,s

negative

hence

s is

decreasing with the time, and the

body

is

descending.

Rates.

Consider any length or distance,


is

r,

which

is

chang-

ing with the time, and so

denote the value of


will be

a function of the time. Let j-q at a given Instant, t = to, and let r' be the
t
t

value of r at a later instant,


r'

t'.

Then the
t'

increase in r

ro = Ar and that in

will be

= ^t.
ta

Thus
t

in
Jj,

the interval of time of At seconds succeeding the instant

average rate of increase of r

At
rate of
this limit

Now let

At approach

as its limit.

Then the average


and
t^

increase will in general approach a limit,

we take

as the definition of the rate of increase of r at the instant

lim (average rate of increase from

{^

to
f

t')

= actual rate of increase


A=o

at instant

<o

= lim = Dir.
At
is
r.

In other words, the rate at which r


stant
is

increasing at

any

in-

defined as the time-derivative of


decreasing, D,r

If r

is

wUl be a negative quantity

versely, if D,r is negative, then r is decreasing.

and conIn either ease,


;
;

the numerical value of D^r gives the rate of change of r just as, in the case of velocities, the numerical value of D,s gives
the speed.

More

generally, instead of

r,

we may have any

physical

quantity, u, as an area or a volume or the current in an electric


circuit or the

number

of calories in a given body.

APPLICATIONS
In
of M
all

77

these cases, the rate at which


i.e.

as the time-derivative of u,
is
I

as D,u

u is increasing ia defined and the rate of change

D,u

|.

Example. At noon, one ship is steaming east at the rate of 18 miles an hour, and a second ship, 40 miles north of the first, At what is steaming south at the rate of 20 miles an hour.
rate are they separating
Solution.

from each other at one

o'clock ?

78

CALCULUS

Let the student determine how long they will continue to approach each other, and what the shortest distance between

them

will he.

Remark. It is important for the student to reflect on the method of solution of this problem, since it is typical. We were asked to find the rate of recession at just one instant, t=l. We began by determining the rate of recession generally, i.e. Having solved the general for an arbitrary instant, t=t.
problem^

we

then, as the last step


f

-in

the process, brought into


for,

play the specific value of


t

which alone we cared


this

namely,

= l.
The student
will

meet

method again and

again,

in

integration, in mechanics, in series, etc.

We
:

can formulate

the foregoing remark suggestively as follows

By means

of

the Calculus acted on by

we

can often determine a particular physical


it

quantity, like a velocity, an area, or the time

takes a body,

known
is

forces, to reach a certain position.

The

method

consists in first determining

a function, whereby the


;

general problem

solved for the variable case

and

then, as

the last step in the process, the special numerical values with

which alone the proposed question


into play.

is

concerned, are brought

EXERCISES
1. The height of a stone thrown bytheformula:

vertically

upward

is

given

s=m-16t^.
its

When

it

velocity for the next

has been rising for one second, find (a) sec. (6) for the next

average
;

^^ sec.
(d)

(c)

its
it

actual velocity at the end of the first second


will rise.

how high
;

Ans. (a) 14.4 a second


2.
;

ft.
ft.

a second

(6)

15.84

ft.

a second

(c)

16

ft.

(d)

36
is

One ship
is sailing

80 miles due south of another ship at noon,

and

north at the rate of 10 miles an hour.

The

APPLICATIONS
second ship
sails

79
Will

west at the rate of 12 miles an hour.

the ships be approaching each other or receding from ^each


-will be the rate at which the dischanging at that time ? How long will they continue to approach each other ?

other at 2 o'clock ? tance between

What

them

is

3. If two ships start abreast half a mile apart and sail due north at the rates of 9 miles an hour and 12 miles an hour, how far apart will they be at the end of half an hour ? How fast will they be receding at that time ?

4. Two ships are steaming east, one at the rate of 18 miles an hour, the other at the rate of 24 miles an hour. At noon, one is 50 miles south of the other. How fast are they sepa-

rating at 7 P.M. ?
5.

ladder 20

ft.

long rests against a house.

A man

takes hold of the lower end of the ladder and walks off with
at the uniform rate of 2 ft. a second. How fast is the upper end of the ladder coming down the wall when the man is 4 ft. from the house ?
it

If the kite

150 ft. high and there are 250 ft. of cord out. moves horizontally at the rate of 4 m. an hour directly away from the person who is flying it, how fast is the cord being paid out ? Ans. 3^ m. an hour.
6.

A kite is

7.

stone

is

dropped into a placid pond and sends out


If the radius of the
ft.

a series of concentric circular ripples.

outer ripple increases steadily at the rate of 6

a second, a second.

how
8.

rapidly

is

the area of the water disturbed increasing at

the end of 2 sec. ?

Ans. 452
is

sq. ft.

spherical raindrop

gathering moisture at such a

rate that the radius is steadily increasing at the rate of 1

mm.

a minute. How fast is the volume of the drop increasing when the diameter is 2 mm. ?

A man is walking over a bridge at the rate of 4 miles an and a boat passes under the bridge immediately below him rowing 8 miles an hour. The bridge is 20 ft. above the
9.

hour,

80
boat.

CALCULUS

How

fast are the boat

and the man separating 3 min-

utes later ?

Suggestion.
this

The student should make a space model for problem by means, for example, of the edge of a table, a crack La the floor, and a string or by two edges of the room which do not intersect, and a string. He should then make a drawing of his model such as is here indicated.
;

10.

locomotive

running

30
the

miles an hour over a high bridge

'"
track.

dislodges a stone lying, near

The

stone begins to fall just as the locomotive passes


it lay.

the point where

How

fast are the stone

and the

loco-

motive separating 2
11.

sec. later ?

40

ft.

Solve the same problem if the stone drops from a point from the track and at the same level, when the locomo-

tive passes.

A lamp-post is distant 10 ft. from a street crossing and from the houses on the opposite side of the street. A man crosses the street, walking on the crossing at the rate of 4 miles an hour. How fast is his shadow moving along the walls of the houses when he is halfway over ?
12.

60

ft.

* BScHEB, Plane Analytic Geometry, p. 230.

CHAPTER IV
INFINITESIMALS AND DIFFERENTIALS
1.

Infinitesimals.

An

infinitesimal is

a variable which

it is

desirable to consider only for values numerically small and

which,

when

the formulation of the problem in hand has prois

gressed to a certain stage,

allowed to approach

as its limit.

Thus in the problem of


(1)

differentiation, or finding the limit

lim^=Z>^,
and Ay are infinitesimals
;

Aa!

for

we

allow Aa; to approach


0..

as its limit, and then

Ay

also approaches

Again,

if

we

denote the value of the difference Ay/ Ax

Dj/

by
(2)

so that

f-2>^ = Ao;
e

e,

then

is

an

infinitesimal.

For,

when

Aa;

approaches

0,

the

left-hand side of equation (2) approaches 0, and so e is a varias its limit, i.e. an infinitesimal. able which approaches

Principal Infinitesimal,

When we

are dealing with a

num-

ber of infinitesimals,

a,

p, y, etc., it is usually possible to

choose any one of them as the independent variable, the others

then becomiag functions of it, or dependent variables. That infinitesimal which is chosen as the independent variable is
called the principal infinitesimal.

Thus,

if

the infinitesimals are a and

jS,

and

if

81

82
it is it is

CALCULUS
natural to choose as the principal infinitesimal.
perfectly possible to take
j8

But

as the principal infinitesimal.


is

then becomes the dependent variable, and


terms of
(4) ^'

expressed ia

p by

solving equation (3) for

a.

= ^

2-3/3

Order of Infinitesimals. We are going to separate infinitesimals into classes, according to the relative speed with vrhich they approach 0. Suppose we let a. set the pace, taking on the values .5, .1, .01, .001, etc. Consider, for example, a'. Then ' takes on the respective values .25, .01, .0001, etc., and

hence runs far ahead of a

INFINITESIMALS

AND DIFFERENTIALS
it is

83
for-

the conception of the order of an infinitesimal, and

mulated in a precise definition as follows


Definition.
the same order
if

Two

infinitesimals,

p and

y,

are said to be of
;

their ratio approaches a limit not

lim^=ir:^'0.
V

Thus
are of the

;8=2 + a?
same
order.

and

= 3a a'
tt

For,
'

j8^ 2ct + 3a y and hence, when approaches


,.

a?
0,

^ 2+ 3 o?
.

2-l-a limS=lim-^ = 2 -=^0.


fl
,.

Similarly, 12 o?

+ 5a^ and
/3 is

Ba^

7 a' are

infinitesimals of the

same

order.

An

infinitesimal

said to be of higher order than y if

limi2

= 0,
y

Thus
j8 is

if

j3=9a*

and
a.

= 2a + 5a*,
9a

of higher order than

For,

^^

9a'

_
2
0,

y~2a+5a*
and hence, when a approaches
,.

+ 5a"

fl

T 2

9a

y
Finally,
(5)

+ 5 '

is

said to be of lower order than y if

lim"=oD,
not ";8/y equals
to Chapter 11, 6,

(read: "j8/y becomes infinite" ;


* The student should
carefully

infinity."*).

now turn back

what is

said there ahout infinity.

and read again In particular, he should im-

84
Thus
/8 is

CALCULUS
if

/8=Va
y.

and
For

y=6a + a,
1

of lower order than

;3_
y

V^ ^
6+a'
is

Va(6 + a')
in-

When

a approaches 0, it creases without limit, or

evident that the last fraction

lim^ = oo.
y
First Order, Second Oi-der,
etc.

An

infinitesimal

/i is

said to

be of the ^rs< order infinitesimal, a i.e.


;

if it is

of the

same order as the

principal

if

a
If
/8 is

of the same order as a\

i.e. if

lim4 = ^=0,
then
is

is

said to be of the second order.

And, generally,

if ff

of the

same order

as a",

i.e. if

limJ?.= ^:^0,
a"

then

is if

said to be of the

n4h

order.

Thus

/8

= 2a

or

p=

^ a
I3

or

= a + (t\

then

j8 is

of the first order.

But

if

^ = 22+a'
then
press
fi

or

= -J^ 3 +a
is

or

B=a\

is

of the second order.

on

his

mind the

fact that infinity

notation used in (5) the


to another

sign does not


is

number. The formula

not a limit and that in the that one number is equal not an equation in the sense in which

mean

2x

=3

or o'

62 = (a 6) (a +
less

no more and no
limit.

6) is an equation. The formula means than that the variable /S/7 increases in value without

INFINITESIMALS

AND DIFFERENTIALS
/3

85

If^=Va,

then

J
and

Iim-^=lgfcO.
a*
/8 is

Hence

of the order

^.

It is easily seen that if

two

infinitesimals

/S

and y

are,

under

the present definition, each of order n, then they also satisfy the earlier definition of being of the same order. For, let

hm.-^
Then,
if

"

= K^O
e

and

lim-^
a"
/S/et"

= i:^0. K
,

we

denote the differences

and y/a" L

respectively by

and

rj,

so that

(6)

K-K=e "
e

and

a"

^-i =

these variables,

and

r),

will be infinitesimals.

For, the left-

hand

side of each of the equations (6) approaches 0.

From

equations (6)

it

follows that

"

^ = K+

and

a"

^ = Z, + .

On

dividing one of these equations

by the other we have

L+

rj

We are now ready to allow a to approach

as its limit.

Then

lim^=lim^^
y

L+

Ti

By Theorem

III of Chapter

2,

5 this last limit has the value

lini^J = i-L(:K+i) = :^.

L+

ri

lim (L

7i)

Hence, finally

j^^ ^^^:^o, L y

q. e. d.

86

CALCULUS
1.

Snow

EXERCISES
tnat

^ = 5a
Show

ll2

+ a3
same

and
order.

= 7a + a<

are infinitesimals of the


2.

that
/8

= 2-3a2

and

= 2a + a*

are infinitesimals of the same order, but that their difference,

y,
3.

is

of higher order than

yS

(or y).

Show

that B

=
a'

-2

is

an infinitesimal of the second


infinitesimal.

order, referred to
4.

as_principal
/8

Show
Show Show

that

= Va^ + 2 a* is
13a'
fi

of the first order, referred

to a.
5.
6.

that

^ = V2 +

is

of lower order than

a.

that the order of

in question 5 is

= ^.

Determine the order of each of the following


referred to a as the principal infinitesimal
7. 8.

infinitesimals,

|a

+ 18a3. -a+V2' + *.
la!'

11.
12.

-s/i^

- a.
i8.

^- aW +

*
10.

13

-a

'

^^-

v2a2-'.

\8-7
If

>/

+ 4tt* +2

15.

tively,

and y are infinitesimals of orders n and m respecshow that their product, ^y, is an infinitesimal of order

+ m.
16.

If

j8

and y are

infinitesimals of the

same

order,

that their
order.

sum

is,

in general,

an

infinitesimal of the

show same

Are there exceptions ?

Illustrate

by examples.

INFINITESIMALS
2.

AND DIFFERENTIALS

87

Continuation

an
let

Infinitesimal.

Fundamental Theorem. Principal Part of Let j8 be an iniiiiiteaimal of order n, and

a be the principal infinitesimal.

Then

lim-^
cC

= ir^O.

Moreover, as pointed out in the last paragraph,


(1)

4=^+^' a"
e is

where
(2)

infinitesimal.

Prom

(1) it follows that

p = Za"

+ ea".

ing up) of

This last equation gives a most important analysis {i.e. break/3 into two parts, each of which is simple for its
peculiar reason.
Ka'^ is the simplest infinitesimal of the nth. order imagii)

own

nable,

a monomial in the independent


y

variable, the function

= Kx\
the principal part of
is
^S.

ii)

cot''

is

an infinitesimal of higher order than the nth.


^a",
is called

The

first part,

By

far the
/8

tesimals

most important case in practice Here of the ^rst order, n = 1.

that of infini-

"

and

/8

= Ka

-\- eot.

Hence we
first order is

see that the principal part of

an

infinitesimal

of the

proportional to the principal infinitesimal.

Example

1.

Let

yS

= 2a

a^.

Then p

is

obviously of the

first order, or

= 1, and here

a"

88
Clearly, theu,

CALCULUS

K=2,
is

= a,

and the principal part of p

2a.

Example

2.

INFINITESIMALS
where q
is

AND DIFFERENTIALS
Hence

89

of higher order than Ka".

-^

Ka"
But lim rj/Ka"

1+-^,
Ka"'

lim-^=l + limJ-.
Ka"
Ka"

0,

Two

infinitesimals

and the statement is established. which have the same principal parts are
but the

equivalent, and conversely.

Equivalent infinitesimals are of the same order


converse
is

not true.

The
y,

difference
/3

namely,

y,

between two equivalent infinitesimals, is of higher order than j8 or y. For

/8

and

y
l'^

hence

lim ^-^^ = lim f *? y

-(?-) = Aim ^V 1 =
0,

q.e.d.

Conversely,
ence,
/8

if (3

and y are two

infinitesimals
/3

y,

is

of higher order than

or

y,

then

whose differ(3 and y are

equivalent.

For, since
follows that

i2^l2 = ^_l, 7 y

it

j.^/^ _ ^\

li^^sU^.
by hypothesis, and

The right-hand
the left-hand side

side of this equation


is

is

equal to

Aim^Vl.
^
yJ
q.e.d.

Hence

lim"=l,
y

We

come now

to a

theorem of prime importance in the

Infinitesimal Calculus.

90

CALCULUS
The
limit of the ratio

Fundamental Theorem.
tesimals,

of two

infini-

7
is

unchanged

if the

equivalent infinitesimal

numerator infinitesimul j8 be replaaed by any /3' and the denominator infinitesimal y be


infinitesimal
y'.

replaced by

any equivalent

In other words
lim " = lim c. y y

provided

li^l^-L
is

^
It
is

li^^^^l.
obvious that

The proof

immediate.

Hence by Theorem

II,

Chapter

II,

we have

lim-'"

But the

first

and third limits on the right-hand side are each

equal to 1 by hypothesis.

Hence

lim " y

= lim ^,
y

q. e. d.

The theorem can be


form:

stated in the following equivalent

The
limit

limit of the ratio of two infinitesimals of the ratio of their principal parts.

is the

same as

the

generalize from this theorem and an infinitesimal can always and for all purposes be replaced by an equivalent infinitesimal. Thus if
infer that

The student must not

P 2a + a^
their difference,
/8

and

y = 2a a\

y = ' + *,

INFINITESIMALS
is

AND DIFFERENTIALS
On
the other hand,

91

an

infinitesimal of the second order.

is

equivalent to
y',

y.

But

it is
,

not true that the difference of

and
is

namely,

_ /3-y' = aS
It is

an

infinitesimal of the second order.


3.

obviously of

order

Thus replacing y by an equivalent


j8

infinitesimal has

here changed the order of the difference


,v^
Differentials.
x,

y.

3.

Let

y=f(x)
Dj/ be
its

be a function of

and

let

derivative

^^t yu,c
-^^/.

lim^ = i?^.
A=a)

Ax

^^/^i ^
e.

Let the difference Ay /Ax

D^y be denoted by
Ax
Ay = DjyAx

Then

and
(1)

+ eAx.
Ax can be taken
as the
it is

Since x

is

the independent variable,

principal infinitesimal.
constant, for

D^y does not vary with Ax;


its

we Ay

are considering

a value at a fixed point

X=

Xa.

Since, moreover,
as the

D^y

is

not in general zero, equation

(1) represents

sum

of its principal part, DjjAx, and

an infinitesimal of higher order, Ax. The expression D^yAx Definition of a Differential. the differential of the function, and is denoted by dy
(2)

is

called

dy = D,yAx,
:

or

df(x)=DJ(x)Ax.
etc.).

(read

" differential y " or " differential f{x) " or " dy,"


if

Thus

y = a?,

dy

2xAx,

or

dx^

= 2xAx.

92

CALCULUS
it

Since the definition (2) holds for every function y =f(x), can be applied to the particular function

Hence
(3)

^('")=^-

dx
it is

= D^x Ak = Ak.
0.

But
is
tial

not ia general true that

in general different

from

i^y and dy are equal, since t Thus we see that the differen-

of the independent variable


;

is

equal to the increment of that


is

variable

but the differential of the dependent variable

not in

general equal to the increment of thai variable.

By means
(4)

of (3) equation (2) can

now be written

in the form

dy = D^dx.

Hence
(5)

dx
is

Geometrically, the increment A?/ of the function

repre-

sented by the line MP', Fig. 33

and the

differential, dy, is

equal to

MQ,
tan T

for

from

(5)

=
dx
t.

or

dy

= dx tan

In other words. Ay represents the distance from the


level of

to the
;

curve,

when x = x'
to the tangent.

dy, the dis-

tance from the level of

Moreover, the difference

Ay dy = eAx
shown geometrically as the line QP', and is obviously from th& figure an infinitesimal of higher order than Aa; = PM. It is also clear from the figure that Ay and dy are equal
is

when and only when

the curve y

= /(a;)

is

a straight line

i.e.

INFINITESIMALS
when f(x)
is

AND DIFFERENTIALS

93

a linear function,
f{x)

= ax +

6.

Hitherto x has been taken as the independent variable, Aa;


as the principal infinitesimal.

We come now to the theorem on which the whole value of differentials for the purpose of performing differentiation depends.
Theoeem.
The relation
dy
is

(4)

= D,ydx,
both dependent on a third vari-

true, even
t.

when x and y are

able,

Suppose, namely, that x and y come to us as functions of a


third variable,
(6)
t

a;=<^(0,
that,

2/

= ^(0,

and

when we

eliminate

between these two equations, we


,
^

obtain the function

Then dx and dy have the following


the above definition, since
t,

values, in accordance with

not

x, is

now

the independent

variable, At the principal infinitesimal

dy

= Diy At,
dy

dx

= D^x At.

We wish to

prove that

= Dj/ dx.

Now by Theorem V

of Chap. II,

D,y

= Dj/D,x.
At,

Hence, multiplying through by

we

get

D,yAt
or

= D^y- DiXAt, dy = D^dx,

q. e.d,

94 With
II,

CALCULUS
this theorem the explicit use of Theorem V in Chap. 5 disappears, Formula V of that theorem now taking on

the form of an algebraic identity

du _ du dy dx dydx

To

this fact is

due the chief advantage of

differentials in the

technique of differentiation.
Differentials of

differentials of higher order


(7)

Higher Order. It is possible to introduce by a similar definition

dH/

= DJh,Aifi,

^y = DJyA3f,

etc.,

X being the independent variable.


(8)

We should then have

by

(3)

d^=DJh/dx^

or

^,= ^-'^'

etc.

hold

Unfortunately, however, relation (8) does not contiaue to when x and y both depend on a third variable, t. For
'

example, suppose ^ ^^

= t%
^

Then

= a + t\ y = a + x.
y
,

When

is

taken as the independent variable,

we have

ac-

cording to relation (8)

d^ = nfydt^ = 2dt'^;
and since
it

dx

= 2tdt,

follows that

^ ^ ^ J^ ^ J_ dx^~4:Pdfi~2fi~2x'
2dt'

On

the other hand,

when x
dh)

is

taken as the independent vari-

able, relation (8)

becomes

= D,hfdx^=%
^2^^ d^_

and consequently

da?'

INFINITESIMALS
Thus the quotient,
values according as
f

AND DIFFERENTIALS

95

^,
or
a;

is

seen to have two entirely distinct

is

taken as the independent variable.

We will
tion

agree, therefore, to discard this definition.

The

nota-

as

meaning DJ'y

is,

however, universally used in the


definitions
etc.,

Calculus,

and so we wUl accept the

^=D^%

g=2>.V,

interpreting the left-hand sides of these equations, however,


not as ratios, but as a single, homogeneous

(and altogether

clumsy

!)

notation for that

which

is

expressed more simply by

Cauchy's D.

Remark.

The operator D^

shall

be written when desired as

dx

Thus
X)j.

appears as

dxa X

Again, the equation


appears as

d'y_ d dy
da?

dx dx
is

Finally, the following notation

sometimes used
sj,

= D^y dx
'"

dx,
'

-2 = n'^ ^
da;2

etc.
'

\^i. Technique of Differentiation. Formula II, Chapter II, 6


:

Consider,

for

example,

On

writing this formula in terms of differentials,

we have

d{u

+ v) _ du
dx

dv

dx

dx

96

CALCULUS
multiply this equation through by dx

Now

d(u
:

+ v)=du + dv.

Hence the theorem 7%e differential of the sum of two ftmctions is equal to the sum of the differentials of these functions. The others of the General Formulas, Chapter II, 6, 7, can be treated in a similar way and lead to corresponding
theorems in differentials, embodied in the following important group of formulas.

Gbnbkal Formulas of Diffkkbntiation.


I.

d(cu)=cdu.
d{u

II.

III.
-TTT

+ v)= du + dv. d{uv)= udv + vdu. ,M vdu udv


V
v'-

As already

explained,

Theorem
w
(a/Hi

reduces to an obvious

algebraic identity

w CEW

dx

/7 WV dy dx'

and so does not need to be tabulated. Of the special formulas hitherto considered, only two need be tabulated, namely
Spboial Formulas of Difpbkentiation.
1.

dc

= 0.

The

first

of these formulas says that the differential of a

constant

is zero.

The second
t.

is valid,
is

independent variable, but when x


the independent variable,
(1)

not only when x is the any function whatever of

Thus

if

M=vr^

INFINITESIMALS
and we
(2)

AND DIFFERENTIALS

97

set

= l-t,
u

equation (1) becomes


(3)

= x^.
-|

Hence
But
and thus
du
.

du
dx

x~^ dx.

= dl + d{t)=0 dt,
or

2V1 -

-dt ^^^
1

du
di

2Vl -

The student should copy


2.,

off

neatly on a card the size of a


1.,

postal the General Formulas I-IV, the Special Formulas

leaving

room

for a

few further

special formulas.

All the

difEerentiations of the elementary function of the Calculus are

based on these two groups of formulas.

To
its
is

differentiate

a function means henceforth to find either

Of course, when one of these known, the other can be found by merely multiplying or dividing by the difEerential of the independent variable. We proceed to show by a few typical examples how differentials are used in differentiation.
derivative or
its differential.

Example

1.

Let

m = 12

Sa; + la?.
and apply

To

find du. side of this equation,

at the

Take the differential of each same time Formula II


du

= c?(12)+

d{- 5x)+

d{l3?).

By Formula By Formula
Hence

1, I,

d(12)

= 0.

d{5x)=:

5dx and d(7af^=7da?. du = 5dx + 21ifidx = (-5 + 21x^)dx

and

^ = -6 + dx

21a;.

98

CALCULUS

These steps correspond precisely to the steps the student would take if he were using derivatives, only he would not have written them all out in detail. He would have written

down

at sight

f.

oh

He

can avail himself of the facility he has already acquired


follows.

and shorten the work as


du

Since

= Dj/idx,
)dx,

he can begin by writing

du=(
and then
fill

in the parenthesis with the derivative.*

Example

2.

Let

_ a^ a?
have
a;')

To find_dM.

By Formula IV we
^

^ (a= + x'^)d(a^ g') - (g" x'')dia' + (a2 + ^ (a^ + x-'-)(-2x dx) (g- - x^)(2xdx) (a2 +
a;2)2
a;2)2

ia'^xdx

du dx

4 a^x
(a2

+ xy
work
this

follows.

The student would probably Remembering that


du =

prefer to

example

as

Djjt,

dx,
'

* The student must be careful not to omit any differentials. If one term of an equation has a differential as a factor, every term must have a differential as a factor. Such an equation as

dM
is

= 5 + 21 x2

absurd, since the left-hand side

not.

Moreover, there

is

is an infinitesimal and the right-hand no such thing as djU.

INFINITESIMALS
iDegiii

AND DIFFERENTIALS

99

by writing
du

dx,

and then

fill in the fraction by the old familiar methods of Chapter II. In the two examples just considered, the processes with difEerentials correspond precisely to those with derivatives^ with which the student is already familiar. This will always be true in any differentiation in which composite functions are not involved i.e. whenever, according to our earlier methods, the vanished Theorem V of Chapter II, 8 was not used. It is in the diEEerentiation of composite functions that the method of differentials presents advantages over the earlier method. We turn in the next paragraphs to such examples.
,

EXERCISES
Differentiate each of the following functions by the

of differentials, and test the result


ter II.

method by the methods of ChapAns. du

i^"!.
2.
3.

= x^ Zx + 1.

y ^=a+bx + ex'': w = a' '.


s

4.

= 96<-162.
= vat + ^gt\
1 =- x
T Ans. du A

5.

6.

= 2dx
-

100
5.

CALCULUS
Continuation.
3.

Differentiation of Composite Fanctions.

Example

Let

= Vl + x+xK

To

find

dx
To do
this,

Here, we begin by computing du.

introduce a

new

variable, y, setting
2/

=1+ +
a;

a;'.

Then

= y^.
By-

Next, take the differential of each side of this equation. Special Formula 2 above,

du
Moreover,

= dy^= \y~^ dy. dy = {1 + 2 x)dx.


= Jll^^
2V1
-\-

Hence

d
du dx

x+x^

and-

+ 20! 2-Vl+x + x^
1

Let the student carry through the above differentiation by


the methods of Chapter II and compare his

work

step

by

step

with the foregoing. He will find that, although the two methods are in substance the same," the method of differentials is simpler in form, since no explicit use of Theorem V here is
made.
Abbreviated Method.*
still

The

solution

by

differentials can be

further abbreviated by not introducing explicitly a

new

* The student should not hasten to take this step himself. He will do till he has worked a score or more of problems in differentiating composite functions as set forth under Example 3, introducing each time explicitly a new variable, as y, z, etc.
well to omit the text that follows
until he comes himself to he attempt to use it.

Not

feel that the abbreviation is

an

aid,

should

INFINITESIMALS AND DIFFERENTIALS


variable, y.

101

The problem

is

to find du,

when

u=(l + x + a:2)i
Now,
Special

Formula

2, as
is

has already been pointed out,

holds, not merely

when x
:

the independent variable, but for

any function whatsoever. be written in the form


d
[<^(a;)]"

It might, for example, equally well

= ra[^(a;)]-id<^(a!).

essential

In the present ease, then, the content of that theorem, the enables us to write and complete truth it contains,

down

at once the equation


-

d{l

+ x + x^)^= 1(1 +

a; -I-

a;^)"*

d(l

+ x + x^).

answer

This last differential is computed at sight, and thus the is obtained in two steps.

Even
process,

these two steps are carried out mentally as a single

when the student has reached

the highest point in the

technique of differentiation.
d-\/x

He
-

then thinks of the formula

r=

dx

2y/x
realizes that it holds, not

variable, but for

any function

merely when x is the independent of x, and so writes down first

the easy part of the right-hand side of the equation, thus


d-\/l

+ +
a;

a;2

=
2V1 +
a;

a;2'

carrying in his head the fact that the numerator


tial of

the radicand,

i.e.

d(l

+ x + x^).
:

is the differenThis differentiation he

performs mentally, and thus has the intermediate work on paper

final

answer with no

102
Example
tive of y
4.

CALCULUS
The method
of differentials
is

especially use-

ful in the case of implicit functions.

Thus, to find the deriva-

with respect to x when


a?

^xy-\-2y* l.

Take the

differential of each side

^x^dx

- 3xdy - 3ydx + 8fdy = 0.


;

Next, collect the terms in dx by themselves contain dy as a factor


(3x^

the others will

- 3y)dx + (8f - 3x)dy = 0.


^^3_y-3^, sy' 3a! dx
EXERCISES

Hence

Differentiate the following twelve functions

of differentials and also


variable, if one

by the method by the methods of Chapter II (in


explicitly

either order), introducing each time


is

the auxiliary

used.
a^.

1/

1.

u=Va^ + a^^ +
y

Ans. du

= J^^^^^Va* aV +
-f!B*

3-

=
=-

Ans. dy

= =
y

"
-y

3.

u M

1-x
Introduce an
auxiliary

j3.ns. Ans.

au du

dx -

(l-)'

A
/V
\

If
\

Suggestion.

variable

= l x.

Then u
4.

-,

1^ "
v^
v/

u= xy (1

Ans.
.

dx
dy dx

H-as"

- x)' 2x (l + x^y
(1

INFINITESIMALS
6.

AND DIFFERENTIALS
Ans.
y

103
-

=
(a

+ ty

=
dt

(a

ty
.

7.

2x''-xy

+ iy' = 5.

Ans.

^ = ^'"~^ dx x Sy

8.

a!2/

= al = 2ma;.

^ns.

^ = -1. dx X
^=^.
da;
2/

9.

2/2

Ans.

10.

^ + ^=L
2a!2

^ns.

^ = -^.
da;

11.

+ 3y2 = 10.
+ = 0.
!/

^ns.

^ = _?^. 3y
2a!

12.

2a!2/-a;

Ans.

^ = 3lJi1|. 4-1 da

II

The student can work the problems at the end of Chapter by the method of differentials. Tor further practice, if deexamples are appended.
Ans.

sired, the following


13.

u={x^+l)y/3?-x.

du = la^lx"- dx
2 Va;'

14.

y={x + 2V)(x-hY.
u

Ans.

^ = 3(x^-b^). dx

15.

16.

- x^ la X u = yj: <l
Va^
X

Ar,

17.

u=-

104
20.

CALCULUS
u

Va" a?.
a?

Ans.

=
cfa;

af Va'-

a?

, 21.

ja^ x-\-l w=-\/


^x''
a;

1 +x+l

Ans.

du = dx

3a*
(3-2+

X +. 1) Va^

+ x^ + l

x^

__ (x

~
23.

a^)'
x^

.
'

du _ dx *^ = dx

S-Vx

3a^

u u

= (xi-aiy. = x(x^ + 5)\ + y^ = a'.

Ans.

iMzi^.
3a,f

24.

du = 5(' Ans. dx

+ l)(a!3 + 5)i
-^ =
da;

25.

a;'

-4ns.

\;'"
*a;

CHAPTER V
TRIGONOMETRIC FUNCTIONS
1.

Radian Measure.
lies in

In Trigonometry, the radian measure

of an angle was introduced, apparently for no good purpose.

The reason
paragraph,
first recall

the importance for the Calculus of this


to differentiate the sine.

new
will

system of measurement, and will become clear in the next

when we come
the definition.

We

Let a

circle
;

be described with

its

centre at the vertex

of

the angle

let r

denote the length of the radius of the circle

and

s,

that of the intercepted arc.


6,
:

Then
is

the radian measure,

of the angle

defined as the ratio s/r


(1)

Fig. 34

For a right angle,


has the measure
61

and hence 6

straight angle

= T = 3.14159

26535 89793

..

Let
d and

<l> <j>

be the measure of the given angle in degrees.


are proportional, e

Then

c<J3,

where c is a constant. To determine c, use a convenient angle whose measure is known in both systems for example, a straight angle. For the latter,
;

and
105

<l>

= 180.

106

CALCULUS
we
find

Substitating these values in the above equation

and hence

TRIGONOMETRIC FUNCTIONS
are in the unit circle
(i.e.

107
its

the circle of unit radius with

centre at 0).

accurately

Graph of sin x. It is important for the student drawn graph of the function
y

to

make an

= siax,
Let the unit of length, as and let it be chosen as 1 cm.
is

X being taken in radian measure.


usual, be the

same on both

axes,

For

this

purpose Peirce's Table of Integrals (the table of


especially convenient,

Trigonometric Functions near the end)


since the outside

column gives the angles in radian measure, and thus as many points of the graph as are desired can be plotted directly from the tables.
j/=sin X

Fig. 35

Since

sin

(tt

a;)

= sin x
{x, y)

each determination of the coordinates


graph, for which

of a point

on the

< <^
a;

yields at once a second point,


is

namely
structed

(tt

x,

y).

Thus one arch of the curve

readily con-

from the Tables.*

lows.

From this arch a templet, or curved ruler, is made as folLay a card under the arch and with a needle prick

through enough points so that the templet can be cut accurately with the scissors.

By means
chanically,

of the templet further arches can be


is

drawn me-

and thus the curve

readily continued in both

* The graph could be


rical considerations.

made directly without tables from purely geometDraw a circle of unit radius. Construct geomet^
as those

rically

convenient

angles,

obtained from a right angle by


of these angles,

successive bisectors.

Measure any one


will

X ABP, in ra^

dians and this

number

be the abscissa of the point on the graph, the

108

CALCULUS
Put
this curve in the

directions to the edges of the paper.*

upper quarter of a sheet of centimetre paper. The graph brings out clearly the property of the function expressed by the word periodic. The function admits the
period
2ir,

since

sm (a; + 2 tt) = By means


y

sin a;

Graph of
function

cos x.

of the templet the graph of the

= cos X
This function also admits
2
^

can

now be drawn

mechanically.
cos
/

the period 27r

(a;

tt)

= cos x.

ordinate being the perpendicular dropped from P on tlie line if n = 3, the coordinates of tlie point on the graph are
:

BA.

Thus,

^=

1.18,

.92.

A second point of the


the same
ing
y, its

arch,

that corresponding to Pj, has

coordinate be-

= 7r-^=1.96,
8

y=.92.

Of course, the distance ir must be laid off on the axis of x by measui'eit cannot be constructed geometrically from the unit length. This done, the further abscissae are found by successive bisectors. * In order to obtain the most satisfactory figure, observe that the curve has a point of inflection at each of its intersections with the axis of Since a X, the tangent there making an angle of 45 with that axis. curve separates very slowly from an inflectional tangent, it will be well to draw these tangents vrith a ruler. On laying dovro the templet, the curve can then be ruled in from the latter vrith great accuracy. It will not separate sensibly from its tangent for a considerable distance from a

ment

point of inflection.

TRIGONOMETRIC FUNCTIONS
Put the graph
axis of

109

in the second quarter of the sheet, choosing the

y for this curve in the

same

vertical line as the axis of

y for the sine curve above. There remains the lower half of the sheet for the next graph.

Graph of tan x. The same tables make profusely on the graph of the function
y
in the interval

it

easy to plot points

= tan x
axis of y in the

0^ < -.
a:
i I

Take the
I

same

ver-

2/=tan

Fig. 38

tical line as in

the case of the preceding graphs.


is

This done, a
is

second templet

made and by means

of

it

the graph
a;

drawn

mechanically for values of x such that


It
is

^ < < 0.

desirable furthermore to plot the function in the two

adjacent intervals

no
" ^^
Stt

CALCULUS

-T<^<-2'
tan {x
,

in order to suggest the fact that this function admits the

period

it:

ir)

tan x.
.

2.

Differentiation of sin x.

To

differentiate the function

(1)

= smx,

apply the definition of a derivative given iu Chap. II, 1. Give to X an arbitrary value and compute the correcBb sponding value y^oiy;
yo

= sin Xocorre-

Then give x an increment Ax,


and compute again the
sponding value of y
yo
:

+ Ay = sia (xa + Ax). = sin


a;,.

M'
Fig. 39

Hence

Ay

(ajo

+ Ax) sin Xq,

(2)

Ay _ sin (xp Ax
is

+ Ax) sin
Ax

It

at this poiat in the process that the specific properties


a; come into play. Here, the representar by means of the unit circle, familiar from the Trigonometry, is the key to the solution. From

of the function sin tion of sin a;

beginning of
the figure

it is

clear that

= MP, Ay = sin (xq + Ax)


siu Xo

sia {x^

sin

aio

+ Ax) = M'P', = QP', Ax = PP'.

Hence
(3)

Ax

ppi'

TRIGONOMETRIC FUNCTIONS
and
so

111

we want

to

know

the limit approached by the latter

limS^.

By virtue

of the

Fundamental Theorem of Chap. IV,

2,

we

can replace this ratio by a simpler one, since the arc the chord PP' are equivalent infinitesimals *
:

PP and

lim5 = l. PP
Hence
lim
p'=p jypf

^' = lim ^.
F'=p ppt

On

the other hand, the triangle


2^

QPP'

is

a triangle of refer-

ence for the

QPP =

<f>,

and

so
sin

PP
When

^ =

<A.

approaches P, the secant PP' (i.e. the indefinite line determined by the two points P and P) approaches the tangent PT at P, and thus
lim
p'=p
<i>

-4.

qPT =1-01^
J

Finally, then,

lim
P'=p

^ = lim sin <^ =

PP

sinf

p'=p

\2
^^^

Xq

)=
J

cos

Xq,

and consequently

j- ^^

A^ _

^^^

Aa) Aa;

The student should

assure himself of the truth of this statement

by

visualizing the figure

pass for angles of

(making an accurate drawing with ruler and com30, 15, and 7J, the circle used being 10 in. in

diameter) and realizing that, when P' is near P, the difference in length between the arc and the chord is but a minute per cent of the length of
either one.

A formal proof will be found below.

112
or,

CALCULUS
subscript,

on dropping the

(4)

DjSin X

= cos x.
dif-

This theorem gives rise to the following theorem in


ferentials
(5)
:

d sin X

= cos x dx.

Reason for the Radian. The reason for measuring angles in terms of the radian as the unit now becomes clear. Had we used the degree, the increment \x would not have been equal
to

PP

we should have had


Aa;

= PP'
2 TT

or

Aa;

^^, = 180 PP\


IT

360

Hence

(3)

would have read

Ay_
Ax
and thus the formula of

TT

QP
would have become

180 ppi'

differentiation
sin

D^

= -^ cos x.
180

by this more important, however, is the elimination of a multiplier which is of the nature of an extraneous constant, whose presence would have obscured the essential simplicity of the formulas of the
of labor in not being obliged to multiply

The saving

constant each time

we

differentiate is great.

Still

Calculus.

EXERCISE
Prove in a similar manner that

D^
3.

cos x

= sin x.

Certain Limits.

In the foregoing paragraph we have made

use of the fact that the ratio of the arc to the chord approaches 1 as its limit. A. formal proof of this theorem, based on the

TRIGONOMETRIC FUNCTIONS

113

axioms of geometry, can be given as follows. Draw the tangent at P and erect a perpendicular at P' cutting the tangent iu Q. Denote the angle 2(! P'PQ by a.

Then
for
i)

PP <PP' <Pq + FQ;


.

a straight line

is

the shortest dis;

tance between two points


extremities.

and ii) a convex curved line is less than a convex broken line which envelops it and has the same

But

PQ = EJ^,
cos a

P'Q = PPta.Tia.
1
1-

Hence
1

< J!P' <

PP
0,
;

tan

a.

cos a

When

approaches
1,

the right-hand

inequality approaches 1

member of the double hence the middle member must also


-^

approach

or

lim

PP

= 1,

q. e. d.

The foregoing proof


is

holds, not merely for a circle, but for

any curve with a convex arc PP'.


established generally.

Consequently the theorem

The Limit lim ^^^.


it is

From

Pig. 41

^"^
MA
Fig. 41

clear that

JfP=sin,
and hence

AP=a,
sin

a_

" AP

MP
it is

By
.(1)

direct inspection of the figure

seen, then, that

lim?i^=l.

114

CALCULUS
formal proof of this equation can be given as foUowa
Fig. 42

From

PP' = 2sina,
Hence
and

PP' = 2a.

sina_PP'

therefore,

by the proposition

just established,

lim?^ = lim^'=l.
0=^0

p'=pppi

is

Another Proof of (1). The area of the sector OAP, Fig. 42, \ a, and it obviously lies between the areas of the triangles

OMP and OPN.


^ sin a.
or
cos

Hence

< ^ a < ^ tan a


cos a

cos a

< -^ <
sm a

jpr

When

approaches 0, each of the extreme terms approaches 1, and so the middle term must also do so, q. e. d.

From

Peirce's Tables, p. 130,


sin 4 40'

we

see that

= .0814,
in radians, also has the value

and the same angle, measured

Thus for values of a not exceeding .0814, sin a differs from a by less than one part in 800, or one-eighth of one per cent.
.0814, to three significant figures.

The Limits lim ^


the
first

- "os
is

^^^ lim

^~'^^"
-

From Fig.
:

42,

of these limits

seen to have the value

lim-

cos

= 0.

TRIGONOMETRIC FUNCTIONS

115

A formal proof can be derived at once by the method employed in the evaluation of the next limit,
lim1
cos

Expressing 1

cos

a.

in terms of the half angle,

we have

l-cos = 2sin2^.

Hence

cos a

Ssin^?
2
1
''2

Of

and

l-cos _li,^ lim


2

2 -^

a
2
.

EXERCISES

when
1.

In the accompanying figure determine the following limits ot approaches


:

2.

3.

AR lim MP AQ lim AP BQ lim


lim

Ans.

2
1.

Ans.

FlQ. 43

MP

4.

lim

HP AP
PQ

6.

6.

7.

lim

AN

8.

PN AP BQ lim
lim

pn'
infini-

Determiae the principal part of each of the following


tesimals, referred to a as principal infinitesimal
9.

MP.
PN.
PQ.

Ans.

a.

10.

PB. Ans.^a.
AQ.

11.
14.

BQ.

12.
15.

13.
16.

MA.

Ans. I a'.

MN.

17.

AQ-MP.

116

CALCULUS

The differenti4. Critique of the Poregoiag Differentiation. ation of sin a; as given in 1 has the advantage of being direct and lucid, and thus easily remembered. Each analytic step is
mirrored in a simple geometric construction.
advantage, however, of incompleteness.
For,
It has the disfirst,

we have

allowed Ax, in approaching


values
;

0,

to pass only through positive

and secondly we have assumed Xg to lie between and Hence there are in all seven more cases to consider. IT. ^ An analytic method that is simple and at the same time general is the following. Eecall the Addition Theorem for
the sine

+b)= sin a cos b + cos a sin b, sin (a 6) = sin a cos 6 cos a sin 6,
sin (a

whence
Let

sin (a

sin (a &) = 2 cos a sin a b = XQ. a + b = XQ+ Ax,


-(-

6)

h.

Solving these last equations for a and


,

b,

we
Ax

get

Ax

and the difference-quotient becomes

-=cos(^a.o+-^j-^2

Aw

Ax\

sm

Ax

The first factor on when Aa; approaches 0.


tor

the right approaches the limit cossjo On setting ^Ax= a, the second fac-

becomes

sma
1.

Hence the

factor approaches

Thus
cos Xq,

lim ctx^Ax

TRIGONOMETRIC FUNCTIONSv
or,

117

on dropping the subscript,


Z)j.

sin X

= cos X.
To
differentiate the

5.

Differentiation of eosa;, tanw, etc.


x,

function cos

introduce a

new

variable, y,

by the equation

y=^x.
cos x

Hence

= - y,
sin y.

and

= cos

- y]=

tained,

Taking the differential of each side of the equation thus obwe have
dcosa;

= dsmy = oos ydy.

But

118

CALCULUS

to difEerentiate the function sec x, set

sec X

= (cos
cos^

a;)~*-

Then
It
is

dsecx =
X

=
cos^

not desirable to tabulate the result, since one rarely


a;

has occasion to difEerentiate either sec


directly, as above.

or csco;,

and when 'the

occasion does arise, the difEerentiation can be worked out

the four

The student should now add to his card of Special Formulas main formulas just obtained. This card will now read

as follows
1.
2. 3.
-

4. 5. 6.
.

= 0. dx" = ma;"~i dx. d sin X = cos x dx. d cos x = sin x dx. d tan x = sec" x dx. d cot x = esc" X dx.
dc

6.

Shop Work.

To

acquire facility in the use of the

new

results, the student should

work a generous number

of simple

examples, for which the following are typical.

Example

TRIGONOMETRIC FUNCTIONS
The
solution can be abbreviated as follows.

119

The equation

d sin X
is

= cos x dx
is

true, not

merely when x

the independent variable.

It

holds, for example, in the

form

dsmy = COS ydy,


where y is any function of
x.

Hence we can write immediately

dsin ax

= cos ax d{ax),
dx.

and thus obtain the

result

d sin ax^a cos ax

Example

2.

To

differentiate the function

Let

M=V1 fc^sin",^. sin" z = 1


fc2

<^.

Then

Let

Then
and
dsin'
</i

u = z^ du = dz' = ^z~^dz; dz = -K'dairs?^. y = sin dy = cos d^ = dy^ = 23/% = 2sin<^ cos <^d<^.
<j}.

tj>

Hence
or

c?m=^"^(

2fc^sin<^ cos (^d<^)


fc2

=
du
d<l>

sin

d ^
A;2

cos-! A
sin2
</.

Vl

Example
to find -^.

3.

If

siax

+ 3my = x- y,
cos

Take the
cos

dx

differential of each side of the equation:

Hence
,

= dx dy. (cos x V)dx + (cos y + l)dy = dy _ 1 cos X dx 1 + cos


xdx +
ydy
3^

120

CALCULUS
EXERCISES

Differentiate the following functions.


1.

M
y y

= cos

oa;.

2.

= cos^x. = CSC X.
X = tan
2

3.

4.

5.

M
M

= cot2a;.

6. 8.

10.
12.

= sec3a;. = sin' X. u = x + tan X. M = sec^ X.


w

14.

sin
-

a;

1
15.

cos X

u u

= Vl + cos X.
a; a;

= 1 cos 1 + cos sin 18. M = a + b cos X 1 20. u = sin + cos x 22.* M = vers X.
16.
a; a;

23.*

It

= covers x.
siTie

* The versed sine and the coversed


vers a;

are defined as follows

= 1 cos

a;

covers a;

= 1 sin a.

TRIGONOMETRIC FUNCTIONS
cos?
24.

121

=x

sin.

2x.

25.

28.

= taii-^-^
1

29.

= 5iBJ!^.
X
a;^

30.

M = sin + cos 2
a;

a;.

31.

m= M

cos

tts;.

32.

M=-

33.

Vl-A!''sin2^
34.
/ a!COS= sm(!B

cos = z=:^===Vl &2


d>

sini* <^

+ w).
.

(^V ^^ = dx

COS
-,

(a;
'-

sm (a;
36.
6.

+ w) ^ "^ + y) +
+
a;

cos
a;

11

sm y

r-^-

35.
37.

tana; cot 2/= sin a; sin 3/.


tan

sin x

sin

jr

= 1.
sin y.

+ tan = 2 tan <^ tan


<^

38.

2/

7.

Maxima and Minima.

By means

of the

studied in this chapter the range of problems in


ally enlarged.

new functions maxima and


;

minima which can be treated by the Calculus has been materi-

No new

principles are involved

the student

should go over carefully the paragraphs of Chap. Ill relating to this subject, before he proceeds farther with the present
paragraph.

Example

1.

A man in

a rowboat 1 mile

off

shore wishes to

go to a point which is 2 miles inland and 4 miles up the If he can row at the rate of 5 miles an hour, but can beach. walk only 3 miles an hour after he lands, in what direction
should he row in order to get to his destination in the shortest
possible time ?

In the

first place, it is clear

that the straight line

AEB is

he rows toward a point P slightly farther up the beach, the amount by which he lengthens the leg AP of his path is very nearly equal to the amount by which
not the best path.

For,

if

122

CALCULUS
Consequently the time
is

he shortens the leg PB.*


ened.

short-

On

the other hand,

obviously ought not to be taken so


far

up the beach as D. The minimum occurs,


therefore, for

some

in-

termediate point.

Let the angles 0, <f> be taken as indicated in the figure. Then,


since
Fig. 44
i

= -,
V

time from

A to P = ^^^ = -
5

6 cos 6

time from

P to 5=:^=

TRIGONOMETRIC FUNCTIONS
On
,.-.

123

setting du/dO
setfi

= 0,
5

we

obtain the equation


<^ sin <^

6 ajg 6

_ _ 2 sec'

d^
dd

Z
(2)

Next, differentiate

sec'=

Odd

+ 2 Bec2 ^

d</,

= 0,

(5)

sec25l

= -2sec2^^.
by equation
or
(5)
:

Now,
(o)
^
'.

divide equation (4)


sin 6

,a\

5/3
=i^
gin

<A 2:

sin 6

=5
3
:

sinc^
is

The

result, stated in

words,

as follows

sia 6

is to

Sia <^ as

the velocity in water is to the velocity

on land.
.

Let the student work the general problem, in which


just stated.

all

the

data are take^ in literal form, and verify the general result

In order actually to determine be solved as simultaneous


:

9,

equations (2) and (6) must

,^
^
'
.

tan 6

l3sine

+ 2 tan = 4, = 5sin<^.
<^

done best by the method of Trial and Error, as it is Successive Approximations being the name usually given to it in Mathematics. It is a most important method in both sciences, and the student should let no opportunity go by to use the method whenever, as here, he meets a Cf. Chap. VII, 5. case which calls for it.
This
is

called in Physics

The Corresponding Problem in


solved a problem which
is

Optics.

We

have stated and

not lacking in interest, but which

appears to have no scientiiic importance.

This very problem,


light is different id

however, occurs in Optics.


*
i.e.

The velocity of

new

left-hand side

divide the left-hand side of (4) by the left-hand side of (5) for a and do the same thing for the right-hand sides. ;

124

CALCULUS

and water. Suppose two media to be in contact with each other, the common boundary being a be a luminous point, from which rays emanate plane. Let When the rays strike the bounding surface, in all directions.
different media, such as air

they are

all

refracted and enter the second

medium

in case the

than in the first. One And of the refracted rays will pass throttgh a given point B. now the law of light is that the time required for the light to to B is less for this path than for any other pass from
velocity of light in that
is less

medium

possible path.
If, then,

the velocity of light in the

first

medium

is

m * and

in the second

medium,

v,

we have
u
V
first

sin 6

sin<^

where n

is

the iTidex of refraction for the passage from the

medium

into the second.

EXERCISES
1.

wall 27

ft.

high

is

64

ft.

from a house.
wall.

of the shortest ladder that will reach the house


rests

Find the length if one end


horizontal

on the ground outside the

Take the angle which the ladder makes with the


as the independent variable.
2.

The equal
area

sides of

an isosceles triangles are each 8

in.

long,

the base being variable.


is

Show

that the triangle of

maximum
3.

the one which has a right angle.

Take one

of the base angles as the independent variable, ^.

A gutter is to be made out of a long strip of copper wide by bending the strip along two lines parallel to the 9 in. edges and distant respectively 3 in. from an edge. Thus the cross-section will be a broken line, made up of three straight How wide should the gutter be a;t the lines, each 3 in. long.
* The letter u used here has nothing to do with the
solving the problem.
v,

used above in

TRIGONOMETRIC FUNCTIONS
top, in order that its carrying capacity

125
as great as

may be

possible ?
4.

Ans. 6
is

ia.

which is to be 12 in. If Johnny may choose the plate on which the pie is to be baked, what size plate would he naturally select ?
5. A can-buoy in the form of a double cone is to be made from two equal circular iron plates by cutting out a sector from each plate and bending up the plate. If the radius of

Johnny

to have a piece of pie, the perimeter of

each plate

is a,

find the radius of the base of the cone

when

the buoy is as large as possible.

Ans. aVf.

6. From a circular piece of filter paper a sector is to be cut and then bent into the form of a cone of revolution. Show that the largest cone will be obtained if the angle of the sector

is

.8165 of four right angles.


7.

Two solid spheres, whose diameters are 8 in. and 18 in., have their centres 35 in. apart. At what point in their line of centres and between the spheres should a light be placed in
order to illuminate the largest

amount

of spherical surface ?

Ans. 8
8. 9.

in.

from the centre of the smaller sphere.


tent.

Find the most economical proportions for a conical

drawn along the fioor by a rope. Find the angle which the rope should make with the horizontal
to be in order that the tension

A block of stone is

may be
to be

as small as possible.

Ans. The angle of


10.

friction.

A block of stone

is

drawn up an

inclined plane

by

Find the angle which the rope should make with the plane, in order that the tension in the rope be as small as
a rope.
possible.
11.

high.

A statue ten feet high stands on a pedestal that is 50 ft. How far ought a man whose eyes are 5 ft. above the
may

ground to stand from the pedestal in order that the statue subtend the greatest possible angle ?
12.

steel girder
ft.

25

ft.

long

is

moved on

rollers

along a

passageway 12.8

wide, and into a corridor at right angles

126
to the passageway.
girder, find

CALCULUS
how wide
Neglecting the horizontal width of the the corridor must be in order that the
the corner.

girder
13.

may go round

Ans. 5.4

ft.

A gutter whose cross-section is an arc of a circle is


into shape a strip of copper.
is a,

to be

made by bending
of the strip

If the width

find the radius of the cross-section


is

when
Ans.

the
a/ir.

carrying capacity of the gutter


14.

a maximum.

end.
side,

A long strip of paper 8 in. wide is cut off square at one A corner of this end is folded over on to the opposite
thus forming a triangle.

Find the area of the smallest

triangle that can thus be formed.


15.

In the preceding question, when will the length of the

crease be a

minimum ?

16. The captain of a man-of-war saw, one dark night, a privateersman crossing his path at right angles and at a distance ahead of c miles. The privateersman was making a miles an hour, while the man-of-war could make only b miles

in the

same time.

The

captain's only

hope was

to cross the

track of the privateersman at as short a distance as possible

under his stem, and to disable him by one or two well-directed so the ship's lights were put out and her course altered Show that the man-of-war in accordance with this plan.
shots
;

crossed the privateersman's track - Va!'

b^

miles astern of

the latter.
If a
17.

= 6,
The
is

this result is absurd.

Explain.

illumination of a small plane surface by a luminous

point

proportional to the cosine of the angle between the

rays of light and the normal to the surface, and inversely proportional to the square of the distance of the luminous point

from the

surface.

At what height on the wall should an

arc
of

light be placed in order to light

most brightly a portion


ft.

the floor a

ft.

distant from the wall ?

Ans. About ^^ a

above the

floor.

TRIGONOMETRIC FUNCTIONS
18.

127

A town A

situated on a straight river,

and another town

B, a miles farther
river, are to

down

the river and 6 miles back from the

be supplied with water from the river pumped The main from the waterworks to will cost $ m per mile and the main to B will cost % n per mile. Where on the river-bank ought the pumps to be placed ?

by a

single station.

pole 25 ft. high is to be braced by a stay end of the stay being fastened to the pole and the other end to a short stake driven into the ground. How far from the pole should the stake be located, ia order that the stay be most effective ?
19.

A telegraph

20

ft.

long, one

20.

Into a full conical wine-glass whose depth


is

is

a and
the

generating angle there


radius of the ball

carefully dropped a spherical ball

of such a size as to cause the greatest overflow.


is

Show that

a.sm a
sin a

+ cos 2 a

21. A foot-ball iield 2 a ft. long and 2 b ft. broad is to be surrounded by a runnhig track consisting of two straight sides (parallel to the length of the field) joined by semicircular ends.

The track

is

to be

4c

ft.

long.

Show how

it

should be made

ia order that the shortest distance


foot-ball field

between the track and the

may

be as great as possible.
of

22.*

The number

ems

(or the

number

of sq. cms. of text)

page and the breadths of the margius being given, what ought the length and breadth of the page to be that the amount of paper used may be as small as possible ? on
this
23.

Assuming that the values of diamonds are proportional,

other things being equal, to the squares of their weights, and

diamond which weighs one carat is worth $ m, show that it is safe to pay at least $ 8 m for two diamonds which together weigh 4 carats, if they are of the same quality
that a certain
as the one mentioned.
* Exs. 22-25 do not involve Trigonometry.

128
24.

CALCULUS

When

a voltaic battery of given electromotive force

(E

and given internal resistance (r ohms) is used to send a steady current through an external circuit of R ohms resistance, an amoimt of work, W, equivalent to
volts)

^^
is

10' ergs
circuit.

done each second in the outside

Show

that, if dif-

ferent values be given to M,

will

be a

maximum when

E=

r.

cream cone is to hold one-eighth of a pint. The I, and half the angle at the vertex is x. Find the value of x that will make the cost of manufacture of the cone a minimum. (Ans. x = 35.27.)
25. ice

An

slant height is

8.

Tangents in Polar Coordinates.

Let

be the equation of a curve in polar coordinates.


find the direction of its tangent.
if

We wish to
known

The

direction will be

we

can determine the angle

duced and the tangent.

between the radius vector proLet P, with the coordinates (tq, 6^, be an arbitrary point iT of the curve and
tj/

P':(ro-FAr, flo+Afl)

a neighboring point.

Draw

the

chord
denote

PP
the

and

Z OP'P by Then obviously


lim^'=
^Q.

To determine
Pjq 45

^qj

drop
ular

perpendic-

PM from P on

the radius vector OP' and draw an arc of a circle with The right triangle MP'P is a triangle of referas centre.

PN

TRIGONOMETRIC FUNCTIONS
ence for the angle
ij/'

129

and
^

MP
cot
ij/'

Hence
(1)

cot ^0

= li-

= lim P'M

convenient infinitesimals

by more In the latter ratio we can replace P'M and We observe that cf. Chap. IV, 2.
;

MP
;

MP =
i.e.

Tq

sin

A6

hence

lim

= lim

=1

MP

and r^AB are equivalent


For

infinitesimals.

Furthermore, P'Jf and


tesimals.

P'iV=Ar
,

are also equivalent infini-

,^ _,,^ ,,_ P'M=P'N+ NM


-?df =?(,

and

ro cos Afl. 1 cos Afl

Hence

NM
3,

Now, by

130

CALCULUS

of the
(3)

In terms of differentials, this result can be written in either two forms dr = rdO
:

cot^=

tani/f
'

rde'

dr

Examvple.

Consider the parabola in polar form


TO

cos

<^

To determine
*""

i/.

Here,

(1-C0S^)2"
<^ d<f>

Hence
C0tl/f

(1

m sin COS
1

cos
md<^

<^

<^)2

sin<^

COS ^

of the latus rectum,

In particular, at the extremity we have

and thus we obtain anew the result that the tangent there makes an angle of 45 with the axis of the parabola.
Again, at the vertex,

COtU==0,
and the tangent there
is verified

,/r

= |,

as perpendicular to the axis.

From

the above equation,

cot^ =
^ 1 a simple relation between ^ and

cos ^
<f>

?^5^-,
can be deduced.
Since

sin^ J ^ l-cos</,

2 sin* cos*

2
;

,.,,^ 2sin2 2

2,,^ =cot5,
2

TRIGONOMETRIC FUNCTIONS
it

131

follows that

eot^ =
,

.A cot^-

But, for any angle,

x,

cot (w

x)= cot X.
=

Setting

a;

= ^ in the above equation, we have


cot
(ir
i/f)

cot
<4

Hence *

or, fke

supplement of

is

equal

to

*.

Thus we have a new

proof of the familiar property of the parabola, that the tangent


at
radius,

any point P of the curve bisects the angle between the OP, and a parallel to the axis, drawn through P. EXERCISES

focal

1.

Plot the spiral,

r=6,
it

and show that the angle at which

crosses the prime direction

whene = 27r
2.

is

80 67'.

Plot the spiral,

_1

Show

that

it

has an asymptote parallel to the prime vector.

Suggestion.

from the prime

Consider the distance of a point of the curve direction, and find the limit of this distance

6 approaches 0. Determine the angle at which the radius vector corresponding to 6 = 17/2 meets this curve.
3.

when

Plot the cardioid,


r

= a (1 cos

</>),

* The
(^

trigonometric
<p/2.

^)

+T=
TT

^ <2

and

equation admits a second solution, namely, however, we agree to take and ^ so that tt, this second solution is ruled out. '^ ^
If,

<

132
and show that
,

CALCULUS
cot
,

d/

=
1

sin

d>

cos

<l>

At what angle

is

the curve cut by a line through the cusp

perpendicular to the axis ?


4.

Prove

that, for the cardioid,

^
5.

2
is parallel to

Show

that the tangent to the cardioid

the

axis of the curve


6.

when

<^

|^7r.

At what

points of the cardioid

is

the tangent perpen-

dicular to the axis of the curve ?


7. Determine the rectangle which circumscribes the cardioid and has two of its sides parallel to the axis of the curve. 8.

Show

that, for the lemniscate,


?-2

= a2cos2e,
:

the angle

i^

is

given by the equation


cot
i/'

= tan 26.
tt
,

Hence, show that

/i

'I'

=2
is

9.

At what

points of the lemniscate

the tangent parallel the points which

to the axis * of the curve ?

Ans. At the point for which correspond to


10.
it

= 7r/6, and

by symmetry.
of the curve

The points

at

which the tangent, is


'

parallel to the

prime vector, are

evi-

dently those for which

= r sm

</),

* The axis of any curve is a line of symmetry. The lemniscate has two such lines. The axis referred to in the text is that one of these lines which passes through the vertices of the curve.

TRIGONOMETRIC FUNCTIONS

133

considered as a function of <^ through the mediation of the equation of the curve, has a maximum, a minimum, or a certain point of inflection. For these points, then,

^=
Show

r cos

d
is

+ sin

d.

0.

that this condition

equivalent to the one used above

in the special cases considered,


}fl

namely
IT.

-\-

<^==

11.

Plot the curve,

= acos2^, = 2 tan 26.


is

taking

a=

5 cm.

Show

that for this curve

cot./'

12.

At what

points of the curve of question 11


?

the tan-

gent parallel to the axis

Ans. For one of the points, tan 6


13.

=
V5

Plot the curve,

= a cos 3fl, = 3tan3e.


is

taking o

=5

cm.

Show

that

eot.^

14. At what points of the curve of question 13 gent parallel to the axis of the lobe ?

the tan-

Ans. For one of these points, tan


15.

=-v/l

-|

The equation

m
1 4- sin
(/>

Give a represents a parabola referred to its focus as pole. direct proof that the tangent to this curve at any point bisects
the angle formed by the focal radius drawn to this point and a parallel to the axis through the point.
16.

Show

that the tangent to the hyperbola


r

=
1

m = V3 cos

<^

134

'

CALCULUS
makes an angle of
60

at the extremity of the latus rectum

with the transverse


17.

axis.

Prove that the -tangent to the

ellipse
,

r=^ V3
with the

cos

ij)

at the extremity of the latus rectum

makes an angle of 30

major

axis.

9.

Differential of Arc.

Let
y
=/{<>')

(1)

be the equation of a given curve. (x, y),he a variable point, and

Let P, with the coordinates


of the curve.
is
s.

A a fixed point Then s Denote the length of the arc AF by of X for, when x is given, we know P and thus
s.
;

a function

It is possible to determine the derivative of

s,

D^,

as fol-

lows.

By the Pythagorean Theorem we have (Chap, IV, Fig. 33),

PP^ = Aa!2 + ^y^'.

Let A* approach
lim

as its limit.

Then

f^7= 1 + 1 ()^= 1 +(Ojyy.


3 the chord
it

Since by

PP' and

the arc

PP'= As

are

equivalent infinitesimals,

follows from the

Theorem

of Chap. IV, 2 that, in the can be replaced by As. Hence

Fundamental above equation, PP'

limf^Y=limf^Y = (i).,)S
and consequently
(2)

(D^sy=i+ip^yy.

TRIGONOMETRIC FUNCTIONS
On
replacing the derivatiTes in (2)

135

by

their values in terms

of differentials,

we have
\aaij
ds^
is

or
(3)

\dxj

= dx^ + dy^.
by means
Since

This formula
of the triangle

easily interpreted geometrically

PMq,

Fig. 33.

PM= dx
it

and

MQ = dy,
Theorem that

follows from the Pythagorean

(4)

PQ = ds.
and As
;

It is obvious geometrically that ds

differ

from each

other

by an

iniiiiitesimal of

higher order

i.e.

that they are

equivalent infinitesimals.*

write
/e\ (5)

Formulas for sin t, cos t. From the triangle down two further formulas
:

PMQ

we can

dy sm T = -^

cos T

= dx
ds

ds

These formulas presuppose a suitable choice of t. As s inP describes the curve in a definite sense. Let this be chosen as the positive sense of the tangent line at P. Then t shall be the angle between the positive axis of x and this line. If t were taken as the. angle which the oppositely directed tangent makes with the positive axis of x, the sign must be written before each right-hand side in (6). The formulas (6) suggest that x and y can be taken as funccreases, the point

tions of s

= ^(s),
.

...
i/f(s).

variahle

* In case the coordinates x and y are expressed as functions of a third t, dx will not in general be equal to Aa;, but wiU differ from it by

an infinitesimal of higher order. The triangle PMQ will then be replaced by a similar triangle PMiQi, in which Mi lies on the line PM, its distance from being an infinitesimal of higher order.

136

CALCULUS
s is given, P,

This is, of course, always possible, since, when and hence also x and y, are determined.
Since
(6)

ds
(5)

= \'dx' + dy

we have from
(7)

sinT

= +--^^ -s/da? + dy^


s

cosr

^
Vdx^ + dy''
Furthermore,
^

no matter what choices of


dy
(8)

and t are made.*

3inT

=+

'^^

cosT

Which

sign

is

to be used in (8)

depends on which of the two


t.

possible determinations has been chosen for

Thus t

in a

given cas^ might be 30 or 30

+ 180 = 210.

If the first

choice were made, t = 30, then sin. t, cos t, and dy/dx = tan t would all be positive quantities, and hence the upper signs must be taken. But if the other choice, t = 210, is made, then sin T and cos t are negative, and the lower signs hold.

Example.

Consider the parabola

y
Let

= x\
lies in

P be a point of the

curve which

the

first

quadrant.

Since

tanT
is

= ^ = 2a;
dx
first

here positive, t

may
2a!

be taken as an angle of the

quad-

rant.

In that

case,

formulas (8) give


.

smT =

cost

Vl + 4a;2

VH-4a;2'
(8).

* The signs in (6) and (7) are not necessarily tlie same ; also in (7) and

TRIGONOMETRIC FUNCTIONS
If

137

P is a point of

the curve which

lies ia

the second quad-

rant, tan T is negative,

an angle of the second or fourth quadrant. If we choose to take t as an angle of the second quadrant, formulas (8) become
t is

and

sinT

2x
.

eosT

1
-

Vl + 4aj2

Vl+4a;2
take t as an angle of the fourth

We may, however, equally well


quadrant.

Then
.

sinT=
Vl+4a;'
In each
ease,

cost

Vl + 4a!2
t, is

one of the numbers, sin t and cos

positive,

the other, negative.

Polar Coordinates.
the curve

Similar considerations in the case of


..

lead to the following formulas

cf Fig.
.

46

PP'^

= P'3P + MP^.

Hence

limf^Y=limr^%limf^'
the chord

Now,

PP

infinitesimals.

Moreover,

and the arc PP' = As are equivalent and Ar are equivalent and

PM

MP and r^B are equivalent.

Hence

Dropping the subscript and writing the derivatives in terms


of differentials

we

have, then

or
(10)
ds2

= dr-2 + r^dB\

138
Furthermore,
(11)

CALCULUS

.it=f,

c<*.|,

the tangent
s,

PT being drawn in the direction of the increasiag and ^ being taken as the angle from the radius vector produced to the positive tangent.
rates

'

Rates and Velocities. The principles of velocities and were treated in Chapter III, 8. We are now in a position to deal with a wider range of problems. We note the following formulas. Let a point P describe
10.

the curve

^/v

Let

denote the length of the arc, measured from an arbitrary

point in an arbitrary sense, and let t be the angle from the


positive direction of the axis of x to the tangent at

drawn

m the direction of the


of the velocity (v
(1
^

increasing arc.

Then the components

= ds/dt) of P along the axes are, respectively:


T,
'

-H,
^

= v cos
da;

dy -s-=
dt

sm T.

dt

Let a point
(2)

P describe the curve


r

= {&).
;

s denote the length of the arc, measured from an arbitrary point ia an arbitrary sense and let \ji be the angle from the

Let

radius vector, produced beyond P, to the tangent at


iu the direction of the increasing arc.

P drawn

Then the components

of the velocity {v

= ds/dt)

of

along the radius vector pro-

duced and perpendicular to the same (the sense of the increasing 6 biing taken as positive for the latter) are respectively
^ ^

dt
1.

dt
is

Example

railroad train

running at the rate of 30

miles an hour along a curve in the form of a parabola


y2

= 1000

a;,

TRIGONOMETRIC FUNCTIONS
taken as the unit of length.

139

the axis of the parabola being east and west, and the foot being

The sun

is

just lising in the east.

Find how fast the shadow of the locomotive is moving along the wall of the station, which is north and south, when the distance of the shadow from the axis of the parabola is 300 ft. The first thing to do is to draw a suitable figure, introduce suitable variables, and set down all the data not already put
into evidence

present case

by the we have,
;'

figure.

Thus
:

in the

in addition to the ac(a) the

companying

figure, the further data

velocity of the train

this

in feet per second, since

must be expressed we wish to retain

the foot as the unit of length for the equa^


tion of the curve.
is

Now, 30 miles an hour equivalent to 44 feet a second. On the other hand, another expression for the velocity is ds/dt. Hence we have, on

equating these two values,

^=44
dt'
(b)

We

must

set

down

explicitly at this "point the equation

of the curve,

^^^
figure

To sum

up, then,

we

first

draw the

and then write

down' the supplementary data

Given

a)

-dt

= 44,

and'

b)

y^

= 1000x.
make
clear

The second thing


is.

to

do
it

is

to

what the problem


:

In the present case Tofind

can be epitomized as follows

(^]

We

posal for solving the problem.

are noj7 ready to consider what methods are at our disobserve that ds occurs in

We

140
the data.
eral

CALCULUS

Obviously, then, we must make use of the one gentheorem we know which gives an expression for ds when the equation of the curve comes to us in Cartesian coordinates, namely, the theorem
ds'

= da? + dy\
This can be done by means of the we have

Since dx occurs neither in the data nor in th conclusion,

we wish

to eliminate

it.

equation of the path

6).

DifiEerentiating 6)

dy

Hence

TRIGONOMETRIC FUNCTIONS
Angular
angle, ^, creasing
Velocity.

141

By

the angular velocity,

line is turning in a given plane is

u>, with which a meant the rate at which the

made by the

rotating line with a fixed line,

is in-

do
dt

Example

2.

point

is

describing the eardioid

r
at the rate of c
ft.

= a (1 cos 6)
Find the rate at which the

a second.

radius vector

drawn

to the point is turning


is

when
:

= ir/2.

The formulation
Given

of this problem

as follows

a(l cos

ff).*

142

TRIGONOMETRIC FUNCTIONS
7.

143
the lamp-

How

fast is the direction of the

man from
8
?

post changing in Ex. 12 of Chap. Ill,

8. The sun is just setting aa a baseball is thrown vertically upward so that its shadow mounts to the highest point of the dome of an observatory. The dome is 50 ft. in diameter. Find how fast the shadow of the ball is moving along the dome one second after it begins to fall,' and also how fast it is

moving just
9.

after

it

begins to

fall.

Let AB, Fig. 48, represent the rod that connects the piston of a stationary engine with the fly-wheel. If u denotes
the velocity of

and V that of show that

A in its rectilinear path, B in its circular path, + cos 6 tan ^)?;.

/V^~-

M =:(sin 6
10.

FiQ. 48

rind the velocity of the piston of


is

a locomotive when the speed of the axle of the drivers


11.

given.

A
is

long

drawbridge 30 ft. being slowly raised

by chains passing over a windlass and being drawn in at


the rate of 8
ft.

a minute.

distant electric light sends

out horizontal rays and the


Fig. 49

bridge thus casts a shadow

on a vertical wall, consisting of the other half of the bridge, which has been already raised. Find how fast the shadow when half the chain has been is creeping up the wall

drawn
12.

in.

A man

walks across the

floor of a semicircular
ft.

rotunda

100

ft.

in diameter, his speed being 4

a second, and his

path the radius perpendicular to the diameter joining the extremities of the semicircle. There is a light at one of the latter points. Find how fast the man's shadow is moving along the wall of the rotunda when he is halfway across.

144
13.

CALCULUS

A man in a train that is


window

running at

full

speed looks out


If

of the

in a direction perpendicular to the track.

he

fixes his attention successively for short intervals

of time

on objects at different distances from the train, show that the rate at which he has to turn his eyes to follow a given ohject is inversely proportional to its distance from him.
14.

Water

is

flowing out of a vessel of the form of an inis 30,

verted cone, whose semi-vertical angle


fast

at the rate of a

quart in 2 minutes, the opening being at the vertex.


is

How

the level of the water falling


still

when

there are 4 qt. of

water

in ?

15. Suppose that the locomotive of the first of the Examples worked in the text is approaching the station at night at the rate of 20 miles an hour, its headlight sending out a bundle of parallel rays. How fast will the spot of light be moving

along the wall of the station


light
line, is

when

the distance of the headin a straight

from the vertex A of the parabola, measured


500
ft.

?
is

Assume

that the wall

perpendicular to the axis of the

parabola and distant 75


16.

ft.

from the vertex.

In the preceding question, how fast will the bundle of

rays be rotating ?
17.

A point describes a circle with constant velocity.


its
is

Show

that the velocity with which

projection moves along a given

diameter
diameter.
18.

proportional to the distance of the point from this

A point P describes
lies in

the arc of the ellipse

which
rant.

the
at

first

quadrant, at the rate of 12

ft.

a second.
quad-

The tangent

F cuts off

a right triangle from the

first

How fast is

the area of this triangle changing

when

passes through the extremity of the latus rectum ?


increasing or decreasing ?

Is the area

TRIGONOMETRIC FUNCTIONS
19.

145

A point P describes the oardioid r = 5 (1 cos


How
fast is

ff)

at the rate of 12 cm. a second.

The tangent

at

P
is

cuts the
7r/2 ?

axis of the curve in Q.


20.

moving when 6 =

The sun

is

just setting in the west as a horse

running

around an

elliptical track at the rate of

miles an hour.

The

axis of the ellipse lies in the meridian.

Find the rate at which the horse's shadow moves on a fence beyond the track and parallel to the axis.

CHAPTER VI
LOGARITHMS AND EXPONENTIALS
1.

Logarithms.
is

The logarithms with which the student


definition of logioic is as
if

is

familiar are those which are ordinarily used for computation.

The base

10,

and the
y

foUows

= logio X

10"

= X.
caai

These are called denary, or Brigg^s, or common logarithms. More generally, any positive number, a, except unity, be taken as the base, the definition of log, x then being
(1)
y
3/

= log,

a;

if

a=x.

= log I

Fig. 50

The accompanying

figure represents in character the graph

of the function log, x for

any a

>

1.

It

is

drawn

to scale for

146

LOGARITHMS AND EXPONENTIALS


a = 2.71828.
shortly.

147

The reason
it

for this

choice of a will appear

From
(2)

the definition
log.l

follows at once that

= 0,

loga

=L

Only positive numbers have logarithms.


positive.
0),

For, a" is always Hence, if x be given a negative value (or the value the second equation under (1) above cannot be satisfied by

any value of y. The two leading properties of logarithms are expressed by


the equations *
:

(I)

logP+logQ = log(PQ)
logP" = nlogP.

(II)

P and Q are any two positive numbers whatever, and n any number, positive, negative, or zero. The base, a, is arbitrary. Thus
Here,
is

and

= log 2 + log 5 log VT = log 7^ = J log 7.


log 10

From
(3)

equation (I)

it

follows that

log^ = -logQ

and
(4)

log

1= log P- log Q.
(I),

For, if

we

set

P = 1/Q in
log 1

we have

= log - + log Q.
log 1

But, by

(2),

= 0.

* The student should recall the proofs of these theorems, which he learned in the earlier study of logarithms, and make sure that he can Proofs of the theorems are given in the author's reproduce them.

DWerential and Integral Calculus,

p. 76.

148

CALCULUS
1 log- = -logQ,
i

Hence

/-

j q.e.d.

Again, write (1) in the form

logiPQ')

= logP+logQ',

and now

set Q'

= 1/Q.

Then

P i log- = logP+log-.
But

logi = -logQ.

Hence

p log-=logP-logQ,
For example,
log (a as

q.e.d.

+ b)- log a = log fl + ^\


b,

we

see

by

setting, in equation (4),

P=a +
As a

Q = a.
we

further example of the application of equation (II)

may

cite

the following
^g

^t ft
Eor,
if

+ ^) = log Ka + }=-,
h the left-hand side of this equa^

P=a + 6

and

tion has the value

n log P.

Further Property of Logarithms.


b,

When

it is

desired to

express a logarithm given to a certain base,

a, in

terms of
is

logarithms .taken to a second base,

the following relation

needed
(III)
log.a;

= j5Si^.
Let
a'

The proof

of (III)

is

as follows.

= log X,

= X.

LOGARITHMS AND EXPONENTIALS


Take the logarithm
(5)

149

of each side of this equation to the base b


logj a

= logt X.
by
(11)^ if in (II)

But the left-hand

side can be transformed

we

take 6 as the base, thus having

logjP=nlogiP.
Here, let

P=a,
Then
and
(5)

= y.
a,

logj

a<'=y log^
a=
or

now becomes

y y=

logj

logi X.

^^, a
logs

log.,

= |2i^,
logs*

q.e.d.

Example.

Let

= 10

and

let

= 2.718.

To

iind

log 2.

From

(III),

log 2
^

= _i2gio2_ _ .3010 ^
logio 2.718

gg32

-4343

'

Two

Identities.

Just

'as,

for example,

-\/q^

=x

and

(Va:)'

= ,
state

no matter what value x


ties for
(1),

may

have, so

we can

two

identi-

logarithms and exponentials. In the second equation replace y by its value from the first equation. Thus the

equation
(6)
is
rfSo"-

a!

seen to hold for all positive values of x. Secondly, replace x in the first equation (1) by from the second equation y

its

value

= log a".

We

now by x any number

can equally well write x instead of y, understanding whatever, and we have, then, the

150
identity
(J)

CALCULUS

log. a'
all

= X.
x, positive,

This equation holds for


zero.

values of

negative, or

EXERCISES
1.

Show
rind

that

logio
2. 3.

-.. .8950

-._ = .0482.
-4ns.

logio .09420.

-1.0259.

Compute
Compute

2.718-6M2.
2.718--'^.

Ans. 1.758. Ans. 0.4186.


6.

4.
5.

Compute

w''.

Compute

V2*'^_

7.

Show

that
log tan 6

= log sin 6 log cos


fl

^,

<6

<-

8.

Show

that
log sin e

+ log cos = log 55:^,

o<e<l.
0<fl<2,r.

9.

Show

that

j^gl-|ose_21ogsin|,
10.

If

(a;,

y) are the Cartesian coordinates of a point distinct

from the origin, and (r, ff) the polar coordinates of the same point, show that , ^ ' log r = log (as' + 2^').
, , , 1^

11.

Prove that
log (a^

- 62) = log (a + 6) + log (a - 6), provided a + 6 and a b are both positive quantities.
12.

Simplify the expression


log (1

+ ;)- log (l+a!2).

13.

Show
e

that

V(e^

e-')2 + 4 = e' + e"*,

where

has the value 2.7182.

LOGARITHMS AND EXPONENTIALS


14.

151

Simplify the expression

15.

Show

that

ilog(l

= log(H-0'.
In order to differentiate

2.

Differentiation of Logarithms.
,

the function

it is

Chap. II,

necessary to go back to the definition of a derivative, 1, and carry through the process step by step.
Xq,

Give to X an arbitrary positive value,


corresponding value,
(1)
2/0,

and compute the

of the function
VO

= loga Xg.
Ax
(subject merely to the restric0)

Next, give to x an increment


tion that Xq
value,
(2)
2/0

+ Ax is

positive

and Ax #

and compute the new

+ Ay,

of the function
yo

+ Ay= log (xa + Ax). + Ax) log x


Ax

From

(1)

and

(2) it follows that

Ay ^ log, Ax

(xq

It is at this point that the specific properties of the loga-

rithmic function come into play for the purpose of transform-

ing the last expression.


log {xo

By

1, (4),

+ Ax) - log xo = log fl + ^\

and hence

We

next replace the variable Ax by a new variable

as

follows

152

CALCULUS

t=
Xf,

or

Aa;

= x^.

Thus

(3) takes

on the form

From

(II), 1, the bracket is seen to

have the value

log. (1

and hence
(4) Xg

+ OS
0'.

^ = ilog(l + Ax
Aa;

As
(6)

approaches

as its limit,

also approaches 0, and so

limf2^

= llimlog,(l + 0^i_

the variable (1 proaches 0, and this limit


in mathematics

Now,

+
is

'

approaches a limit when


the

ap-

number which
cf
.

is

represented
to five

by the
is

letter e;

3.

Its value

places of decimals

= 2.71828
is

.
;

cf

3.

Moreover, log

a;

a continuous function of

x,

as

is

shown

in a detailed study of this function.*

Hence

lim log

(1

+ 0'"= log

lim (1

+ *)4 = log
we

On
have:

substituting this value in the right-hand side of (5)

Xq

* Such a treatment
stage.

is

too advanced to be pursued with profit at this

Cf. the author's Differential

and Integral Calculus, Appendix,

p. 417.

LOGARITHMS AND EXPONENTIALS


or,

153

on dropping the subscript


D,loga;
the usual

(6)

= ?:5g^.
X

Thus
becomes
f7\ (7)

if

base,

= 10,

be taken, the formula

n 1 Z)Jogioa!=

.4343...

before, in the differentiation of the sine.

have met a similar situation There, if angles be measured in degrees, the fundamental formula reads
Discussion of the Result.
:

We

J5, sin

= -^ cos x.
180

In order to get rid of this inconvenient mxdtiplier, we changed the unit of angle from the degree to the radian, and then the formula became
:

Dj
In the present
for

sin

= cos x.
do a similar thiag.

case, it is possible to

base, a, is wholly in our control, to choose as

we

like.

The Now,

any

base, the logarithm of the base is unity, 1, (2)


log

= 1.
e

If,

then,

we

choose as our base the number

a
the multiplier becomes
(8)

= e = 2.71828 = log. e = 1.

log e

For

this reason, e is taken as the base of the logarithms

used in the Calculus.*

These are called natural logarithms.


inventor of logarithms.
is

They
latter

are also called hyperbolic, or Naperian logarithms,

the
But

name

after Napier, the


e for this

* The notation

number

due to Euler, 1728.

154

CALCULUS

Napier * was the very man who introduced denary logarithms into mathematics, and so the use of his name in connection with natural logarithms is misleading. Since natural "logarithms are always meant in the formulas
of the calculus, unless the contrary
is

explicitly stated,
log,
a;

it is

customary to drop the index


to write
(9)

from the notation

and

y
identities (6)

= log X,
and
6'^"

if

e"

= x.

The
(10)

(7) of 1

now

take on the form

(11)

= X, log e' = X.
of differentiation

The formula
(12)

becomes

LOGARITHMS AND EXPONENTIALS


Hence
or

155

d log sin x = cot x dx,

log sin x = cot


dx

x.

EXERCISES
Differentiate the following functions.
1.

M
M

= log cos X. = log tan x. = log cot . = log sec K.


= log:j
,

-r-

dx

= tan x.
cot x

2.

=
die

+ tan x.

3.

M
M

du
,

-2
.

aa;

sinzoj
CSC
a;.

r>

4.

5.

M = log
1 du =,

6.

0! a;

7.

a+ M = log a
1

+ K da!a!l
1

a;

du _
da; (Zm

2a
a^

a;

_
a;

a;2

8.

M M
u

= log Va^ + xK
/< = log (1 cos
1

dx
\

a?

+ x^
.x a;
.

9.

a;).

du dw =
da;

cot

2
,

10.

= log (1 + cos
(1

du
a;).

= ^^^ 2 T'~~ tan n


da; is

3.

The Limit lim

t)

'-

Since this limit

fundamental

in the differentiation of the logarithm, a detailed discussion

of

it is

essential to completeness.

Let us

set

(1)

s=(l + 0^
s for

and compute the value of


t

values of

near

0.

Suppose

.1.

Then

s= (1.1)10,

156
and
this

CALCULUS
number
is

found by the usual processes with

loga-

rithms to be 2.69.
similar manner.

Further pairs of corresponding values {t, s) are found in a In particular, the student can verify the
:

correctness of the following table of values *


t

LOGARITHMS AND EXPONENTIALS


It is readily seen that if the interest is

157

compounded n times

in a year, the principal and interest at the end of the year will

amount

to

1000(^1
dollars,
.

+ :^)
this expression

00.

and we wish to find the limit of To' do so, write it in the form
:

when

1000

n
and
set

t='

The bracket thus becomes

and

its

limit

is e.

Hence the desired


1000e

result

is

= 1061.84.*

EXERCISE
If $ 1000 is put at interest at 4

principal
interest

% compare the amounts of and interest at the end of 10 years, (a) when the is compounded semiannually, and (6) when it is com,

pounded continuously.
5.

Ans.

diEEerence of $ 6.88.

Differentiation of

e'.

the student

wUl turn

to

Before beginning this paragraph Chap. VIII and study carefully 1.

Since
(1)

=^

and

= log y

are equivalent equations, the former function can be differentiated by taking the differential of each side of the latter

equation:

ax
*

^ = diogy = -^y
is

The
;

actual computation here

expeditiously done

by means

of

series

see the chapter

on Taylor's Theorem.

158
Hence
or
(2)

CALCULUS

(3)

The function

LOGARITHMS AND EXPONENl

160
6.

CALCULUS
Graph
of the Function ".

Por

positive values of

n the

curves

= x'

Fis. 52

lie

as indicated in the figure.

When

= 1, we

from the origin, which axes of X and y.

bisects the angle

have the ray between the positive

LOGARITHMS AND EXPONENTIALS


When n >
w
1,

161
;

the curve

is

always concave upward

when

<

1, it is

concave downward.

All the curves start at the origia and pass through the
point
(1, 1).
lies.

Por values of a; > 1, the larger n, the higher the curve For values of a; < 1, the reverse is the ease. Let X have any fixed value greater than unity a; = a;'
:

>

1.

Consider the ordiaate


y

= x'".
,

As n

increases, x'" increases continuously.

This property

is

the basis of the property of logarithms included


continuous.

the word

Differential

For proofs of the foregoing statements of. the author's and Integral Calculus, p. 27 and Appendix, p. 417.
7.

The Fornmlas

of Differentiation

to

Date.

The student

will

now

bring his card of formulas up to date by suppleso that it will read as follows

menting

it

Genebal Formulas of Diffbrbsttiation


I.

IL
III.

TV

= c du. d(u + v) = du + dv. d (uv) =udv + v du. ,fu\_ vdu udv


dcu
\vj
ifl

Special Formulas of Differentiation


1)

2)
3)

4)

5)

= 0. da;'' = wa;"~ida;. d sin X = cos. x dx. d cos = sin x dx. dtan x = sec^ x dx.
dc
a;

162
6)

CALCULUS
dcota! dloga;

= csc^a; da;.

7)

= ^.
X

8)
9)

d^ = ^dx.
da"

= a" log a dx.


new results
it is

To

obtain facility in the use of the

desirable

that the student

work a good number

of simple exercises.

Example

1.

To

differentiate the function

u=
Let y

e"'.

Then
Hence
'

= ax. u = e", du = de" = e''dy = e"(adx).


de"
-"'

= ae'

LOGARITHMS AND EXPONENTIALS

163

= An sin
Next, compute

{nt

+ y).

Since

dH_d^du\
dP~di\dt)
/

~~dF dt
dM_
dt'

we take the

differential of each, side of the equation for


df - = An d sin (nt + y)
J

An [cos {nt + y)d{nt + y)] = An^ cos (nt+y) dt.


Hence, on dividing through by
dt,

we have

^ = -An^
If

COS (nt

+ y).

now we

multiply the given value of u by n^ and add the

product to the value just obtained for


tically 0,
i.e.

dhi
,

the result

is

iden-

for all values of

+
dt^

n^M

= 0,

'

q. e. d: ^

EXERCISES
Differentiate the following functions.

1.

e-''.

~ = -2xe-". dx

2.

w=

e='"^

=e^cosa;.
dit

da;

3.

M=

(e

+ e--)2.

^ = 2(e2'-e-2'). dx

164
4.

CALCULUS
M

= 10'.
a!i<'10'.

= (2.30259
da;

OlO*.

5.

w= u M
M

^=9 10-(10 + 2.30259a!).


dx

6.

= log (sec X + tan x).

7.

a!2

1oga;.

= dx = dx
9.

see x.

a;(l

+ 2 loga;).

8.

10.

= a' log (a u = e""' COS (nt y).


a;).

11.
,

= e"'log(2a; + 3). u = e~*(^ cos ni-f-B sin n<).


m
dw
da;

12.

13.

log i\ log(a; + l). M= +l u = log(a;+Va;2_a2).


a; a; i^:^
,

loga;
.

a;

(a;

f^, + 1)2

dw

^ = Va;"1
da;

a?

14.

= log + Va^ + a
(as

du

15.

u
M

= log (e* + e^). = log tan

17.

18.

M u

= logtan^| + j\ = cotg-|

19.

20.

21.

M = log

VI + sin 0.

23.

u = log Vl

cos X.

25. M=(10i+')=. 27.

=r^Y-

LOGARITHMS AND EXPONENTIALS


29.

165

u
M u
M

= af"". = (siiia;)'=".

^=

a!'"''-i(sina;

+ a!cosa!loga;).
sitfajlogsina;).

30.

^= (suia!)--i(cos2a!32.

31.

a;"'.

u=

(cos

a;)*'""^.

33.

u
u

(tan x)". = {x^f.

34. 37.

= (loga;2)^ 35. m = (1 + a). If M = J. cos ni + JB sin nJ, show that

36.

38.

If

= Oe-" cos (Vw^ K^f + y),


h 2K
d2

show that

w% = 0.

df

CHAPTER

VII

APPLICATIONS
It often 1. The Problem of ITninerical Compntation. happens in practice that we wish to solve a numerical equation in one unknown quantity, or a pair of simultaneous equations in two unknowns, to which the standard methods with which we are familiar do not apply for example,
;

cos x
f

2 cot

fl

2 cos

= X, + 2 = cot + cos = 2.
<^,
<f>

Such equations usually come to us from physical problems, and the solution is required only to a limited degree of
accuracy,
ures.

say, to two, three, or possibly four significant fig-

Any

method, therefore, which yields an approximate

solution correct to the prescribed degree of accuracy furnishes

a solution of the problem. In particular, the problem of the determination of the error in the result due to errors in the observations comes

under this head.


2.

Solution of Equations.
1.

Known

Graphs.

Example
1)

Let

it

be required to solve the equation


cos x

= x.

We
To
2)

can evidently replace this problem by the following:

find the abscissa of the point of intersection of the curves

= cos x,
166

= x.

APPLICATIONS

167

The first of these curves we have plotted accurately to scale. The second is the right line through the origin, which bisects
the angle between the positive coordinate axes.
fore, sufficient to lay

It

is,

there-

down a

ruler

on the graph of the former

curve, so that its edge lies along the right line in question,

and observe where between

this line cuts the curve.

The

result lies

x= .1

and

= .8,

and may fairly be taken as a; = .76. It is understood, as usual iu approximate values, that the last figure tabulated does
not claim complete accuracy
alone.
;

but

we

are entitled to a somefirst figure

what better result than would be given by the

'Example
3)

2.

To

solve the equation

a!3-f-2a;-2

= 0.

Suppose we have plotted the curve


4)

= 0?
Then
:

accurately from a table of cubes.

the problem can con-

veniently be formulated as follows

To
5)

find the abscissa of the point of intersection of the curves

s?

and

2 2x.

The

details are left to the student.

Example
6)

3.

To

find the positive root of the equation


e-i''4- 2.92 a;

= 2.14.
e*

Here, we can connect up with the graph of the function by making a simple transformation. Let
7)

x'=:

^x;
es''

= 2x'.

The equation then becomes


8)

- 5.84a;' = 2.14,

168
and we seek

CALCULUS
to determine the abscissa of that point of inter-

section of the curves (for simplicity,


9)

we drop

the accent)

= 6^

and

= 5.84a; + 2.14
The second
place of
;

which

lies to

the left of the origin.


is

decimals in the coefllcients

not to be taken too seriously

we

make

drawing as the graph and a well-sharpened pencil permit. Having thus determined the negative a' from the graphs of 9), we find the desired positive x by substituting
as accurate a
this value in equations 7).
left to the student.

The execution

of the details

is

Example

4.

Solve the equation


e*

= tan x,
or

< <~
a;

If one of the curves


2/

= e*

y=. tan x

were plotted on transparent paper, or celluloid, it could be laid down on the other with the axes coinciding and the intersection read
off.

the actual grapTis

The same result can be attained by holding up in front of a bright light.


first

In cases as simple as
often yield a good
tions can be secured

this, however, free-hand graphs will approximation, and further approxima-

by the numerical methods


EXERCISES*

of the later

paragraphs.

1.

Solve the equation


cos X

= 2x.

2.

Find the root of the equation


3 sin x

= 2x

which

lies

between

and

tt.

* In solving these exercises only so great accuracy is expected as can be attained from well-drawn graphs of the standard cviTves. It will be shown in later paragraphs how the solutions can be improved analytically

and carried

to

any desired degree

of accuracy.

170

CALCULUS
is

16. rind the radius of the circle whose center and which is tangent to the parabola

at (0, 2)

y^

= x.

3.

Interpolation.

Consider the equation

1)

f{x)=0.
some degree
of accuracy.
precisely, suppose that
/(rBi)

Suppose a root has been located with

More

and

f{x^

are of opposite signs.

If the function f(x) is continuous in

if its derivative is always posialways negative) in this interval, then the function is always increasing (or always decreasing) and so must have just one root between asj and x^. The root can be found approximately as follows. Consider the graph of the function

the interval
tive (or

Xi'^x^x^ and

2)

y=m.
2/i

Let

=/(!),

Vi

=f{x2),

and draw the chord through the points (a^, y{) and (ajj, y^. The point in which this chord cuts the axis of x will obviously
yield a further approximation to the root

sought.

Denote

this last value


is

by X.

The equation
Jg

of the chord
a^i
2/1

3)
s^
Fig. 53

On

setting

2/

- ^y ^i = and solving
2/2
2/1

for x,

we

have, as the value of X, the following


4)

X=xi-?i^^2,j,
2/2-2/1

or
5)

X=x,--^^^^^^f(x,).

APPLICATIONS

171

We have explained
equations 4) and
tion of the

the method in detail and developed, in

new

5), the analytic formula for the determinaapproximation, X. In practice, however, it is

usually simpler to

draw the

straight lines of Fig. 63 accurately


off

on a generous scale and read


Example.
6)

from the

figure the value of

X.

Consider equation 3) of
a'

2,

Ex. 2

+ 2a; - 2 = 0.
=
ay'

The curve
7)

in question is here

+ 2x 2,

and the graphical solution of


a;

2 shows that the root

is

about

= .7 or .8. = .7 Let =
a;

aji

then

j/i

is

found to have the value

2/1

= -.267.
y^

Next,

let

= X2 = .S;

then

.112.

We have,
i^i, Vi)

then, to lay a secant through the points

= (-7, - .257)
by
a;

and
:

{x y,)

= (.8,

.112).

Its equation is given

3) *

.8

-.7 ^ y + .257 -.7 .112 + .257'


and solving for
x,

On
''.'

setting y

in this equation

we

get,

X=.7+^^ = .7693. .369


this
:

In order to see about how close pute the corresponding value of y


2/|z=.7693

approximation

is,

com-

= -0063. = .77.
make
this determination should take 10 cm. to

We get,
* It
is

then, about

two places of decimals, x

desirable that the student should

graphically, as indicated above in the text.

He
.7

represent the interval of length

.1,

from

Xi

to aj

.8.

172

CALCULUS
method again, taking now

It is possible to apply the


(ao,

3/i)=(.7693,

- .0063)
graphical determination

and

(a^, ^2)

as before.

We

leave this as an exercise to the

student.

He

should

make both the

with an enlarged scale and the analytic determination of formula 4).

The Method; Not, the Formula. The student may be tempted to use the formula 4) or 5), rather than to go back This would be unto the method by which it was derived. fortunate, for the formula is not easily remembered, whereas the method, once appreciated, can never be forgotten. If the student finds himself in a lumber camp with nothing but the ordinary tables at hand, he may solve his equation if he has once laid hold of the method. It is true that the best way is for him to treat first the literal case and deduce the formula. But this he may not be able to do if he has relied on the formula in the book.

EXERCISES

Apply the method


end of
4.

to a

good number of the problems at the

2.

Newton's Method.

Suppose again that

it is

a question of

solving the equation


1)

f{x)

= Q,
fairly

and suppose we have already succeeded in finding a good approximation, x = Xi. Consider the graph of the function
2)

y =/{<>=)

Compute Hi =f(x,). tangent at the point

To improve the approximation, draw the


(xi,

y^.

Its equation is

APPLICATIONS
Evidently, this luie will cut the axis of
a;

173
at a poiat very near
If,

the point in which the curve 2) cuts this axis. in 3) and solve for x, we shall ob- y set y

then,

we

tain a second approximation to the root of


1)

which we

seek.

The value

of this root

will be 4)

X = xiExample
1.

Fig. 54

studied in
to
yi
;

Let us apply the method to the Example In order, however, to have simpler numbers work with, take Xi = .77 and compute the corresponding it is found to be .0035. j/j
3.
:

(i,2/i)

= (-77,

-.0036).

We

must next compute dy/dx from the equation


y

= a^ + 2x2;
(dy\

=3.779.

dx

On

substituting these values in 3),


2/

we have
-.77).

+ .0035 =3.779
solve.

(a;

Now

set

2/

and

The

result is that given


.7709.

by

4)

a;

.0035 = .77 + 3.779

We have tabidated four figures in the result because this is about the degree of accuracy that seems likely. To test this point, compute y for the value of x which has been found
3/U=.7709

= --000L

is greater than unity, the error than one unit in the fourth place. It is easy to verify the result by computing y for the next larger four-place

Since the slope of the graph


a;

in

is less

value of

a.:

|^..,

= + .0003.

174

CALCULUS

Thus we have a complete proof that the root lies between 7709- and .7710, and we see that it lies about one quarter of the way from the first to the second value.
.

Example 2. It is shown that the equation of the curve which a chain hangs, the Catenary, is

in

5)

2,=|^e^
is

+ e-^
of the arc, measujed from

where a

a constant.
is

The length

the vertex,
6)

Let
its

it be required to compute the dip in a chain 32 feet long, ends being supported at the same level, 30 feet apart. We can determine the dip from 5) if we know a, and we s

can get the value of a from 6) by setting


/
15 15,

= 16, x = 15:

16
Leta;

Then
f(x)

= ei'-e-'-^x = 0,
the curve

and we wish
7)

to

know where

y=f{x) = e-e-'-^x
This curve starts from the origin and, since

crosses the axis of x.

ax
is

negative for small values of x, the curve enters the fourth

quadrant.

Moreover,

^=6^-6-- >0,

x>0,

APPLICATIONS
and hence the graph
f(l)
is

175
Einally,

always concave upward.

= e-e-^-2^ = .217>0,

and

and only one positive, root, and between and 1. It will probably be better to locate the root with somewhat greater accuracy before beginning to apply the above method. Let us compute, therefore, /(^). By the aid of Peirce's Tables
so the equation has one
this root lies

we

find
/(.5)

= 1.6487 - .6066 - 1.0667 = - .0245 <


/(.5)

0.

Comparing these two values of the function

= -.02,

/(I)

= .22,

and remembering that the curve is concave upward, so that the root is somewhat larger than the value obtained by direct
interpolation (this value corresponding to the intersection of

the chord with the axis of

a;)

we

are led to choose as our first

approximation
/(.6)

Kj

= .6

= 1.8221 - .5488 - 1.2800 = -.0067, .2376. /'(.6) = 1.8221 + .5488 - 2.1333 =


Hence the value
of the next approximation
.6
is

X = - z:j99K = + .0282 = .628. .2376


.6

To

get the next approximation

we compute

/(.628)

= 1.8739 - .5337 - 1.3397 = .0005.


root to three significant figures is .628

Hence the value of the

with a possible error of a unit or two in the last place, and the value of a we set out to compute is, therefore, 15/.628 = 23.9.

Remark.

Newton's method, like the other methods of

this

chapter, has the advantage that an error in computing the

new

approximation wUl not be propagated in later computations. Such an error wUl in general hinder us, because we are not

176
likely to get so

CALCULUS
good an approximation.
is

But the one


done
right,

test for

the accuracy of the approximation


tion of the corresponding y,

the accurate computa-

and

if this is

we

see

precisely

how

close

we
is

are to the desired root.

The function /()

usually simple, and

it is

easy to see

whether the curve is concave upward or concave downward near the point where it crosses the axis. We thus have a means of improving the approximation at the same time that

we

simplify the

new

value of

x.

For,

if

the curve

lies to the

right of its chord, the approximation too small


;

by

interpolation will be

and if the curve lies to the right of its tangent between the point of tangency and the axis of x, the approximation given by Newton's method will also be too small.
Comparison of the Two Methods. When looked at from their geometric side the two methods appear much alike, the first
seeming somewhat simpler, since
of derivatives.
it

does not involve the use

Why

bother, then, with Newton's

method?

It is not a theoretical question, but purely one of convenience

in carrying out the numerical work.

It will be found that, as

rule,

the first method

is

preferable in the early stages

When, however, a fairly (usually, merely in the first stage). good approximation has been reached, the numerical work ravolved in Newton's method is generally shorter than that
required

by

interpolation.

EXERCISES

Apply the method


crude, the

to the Exercises of 2.

the approximation given

When, however, by the graphical method of 1 is


to improve
it.

method of interpolation may be used

5.

Direct Use of the Tables.


1.

Example 2:
1)

Let us recur to the

first

example studied, Ex.

1,

cos

a;

a?.

APPLICATIONS
The graphical
solution gave x

177

= .75.

Turn now
:

to a table

of natural cosines in radian measure, preferably Peiroe's Tables.

As we run down the

table,

we

find the entries

RADIANS
.7389 .7418

cos

NAT

.7392

.7373
.7418.
It is

Thus X

is

seen to

lie

between .7389 and


it

an ex-

cellent exercise for the student to

work out the

interpolation

for himself before

we

take

up

at the end of the paragraph.

The answer
Example
2)

is

a;

= .7391. = e',
and
Tr/2.

2.

Consider the equation


tan X

the desired root lying between

free-hand drawing of the graphs of the functions

= tan x,
1.5.

y=e''

shows that x

lies

between 1 and

So the next step

is

taken

conveniently by opening Peirce's Tables to the Trigonometric

Functions and Huntington's to the Exponentials, and writing down the two pairs of values of the functions which came
nearest together

tan X
1.3 1.4

3.60
5.80
lie

3.67

4.06
1.3

Thus the root

is

seen to

between

and

1.4.

The general

case

which the above examples are intended


:

illustrate is the following

To solve thf equation


= 4>{x),

to

f{x)

where f{x) and


readily computed.

<^{x)

are tabulated functions, or functions

178

CALCULUS
the solution has progressed to the point indicated

When

by the examples, the next step can be taken by interpolation, or by Newton's method, as will now be explained.
Interpolation.

When

ble near together, Xi

greater than

<^(a!)

two values of the independent variar x^, have been found such that /() is for one of them and less than ^(x) for the
and

other, the best approximation to take next is the one given by

the abscissa of the point of intersection of the chords of the

graphs of the functions.

This value, X, can be found as follows.

Suppose that
/(oh)

<

<^(a;i)

and

/(ajj)

>

(^(k^).

Introduce the following notation

X Xi^
From
and

h.

the figure, the triangles


similar,

AiCBi and A2CB2 are


AiBi

= Ai,

A^Bi

= Aj.
taken

Their altitudes, when

is

as the vertex, are respectively h

and

&

h.

Hence
h Ai

^ B-h
Aj

On
3)

solving this equation for h

we

find

A,
-

A1
If /(a!i)

+ A2

8.

>

^(xi)

and/(a^)

<

^(3:2),

the result

still

holds, for

Ai and A2

now become

negative, but their numerical values

APPLICATIONS

179

correspond to the lengths of the sides of the triangles in


question.
It
is

easy to express in words the result embodied in

3).

Etjle.

In order
order

to see
to give

what fraction of

8= x^ Xi

must be

added

to x^ in

X, form

the differences

Then
their

the fraction is the quotient

of the first of

these differences by

sum.

In practice, an accurately drawn figure on a large scale will


often afford a quicker and sufficiently accurate solution.

Example.

Eetiiming to Ex. 1 above, we have


f(x)

= cos X, = .0029, 8 = X2-Xi


4>(x{)-f{x0=-.OOO4:
;

^(^)= ^
! = .7389,

% = .7418.
<t>(,x^)=- .0045.

f{x^)49

.0004_oo29
.0049

= :0116^_0002.
is

Hence the value of the new approximation

X = .7389 + .0002 = .7391.


The student
less

will

have no

difficulty in

in a similar manner.

It turns out that the correction

completing Ex. 2 above is here

than one tenth of 8, and hence it does not influence the x' = 1.30. second place of decimals
:

it is

is desired, Newton's Method. well now to apply Newton's method to the function

If a higher degree of accuracy

F{x)=f{x)-

<l>{x).

In the case of Ex. 1 above it is pretty clear that we already have four-plaee accuracy, and the computation of F{x) for the = .7391 would only verify the result. This is as far value If we needed greater as we can go with four-place tables.

180
accuracy,
tables.

CALCULUS
we
should use Newton's method and five or six-place
carried only to two-place accuracy, or

Example 2 has been

three significant figures.

We

can obtain two further figures

with the tables at our disposal,

2/1

= F{x)= tan x = F(1.30)= 3.602 - 3.669 = - .067.


y
e'.

^ = sec^a;-e^ dx
To

^ dx
10.3

= 13.97 - 3.67 = 10.30

X=1.30-f

1.3067.

test this result, however,

would require

five-place tables.

EXERCISES
Solve the following equations
1.

cot

a;

as,

< <
a;

tt.

2.

e*

-|-

loga!=

1.

3.

The hyperbolic

sine (sh

x or sinh

a;)

and cosine (ch x or

cosh x) are defined as follows


sha5

cha!

= --L

By means

and are tabulated in Peirce's Tables, pp. 120-123.


the present paragraph.

of these, reduce the treatment of Ex. 2, 4, to the methods of

We come now to one of the 6. Successive Approximations. most important of all the methods of numerical computation. In physics it is known as the method of Trial and Error in mathematics it goes under the name of the method of Succes;

sive Approximations.

The problem
equations,
1)

is

that of solving a pair of

simultaneous

F{x,y)=0,

^(x,y)=0.

APPLICATIONS
The
by
to

181

cases

which
:

arise in practice are characterized in general

t-wo things

First, there is only

one solution of the equa-

tions -which interests us,

and the physical problem enables us

make a fairly good guess at it for the first approximation. Secondly, each of the equations 1) is simple, the curve can readily be plotted in character, and the equation can be solved
with ease numerically for the dependent variable when a numerical value has been given to the independent variable. But elimination of one of the unknowns, though sometimes possible, is not expedient, since the resulting equation is hard to solve.

The method is as follows. Plot the curves 1) in character with sufficient accuracy to determine which of them is steeper (i.e. has the numerically larger slope) at their point of intersection. Let
Ci:

F(x,y)=0
is less steep.

or

y =/()

be the one that

or
y

(l>{y),

182

CALCULUS

The successive steps of the process are shown geometrically by the broken lines of the figures. The success of the method depends on the ease with which y can be determined when x is given in the case of Ci, while for Q^ X must be easily attainable from y. If the curves hapto have slopes numerically equal but opposite in sign, the process would converge slowly or not at all. But in this case the arithmetic mean of x^ and x^ will obviously give a good approximation.

pened

The method has the advantage


independent of
its

that each computation

is

predecessor.

An

error, therefore, while it

may

delay the computation, will not vitiate the result.

Example. A beam 1 ft. thick is to be inserted in a panel 10 X 15 ft. as shown in the figure. How long must the beam be made?

We have
'

sin

15

cos

Hence
Fig. 68

+ cos = 15, ^ + sin = 10. = 10 cos 15 sin cos" ^ sin"


4>
l 1

(^

<^

<^

<^

<^.

Now an

expression of the form

a cos ^
can always be written as

6 sin
sin

<^

Va^ +
where

V^(

"

cos

<^

= Va" + 6" cos

(<t>

+ a),

cos a

In the present case, then,


cos

2<^=V325cos
10
,
.

(<^

+ a),
= 15
V325

where

cos a
is

sin

V325
Thus a
an angle of the
tan
first

quadrant and
a

= I,

= 56 16'.

APPLICATIONS
Our problem may be formulated,
then, as follows
:

183

To
:

find

the abscissa of the point of intersection of the curves

= cos 2

<^,

= V326 cos

(<^

+ a).

We
with,

know from
namely

the figure a good approximation to start

tan

^ = I,

^= 33 44'.
:

For

this value of

<^

the slopes are given by the equations *


sin 2<^

i^
IT

^=_2 d^

= - 2 sin 67 28' = - 1.8,


(<^

i^

^ = - V325
y

sin

+ ) = - V325 = - 18.

Hence we have
Ci
On-,

cos 2
or

<f>

= V326 cos (d> + a)

d)

= cos^i

^::rz

a-

V325
BegiiiniQg with the approximation

we compute
Passing

2/1

= 33 44', = cos 67 28' = .3832.


^1
Cj,

now
.3832

to the curve

we compute
<^2

its

<^

when

its

= V326 cos (<^2 + ),

= 32 31'.
<j}2

We now repeat the process, beginning with fiiid = cos 65 02' = .4221,
2/2

= 32 31'

and

.4221

= V325 cos (.^3 + a),


<^4

<^3

= 32 23'.
this
is

A further repetition gives


of the root

= 32 22',

and

the value

we

set out to determine.


is

* Since the degree

here taken as the unit of angle, the formulas of


;

differentiation involve the factor ir/lSO

cf

Chap. V,

2.

184

CALCULUS
EXERCISES

1.

Solve the same problem for a

beam 2

ft.

thick.

2.

A cord

ft.

long has one end fastened at a point

ft.

above a rough
tied to a rod 2

table,
ft.

and the other end

is

long.

How

far can the

rod be displaced from the vertical through and still remain in equilibrium when
released ?
solution depends are

The equations on which the


2 cot d
J

+ -= cot = 2. 2 cos 5 + cos = ^, find the value of ^. If the coefficient of friction 3. A heavy ring can slide on a smooth vertical rod.
<j>,
I

<^

/a

To

fastened a weightless cord of length 2 a, carrying an equal ring knotted at its middle point and having its further
the ring
is

end made fast at a distance a from the


of equilibrium of the system.
4.

rod.

Find the position

Solve Example 2,

4,

by the method of successive

ap-

proximations.
7. Arrangement of the Xumerical Work in Tabular Form. In the foregoing paragraphs we have laid the chief stress on setting forth the great ideas which underlie these powerful methods of numerical computation. There are, however, certain details of technique which are important, not only for ease in keeping in view the results obtained, but also for

accuracy, since they reduce the numerical

We will illustrate what we mean

work by an example.

to a system.

Example. Let it be required to find all the values of x between 0 and 360 which satisfy the equation
sin
a;

= logio (1 cos x).

APPLICATIONS

185

A free-hand graph of each of the functions 1) y = auix, y = logjo (1 cos x)


shows that there is one root between 0 and 180 and a second between 180 and 360. But these roots cannot be located with any great accuracy iu this manner. It is necessary to do exact table work, and to keep the successive results in such form that they are convenient
for later reference.

Fig. 60

To

this

with the

trial

end such a table as the following value x = 150.

is

useful.*

Begin

186
1

CALCULUS

cos = 2 sin^-, 2'


a;

logio (1

cos x) = login ain^ - + logio 2

= 2 log sin I +.3010.


Hence
it is

possible to get along with, only

two

entries of

tlie

tables if

we make
X

use of

tlie

following scheme.

APPLICATIONS
For
<^

187

x the values of the two functions, /(as) and by a quantity -which is comparable with the error of the tables, and the problem is solved.
this value of
(x), differ

EXERCISES
1.

Determine the other root in the above problem.


Solve the equation
cot X

3.

= login (1 + sin
e~'

a;),

< <
a;

90.

3.

Find the positive root of the equation

= 0? X.
(from a table of cubes),
a;'

Suggestion.

Tabulate

x, a?

x,

and
8.

e""'.

Algebraic Equations.

By an

algebraic equation is

meant

an equation of the form


1)
aoa;"

+ aiX^-^ + - + a = 0,

ao=

0,

where n denotes a positive integer.


If the coefBcients Oq, aj, are numerical, the roots can be approximated to by the method of interpolation or by" Newton's method. In either case it becomes necessary to compute the value of the polynomial

f(x)

= OqW + aia;-i + + a

for several values of x, the later ones of


three- or four-place numbers.

which will be at least There are labor-saving devices for performing these computations, to which we now turn.
Numerical Computation of Polynomials. nomial, for example, be given
f(x)

Let a cubic poly-

= ax^ + bx^ + cx-\-d, = m.

and let it be required to compute f{x) for the value x Write down the following scheme
_a
am,

am
am^

-\-

+ bm

am^ + bm-\-c am? -\- hm? + cm

f(m)

188
the explanation of
coeflB.cient, a,

CALCULUS
wMeh
is

as follows.

Begin with the

first

and multiply it by m which stands below the line. To


second
line,

to get the expression


this

am

expression add the

coefficient, 6, to get
b.

the second expression above the

am +

Next, multiply this expression by


it,

to

get

the expression which stands below

and continue the

process.

The

last entry

above the line will be the required value,

/(m)
Example.

= aw? + bm? + <mi + d. = 7a? -&X-- + Zx -1,


= .8.

Let
f{x)

and

let it

be required to compute the value of /() for x


is

Here, the scheme


7
5.6

as follows

-.4
-.32
/(.8)

2.68

-4.856

2.144

and hence

= - 4.866.

It will be observed that the process requires only additions

and multiplications. The former can be performed mentally. The latter are executed most simply by one of the machines now in general use with computers. These instruments, combined with the method of this paragraph, have rendered Horner's method for solving numerical
(or subtractions)

algebraic equations obsolete.

EXERCISE

Compute the value


5.1a^

of

- 3.42a;'- -H 1.432a;

-|-

.8543

for X

= .1876.

In the problems which arise in physics, however, it is not a question of computing all the roots of a numerical equation, about which nothing is known beyond the coefficients. Usually, the equation is a cubic or biquadratic, and only one root Moreover, from the nature of the problem, a close is required.

APPLICATIONS

189

guess at the value of this root can be made at the outset. Then the methods set forth in this paragraph and in 2, 3 lead quickly to the desired result.

EXERCISES
Solve the following equations, being given that there and only one, between 0 and 90
1.

is

one

root,

cos3

(9

- 3 cos ^ = .6283,

0
0

2.

sin' e

- .75 sin $ = .1278,

< < <


<
6
(9

90.

90

3.

Find the root of the equation


!B*

+ 2.6SC3 - 5.2a;2 - 10.4a; + 5.0 =


and
1.

which
4.

lies

between

Find the root of the equation


3x*

- 12a? + 12a;2 -4 =
3.

which
9.

lies

between 2 and

Cubics and Biquadratics. Aside from the problem of numerical computation, the simpler algebraic equations present an intrinsic interest which should not
Continuation.
special

be ignored.
Transformations,
1)

a)

Let the cubic equation


bx'^

/()
let

= aa? +

+ cx + d = 0,
y,

a^O,

be given, and
2)

x be replaced by
y

where

= x h,

x = y -\-h.

Then
f(x)

= a(y + hy + b(y + hf J^c{y + h) + d = ^{y) = af + (3ah + b)y''+-,


later coefficients are easily written

where the

down.

190
If y
3)

CALCULUS

then
will

= )8 is a root of the equation <^(y) = o, x=p+h


1).

he a root of equation

For,

it is

always true that

/{"')

= Hy)

when X and y are connected hy the relation 2). Here, h is any number we please. In particidar, h can always he so chosen that the coefficient of the second term of
3) will

drop

out.

It

is sufficient

to set

4)

3ah +

= 0,

or

= -A.

algebraic equation of any degree into a

Obviously, the same method can be used to transform an new equation whose
is

second term

lacking.

EXERCISES
Transform the following equations into equations in which
the second term
1.
a.-3
a;'-

is lacking.
2. 4.
6.

3. 5.

+ + 1 = 0. + a^ + l = 0. 3x^7ai' + x^ x l = 0.
a;
a;*

a;2

ix'^ +2 = 0. 5a^ ia^ + x^ +x SO=zO. + a^ + iB2^a,.^l_0.


So?
a;6

6)

Let the equation

5)

f(^x)=x>+pa? + qx +
let

r=0

be given, and
6)

x be replaced by
y

y,

where

= ^,

x=ky.
r.

Then
f(x)

= kY + k-^ + kqy +
<^ (y),

Denoting this

last

polynomial by
fip)

we have

= 'f>{y)
relation
6).

for all values of x

and y which are connected by the

APPLICATIONS
It
is

191

clear that, if

j8 is

a root of the equation

then x

= kp will be a root of 5).


it

The
that,

factor k is arbitrary, and we can always determine on dividing equation 7) through by fc

so

the coefficient of
that p^O):
i)

j/'

will

be numerically equal to unity (provided

^=1
=2 k^

or

= \/p, = ^/p,

if

p>0;
p <0.

ii)

= 1

or

if

In this way, equation 5) can be reduced to one of the two forms


a)
/8)

+ y^ + Ay + B = 0; y*-y^ + Ay + B = 0.
y*
If,

in particular,

and q ^0,
y'

5)

can be reduced to the

form
y)

+ y + B = 0.
to

The method can be applied


second term
is

any algebraic equation whose

lacking
x"

+ CiX"-^ + CsK"-' 4- - + c = 0.
EXERCISES

1,

Replace the equation


7a!^

- 175a;2 + 16a; + 10 =
j8),

by an equation

of the type

and

state precisely the relation


first.

of the roots of the second equation to those of the

192
2.

Calculus
Show
that, if in the equation

Ooa"

+ aiX~^ + + a = 0,
=1

where

Oo

=?^

and a

=^ 0, the transformation

2/

is

made, the roots of the new equation,

a^"

+ -i3r"' H

1-

oo

are the reciprocals of the roots of the given equation,


3. If on transforming equation 1) by 2), where h is determined by 4), the constant term in the resulting equation 3),

<^(y)

= 0,
'

does not vanish, the further transformation

8)

= -,
z

or

= - + h,
z
is lack-

will carry 1) into

an equation ia which the linear term


Az^

^SThe theorem holds


tion of
4.

+ B^ + D=:0.
State
it

A^O,

D=^Q.

in full generality for an algebraic equaaccurately.

any higher degree.

Replace the equation


a^

4:a^ 6af + 16x-4: =


Ay* + Bf+Of'

by an equation

of the type

+ D = 0.

Graphical Treatment.

We

have already seen that the cubic

a^+px + q =
can be solved graphically by cuttiag the standard graph

ai>

by the
'

straight liae,

= -px-q.
we may consider
the general problem

Since the general cubic can be reduced by the transformation


2) to

a cubic of

this type,

of the graphical solution of a cubic as solved.

APPLICATIONS
To
9)

193

obtain a similar solution for


aoil^

tlie

general biquadratic,

+ ba^ +
y*

ex"-

dx

+ e = 0,

a 4=0,

begin by reducing
i)
ii)

it

to one of the three forms

iii)

+ y^ + Ay + B = 0; + Ay + B = 0; y' +Ay + B = 0.
y*-y2
equation of type
a^
i)
:

An

+ x'-+ Ax + B = 0,
y

can be solved graphically by cutting the standard curve

!e^

by the parabola
y

= x' Ax B.
ii)

similar procedure leads to a solution in the case of each

of the other

two types,

and

iii).

The Method of Curve Plotting. Let the coefficients a, e in equation 9) be different from 0. By means of Ex. 3, p. 192, the equation can be reduced to one of the following type
J[a;

+ B3? + Cx^ + E =

(i.

In order to discuss the number and location of the roots of


this equation, it
is

sufficient to plot the curve

= Aa^ + Ba?+ Cx^+E.

Since all the maxima, minima, and points of inflection of this curve can be determined by means, at most, of quadratic equations, the problem is readily solved in any given numerical case.

EXERCISES
Determine the number of real roots of each of the following and locate them approximately.
1.

equations,
3a!*

2.

+ 8a;3 -90a:2+ 100 = 0. Sa;* + 8a;' -90a;5 + 600 = 0,

194
3.

CALCULUS
3a!*

+ 8 a? -90aii2 + 1600 = 0.
that the eqiiation

4.

Show

has

rlo

real roots.

How many real


5.

roots has each of the following equations ?


6.
a;'

7.
9.

a;^ 5x 1 = 0. a? 4a; + l=0.

8.

a;'

+ 7a; 1=0. -3a; 2 = 0.

a;'-x + 3
3a;<

= 0.

10.

11.

+ 4a;' + 6a!2-l = 0.
6a;*

12.

+ 12a;+ 1 = 0. 3a;< - 4a?' + 12a;= + 7 = 0.


43;=- 15a;2
?

13.

How many positive


Has the equation

roots has the equation

+ 8a;3 - 12a;2 - 24a; - 1 =


3a;_8a;+12ar'

14.

+l=

any

real roots ?

16.

By means

of the graph of the function

= a?+px + q
+px + q =

show that the equation


a?

has
(a)

1 real root

when
when

(6) ^
^

real roots

^+ > ^7 4 ^+ 4 < o 27
2.

'

(c)

2 real roots when

^+
27

i-

= 0,
0,

{p and q not both

fd) 1 real root '


^

when

^ + 21 = 4

{p

= g = 0}

In case

(c) it is

customary to count one of the roots twice

in case (d), to count the root three times.

APPLICATIONS
16.

195

Extend the

criterion of Ex. 15 to the case of the general


aa:?

^^^

bofi

+ cx + d = 0.
close this chapter

10.

Curve Plotting.

We will

by

consider-

ing the application Of the principles set forth in the earlier

paragraph-on curve plotting (Chap. Ill, ing curves of a more complex nature.

6) to

some

interest-

Example
1)

1.

To

plot the curve

y-

1
l''"a!-|-l

The curve is obviously not symmetric in either axis but symmetry in the origin is fulfilled, since on replacing a; by K and yhjy the new equation,
;

the test for

-y =
is

X+1
1).

equivalent to the original equation,

Incidentally

we
be

observe that the curve passes through the origin.

In consequence of the symmetry just noted


suffi.cient to

it

will

plot the curve for positive values of x and then

rotate the figure about the origin through 180.

To each
there

positive

value of X but one

corresponds

one

value

of

y.

When

X approaches 1 as its limit from above (i.e. always greater remaining than 1), y becomes
positively
infinite.

Hence the line x=l


is

an asymptote for
FlQ. 61

one branch of the


curve.

196

CALCULUS

infinite,

X approaches 1 from below, y becomes negatively and hence this same line, x=l, is an asymptote for a second branch of the curve. Tor all other positive values of x, y is continuous. The slope of the curve is given by the equation

When

2) '

^ = -(.
dx
is
V'

{x-iy
all

(a;

+ 1)2/

and

seen to be negative for


or minima, or in fact

continuous.

Thus, ia particular, the curve

values of x for which y is is seen to have no


at

maxima
is

any points

which the tangent

horizontal.

The second
'

derivative

is

given by the formula

dx''

\{x

- Vf

(x

+ 1)V
is

When
positive,

a;

>

1,

the right-hand side of this equation

always

and so the curve is concave upward in this interval. Moreover, it is evident from 1) that, when a; = + oo, y apfrom above, and so the positive axis of x is also an proaches
asymptote.

In the interval < a; < 1, the second derivative is surely sometimes negative, for this is obviously the case when x is only slightly less than 1. Is dhj/dx^ always negative in If not, it must pass through the value 0; this iaterval? for a continuous function cannot change from a positive to a negative value without taking on the intermediate value 0.* Let us set, then, the right-hand side of equation 3)
equal to

and solve:

2fL_ + _^_Vo.
*

How must the graph


must it not
0.
?

of

a,

continuous function look, which


It

is'

some-

times positive and sometimes negative ?


scissas,

must cross the axis of ahAt the point or points where It crosses, the function

has the value

APPLICATIONS
This equation
is

197

equivalent to the following

^
(x

(x-iy
1

+ iy'
we have
1_

Extracting the cube root of each side of this equation,

x1
Clearing of fractions

x+1
:

we

find

x
or

+ l=-(x-l), 2a! = 0.
a!

vanish,

Hence a; = and we
a!

is

the only value of

for

which

(Py/dx'^

can

see at once that the right-hand side of 3) does

vanish for

= 0. We have thus proven that where iu the interval <


curve
is

a!

the contiauous function 3) is no<.l, and since it is negative in

part of this interval,

It is

it is negative throughout. Hence the concave downward throughout the interval. now easy to complete the graph. The curve has one

point of inflection,
is,

namely, the
:

origin,

and the slope there

by

2), equal to

2.
EXERCISES

Plot the following curves

198
11.

CALCULUS
y

= x-\
A.

12.

X
14.

=X
X

13.

= ^ + 3? 2x. 1
a;

=d

.&x a?.

15.,=^-^. a; la; +
l

16.

= 1A_. x x1

Example
4)

2.

To

plot the curve


y^

= x^ + a?.
is

We observe first
axis of
a;.

of all that the curve

symmetric in the

It

is sufficient,

therefore, to plot the e^rve for posithis part of the curve over on
origin.

tive values of y,

and then fold

the axis of

x.

The curve goes through the

Unlike the examples hitherto considered, this curve does not permit ah arbitrary choice of x. It is only when the right-

hand

side is positive or zero,


a!2

i.e.

when
0,

+ a? ^
or

or
a?(l

+ x)>0

x>l,

that there will be a corresponding value of y and thus a point

with the given abscissa. Obviously, the curve cuts the axis of x at the origin and at the point x = l. We have, then, essentially two problems
i)

to plot the curve for


to plot the curve for

a;

ii)

> 1<

a;

<

0.
Is

i)

When

a;

>

0,

the positive value of y

given by the

equation
6)

= xVT+x.
2

Hence

dy^
dx

+ 3a;

2Vl-|-a;

APPLICATIONS

199

For positive values of x the right-hand side of this equation is always positive, and hence there are no horizontal tangents in the interval under consideration the slope of this part of the curve is always positive. In particular, the slope at the
;

origin

is

unity

= 1.
dx

The second
7)

derivative has the value


d^y
<*''

+ 3a; 4(1 + x)^


4
is

The right-hand
that the curve

side of this equation

always

Fig. 62

positive in this interval,


is

and thus it appears concave upward for all posi-

tive values of x.
ii)

When

< <
a;

0,

given by the formula


present case,
8)

6), since

the positive value of y is no longer x is now negative.* In the

200
It
is,

CALCULUS
therefore, important to determine the

corresponding

point on the curve and draw the tangent there

2/|x=-!=-(-|)V5rri=^=.38.
Two
other important poiots for the present curve are the
origin

and the point

a;

= 1, y = 0.
dy dx

At

these

points the slope has the following values

-1

.1.

dx

1;

Fto. 63

Draw

the corresponding tangents.

the expression 10) for the second derivative it is clear that, when 1 a; 0, the right-hand side of this

From

< <

always negative, and hence the curve is concave downward throughout the whole interval in question. We can now draw in the curve in this interval,
equation
is

Kg. 63. The curve


the axis of
x.

is

now complete above

It remains, therefore,

axis.

merely to fold this part over on that The entire curve is shown in
Fig. 64.
Fio. 64

EXERCISES
Plot the following curves
1.
:

y^ = x^-a?.

2. y^

= x- Ix"-

-|- 7?.

3.

y'i={x
:

a)\Ax-\-B)

Suggestion

Write the second factor in the form

Jx + B = A(x b), where b = ,


and make two cases
case,
:

i)

A>0;

ii)

A<0.

Discuss the omitted

^ = 0.

APPLICATIONS
4.
2/2

201

a;2

*.

5.

yi

y^^

jgi.

Example
11)

3.

To

plot the curve


y''

= x{x-l){x-2).
and
x,

The curve

lies

wholly in the regions

^ ^1
a)

sc.

It is

symmetric in the axis of


it

and hence

it is sufficient

to

plot

for positive values of

y.

The function
y
is

=:-yJx(x

l)(x 2) ^ ^ 1.
a;

continuous in the interval

It starts with

the

value

when x =
X,

0, increases,

and
"

finally decreases to

when

a;=l.

When

starting
y,

with the
starting

value 2, increases,

with the value 0, increases, always remaining positive, and increasing without limit as x be-

Fia. 65

comes infinite. So much from considerations of continuity. A piore specific discussion of the character of the curve can be given by means
of the derivatives of the function.

The
12) or 13)

slope

is

given by the formula

2y^=Zx^-&x + 2
dx dy ^ dx ~
slope
is infinite

Zx^Qx + 2 2^x{x-l){x-2)
a;

The

when

or 1

dy dx

= co.

dy dx

At these

points, the tangent is vertical.

202 The
slope
is

CALCULUS
when

3a;2-6a!+2
The
roots of this equation are

= 0.
= l-V3

x=l+,
V3
The
first

of these values does not correspond to any point on

the curve.

The

second, x

= .42,
\3V3 3V3

yields a horizontal tangent,

the ordinate being

"

Plot this point and draw the tangent.


cussion on the basis of continuity
must'lbe a
it is

From the above disobvious that this point

are no other
is

maximum, and we see that there maxima or minima. But it

not clear that the curve has no points of


rpg

inflection in this interval.


iG. bb
.

treat this question,

compute the

sec;

ond derivative.
but
it is

This might be done by means of formula 13) simpler to use 12)


:

2y^ + 2^=6x-6,

y^ = 3x-3-^. "dx^ dx^


Substitute here the value of dy/dx from 13) and reduce
'

'^dx"'

4 a; (a; -1) (a;


to be in difficulty.

-2)

And now we seem


tell

How are we

going to

when

dh//dx^

is positive,

when

negative ?

First of all, y is positive, and so the sign of dhjjd^ will be the same as that of the right-hand side of the equation.

Secondly, in the interval in question,

< < 1, the denomia;

nator

is

positive.

APPLICATIONS
All turns, then, on whether the numerator,
15)
is
i.e.

203
the funotion

= 3!B*-12a!8 + 12a;2-4,
15).

positive or negative.

graph of the function


the equation

To answer The slope

this

question, plot the


is

of the graph

given by

16)

~ = 12a^ dx

36a;!

+ 24a! =

12

a; (a;

- 1)

(a;

- 2).

In the interval in question, the right-hand side of this last equation is always positive. Hence u increases with x throughout the interval ^ a; 1, and consequently attains its greatHere, est value at the end-point, x = 1.

Mi=l

= -1is

We

see,

therefore,

that

negative

throughout the whole interval in question, and consequently the graph of 1) is Concave

downward in this interval. The reasoning by which we determined whether u


itive or negative is

FlQ. 67
is

pos-

an excellent illustration of the practical application of the methods of curve plotting which we have It is in no wise a question of the precise values of learned. u which correspond to x. The question is merely Is u posiWithout the labor of a single comtive, or is it negative? putation involving table work we have answered this quesSuch questions as these arise tion with the greatest ease. again and again in physics, and the aid which the calculus is
:

most important. It may seem to have been a fluke that we were able to factor the polynomial in 16) and thus simplify
able to render here
is

One further

point.

so materially the further discussion.

which

arise in practice,'

the problems with a pedigree, just

And

yet, in the

problems

such simplifications as this present themselves with great


frequency.

204

CALCULUS
the graph,
.

To complete
2

it

remains to consider the interval

a;

<

oo.

Since

the tangent to the curve

is

vertical at the point

where the curve

meets the axis of x. It is clear, then, that the curve must be concave downward for a while, and so dPy/da? < for values of X slightly greater than 2. This is verified from 14), since
17)

m|^=-4.
the other hand,

On
is

positive.

when x is large, u is positive and (Py/da^ Hence the curve is concave upward. There must
a point of inflection in the interval, and there
that the second derivative will vanish

be, therefore,

may be several. From 14) we see


and only when

when

3^ _ jgj^ + i2a,2 -4 = 0.
is,

The problem
this equation

then, to determine the


2,

number

of roots of

which are greater than

and to compute them. Again, it is a question of the graph of 15). When > 2, we see from 16)
that

du dx

> 0.

Hence u steadily increases X. Now, from 17), u starts with a negative value, and u is positive
with

and large when x

is large.

Hence u vanishes for just one value of x which is


greater
m|j_3

than

2.

Since

= 23, this root is seen


between 2 and
3.

to lie
Fig. 68

It can be determined to

any required degree of accuracy by the foregoing methods of

APPLICATIONS
which find herewith a practical application. two places of decimals it is 2.47.
this chapter,

205
To

EXERCISES
Plot the following curves
1.
;

y
2/2

3.

= a^ X. = X.
ajs Ij.

2.
4.'

y
2/"!

5.^2/2=(a!2
2

l)(a;2-4).

6.

y"^

= x a?. =1 = {1 =

ar*.

x'^){x''-

i).

CHAPTER

VIII

THE INVERSE TRIGONOMETRIC FUNCTIONS


1.

Inverse Functions.

Let

(1)

y^m
x,

be a given function of

and

let

us solve this equation for x as

a function of y
(2)
'^

't>(y)-

Then

<j}(y) is

called the inverse function, or the inverse of the

function /(k).

Thus

if /()

Hence
and
<^(2/) is

= a^, we have y= x ^ Vy,


af'.

here the function


is

-y^
tabulated, the table also serves
It is necessary merely

When

the given function

as a tabulation of the inverse function. to enter it

from the opposite

direction.

Thus,

if

we have a
re-

table of cubes,

we can

use

it

to find cube roots

by simply

versing the roles of the two columns.

In the same way, the graph of the fxmction (1) serves as the graph of the function (2), provided in the latter case we take y as the independent variable, and x as the dependent variable,
or function.

The graph of the inverse function, plotted with x as the independent variable, can be obtained from the former graph as follows. Make the transformation of the plane which is defined

by the equations
.

(3)

^ = ^'1

or
206

''=y''\

THE INVERSE TRIGONOMETRIC FUNCTIONS


It is easy to interpret this transformation.

207
whose

Any

poipt,

coordinates are

(x, y), is
:

carried over into a point


line

{x', y') situ-

ated as follows

Draw a

L tlirough the origin bisecting the

angle between the positive axes of coordinates. Drop a perpendicular from


{x,

y)

on

L and produce it to an
The poiat thus
is

equal distance on the other


side of L.

determined
(a;',

the point
this

y').

The proof of
is

statement

immediately

evident from the figure.

Thus it appears that the


transformation (3) can be generated by rotating the
plane about
180.

through
Fig.
is

The transformation
since if a plane mirror
(x, y)

also spoken of as a reflection in L,


set at right angles to the plane of

were

and so that the line L would lie iu the surface of the mirror, the image of any figure, as seen in the mirror, would
be the transformed figure.

Monotonic Functions.
increases, or else

function, f(x),

is

said to be mono-

tonic if it is single-valued

and

if,

as x increases, /(x) always

always decreases. We shall be concerned only with functions which are, in general, continuous. It is obvious that the inverse of a monotonic function is also monotonic. A given function,
2/

=/('),

can in general be considered as made up of a number of pieces, each of which is monotonic in a certain interval.* Thus the
function
(4)

sfi

* There are functions which do not have this property not play an important rdle in the elements of the Calculus.

but they do

208

CALCULUS

can be taken as made up of two pieces, corresponding respectively to those portions of the

graph which

lie

in the first and

the second quadrants, the corresponding intervals for x being


liere

-oo<a;<0,

0<a;<oo.

inverse,

Each of the pieces, of which /(cc) is made up, has a monotonic and thus the function ^(a;) inverse to f(x) is represented by a number of monotonic functions. In the example just cited, the inverse function is multiple:

valued
(5)

y=-Vx.
pieces into

But one of the two

which the

original function was

divided yields the single-valued function


(6)

y=V5,
y
(6).

the so-called principal value of the multiple-valued function


(5); the other,

= _V^,

the remainder of

The
but
it

derivative of a monotonic function cannot change sign

can vanish or become infinite at special points.

Thus

y
is

= Va^

a;2,

^ ^ a,
a;

eral,
it

a decreasing monotonic function. Its derivative is, in gennegative but when a; = 0, it vanishes, and- when x=a, becomes infinite.
;

Differentiation of

an Inverse Function.

The function

<f>{x)

inverse to a given function f(x) can be differentiated as follows. By definition, the two equations
(7)

y=^{x)
;

and

x=f(y)

they are two forms of one and the same relar tion between the variables x and y. Their graphs are identical. Take the difEerential of each side of the second equation
are equivalent

dx

= d/{y) = DJiy).dy.

THE INVERSE TRIGONOMETRIC FUNCTIONS


Hence
(8)

209

dx

DJ{y)
side of (8)

in terms of x

To complete the formula, express the right-hand by means of (7).

2. The Inverse Trigonometric Functions. The inverse trigonometric functions are chiefly important because of their application in the Integral Calculus. They are defined as

follows.
(a) (1)
is

The Function

sin~i x.

The

inverse of the function

= sin X
1

obtained as explained in

by solving

this equation for

as a function of y,
(!')

and

is

written

= sin~i y,
In order
to

read " the anti-sine of y." *

obtain the graph of the function


(2)

y-=sm~^x

we have, then, merely to reflect the graph of (1) in the bisector of the angle made by the positive coordinate axes. We
are thus led to a multiple-valued func-

the line x = x'( 1 ^ a;' ^ 1) graph in more than one point, in fact, in an infinite number of points. For most purposes of the Calculus, however, it is allowable and advisable to pick
tion, since

cuts the

* The usual notation^on the Continent for sin-i a;, tan-i x, etc., is arc sin x, It is clumsy, and Is followed for a purely academic reason namely, that sin-' x might he misunderstood as meaning the minus first power of sin x. It is seldom that one has occasion to write the reciproWhen qne wishes to do so, cal of sin X in terms of a negative exponent.
arc tan x, etc.
;

all

ambiguity can he avoided by writing (sin x)-K

210

CALCULUS
(2),

out just one value of the function

most simply the value

rr/2, and to understand t/2 and y that lies between y by sin~'a! the single-valued function thus obtained. This de-

=+

termination

is

called the 2>rincipal value of the multiple-valued


a;.

function sin"'

Its

graph

is

the portion of the curve in


line.

Fig. 70 that is

marked by a heavy

This shall be our


is explic-

convention, then, in the future unless the contrary


itly stated,
(3)
is

and thus
y

= sin"' x
:

equivalent to the relations

(30

a;=siny,

-2^2/<|sin-i(- 1) = -|.
plate,

In particular,
sin-iO

= 0,

sin-il=^,

The student should now prepare a second

showing
Place

the graphs of the three functions sin"' x, cos~' as, tan"' w. the first in the upper left-hand corner of the sheet
in the
;

the second,

upper right-hand corner


fine lead-pencil

and the third on the lower

half-sheet.

All of these curves can be ruled from the templets.


;

Use a

then mark in the principal value of

the function in a clean, firm red line.


ure, all the principal points, as is

Also mark, in each figdone in Fig. 70 of the text.

Differentiation of sin"' x.

In order to differentiate the funcy

tion

= sm_, X,
.

'

make the equivalent

equation,
a;

= sin

3/

the point of departure.

Then

dx
'

= d sin y = cos y dy.

Hence

^ dx

^.
cosy

THE INVERSE TRIGONOMETRIC FUNCTIONS


terms of x as follows.
Since
sin2

211

Tlie right-hand side of this eqiiation. cau be expressed in

+ C032 y = l
cos y

and since sin y

= x, we have cos' y = 1 x%

= Vl
is

x''.

We

have agreed, however, to understand by

sin~ia!

the

principal value of this function.


restriction
:

Hence y

subject to the

tive (or zero).

ir/2 ^y^ jr/2, and consequently cos y is posiWe must, therefore, take- the upper sign before
d dx
.

the radical,* the final result thus being


(4)

sin

_, 1

= 1 Vl=
dx

(4')

d sin~i X

Vl-a?
(b)

The Function

cos~ia;.

The treatment here


:

is

precisely

similar.
(5)

The
y

definition is as follows
if

= cos~i X
of the
71.

= cos y,
is

(read

" anti-cosine x ").

The graph shown in Fig.


is

function cos~ia;
sin^ia;, this

as

Like

function

also infinitely multiple-valued.


is

single-

valued branch

selected

by imposing the

further condition

O^y^ir.
This determination
value of cos~i x
(6)
:

is

known

as the principal

y-

O^y^TT.
is

* Geometrically the slope of the portion of the graph in question always positive, and so we must use the positive square root of 1 x''.

212

CALCULUS
is

It will be understood lieiiceforth. that the principal value

meant unless the contrary is In preparing the graph of value as a firm red line.

explicitly stated.
this function,

mark the

principal

To

differentiate the function cos~ia!, use the implicit

form

of equation (5)

= cos y.

Hence
dx
3>iid.

= d cos y = sin y dy

THE INVERSE TRIGONOMETRIC FUNCTIONS 213

214
The
cal

CALCULUS
differentiation can be
still

performed as in the case of the


identi-

function tan"ia;, but


relation
cot"ia;:

more simply by means of the

connecting the principal values of tan"'* and

(15)

tan-*

+ cot"* = f.
=
1

Hence
(16)
^ '

lco1r>a;

dx

+ 3^'

^,

or
(17)
ci!cot-ia;

^.
this function, red.
x,

It is well for the student to


also,

make a graph of drawing in the principal value, as usual, in

The following identity holds for positive values of the principal values of the functions are used
(18)

when

tan-i-

= cot-i x,
:

0<x.

For negative values of x


(180

it

reads

tan-ii

= cot-i x-tt,

X
Remarks.
cacr^x, can

x<0.
sec-'o!,

The other

inverse trigonometric functions,

be treated in a similar manner.

They

are,

how-

ever, without

importance in practice.

Their principal values

cannot be defined by means of a single continuous curve.


necessarily consists of more than one piece it is most natural to take it as consisting of two pieces. Corresponding to the Addition Theorem for each of the

The graph

trigonometric functions, there are functional relations for the


inverse trigonometric functions.
(19)
^

Thus, for tan-'a!


'"

tan-i u

'

+ + tan-i v = tan-i 1 ^
is

WW

These

relations,

however, are not always true when the


taken,

principal value of each of the functions

and

for this

THE INVERSE TRIGONOMETRIC FUNCTIONS


reason
it is

215

usually better not to employ them.

If,

however,
less

ia a particular case,

u and v are each numerically

than

unity, the principal values can be used throughout in (19).

3.

Shop Work.
of formulas of

Special Formulas the four


list

The student will now add to his list of new formulas of this chapter. The It reads dififerentiation is now complete.

as follows.

Special Formulas of Differentiation


1.

2.
3.

4. 5.
6.

= 0. d a" = na;"~' sin x = cos x dx. d d cos x = sin x dx. dtan X = sec' x dx. d cot x = esc' X dx.
dc
da;.

7.

dloga;

8. 9.

10.

11.

12.

13.

216
Exa/mple
1.

CALCULUS
Differentiate the function

u
Let

= cos~i y =s a

> 0.

Then

= cos"' y, du = d cos~i y ^y
u
dy
,

= dx
a

dx a

Hence

dy
-

dx

VI

r.

7Y
'''
-

Va2

- X'

and, finally,

dC08-^ =

In abbreviated form.

Va2

a!2

a cos""^ -

4
-

Va2
Example
Differentiate the function

2.

THE INVERSE TRIGONOMETRIC FUNCTIONS


The student should
notice that the

217

method used
is

in the text
is

for deriving the fundamental formulas of differentiation


to he repeated in the applications.

not

It

these formulas them-

selves that should be used.

Thus, to solve Ex. 1 by writiag


cos

=X

and then differentiating would be logically irreproachable, but bad technique. EXERCISES
Differentiate each of the following functions.
1.

= smi
a

dx

iH-. --^11

2.

M M M

= tan~i
a

3.

= cot~'-a

4.

= sin~i(w sin x).

5.

- = cos 1
-1
1

dx

7.
?t

V3-|-2a;

a;2

6.

,2a! = sm-i=^^^^ =
.

i.-,x+a = coti--f
*

V2
8.

M u

= cot-1 =
X

9.

= tan-'--

10.

= tan

'

X-

218
15.

CALCULUS

= C0S-1-.
2

16.

t=sin-ii.

17.

=cos-i-+Y.
n

18.

u= X sin^i
1

a;.

THE INVERSE TRIGONOMETRIC FUNCTIONS


iFirBt of all,

219

however,

it

should be pointed out that there are


is

two distinct problems.


satisfy such equations as
(a)
(6)
(c)

One
sin

to find all values of x

which

a;

cos

a;

tan
is

a;

= .2318 = - .4322 = - 1.4861.


;

The other

to find the principal value of


;

an inverse trigono-

metric function
sin-1.2318;

for example,
cos-i

(-

.4322)

tan-i(- 1.4861)

The methods
First

of treating these problems are identical-

Geometric Method.

Equations

(a),

(6),

(c)

can be

solved graphically

by the aid

of the unit circle representation

with an error corresponding to a degree or two, the results


being expressed in radians
Calculus.
if

the problem comes from the

For example, consider equation

(6).

provide himself with an accurately drawn circle of his

The student should own

construction, executed on the accurate centimeter-millimeter

paper commercially procurable the radius of the circle berug 10 cm. and its center at a principal intersection of the rulings. To solve equation (6), he will lay a straight-edge on his plate, parallel to the secondary (or y-) axis and at a distance
;

of

4 cm., 3^ mm. to the

left of that axis.

points of intersection of the straight-edge with the circle


fine pencil lines

Marking the two by

easUy erased, he now measures one of the

acute angles involved by means of his protractor and thus determines the two solutions of (6) lying between 0 and 360
correct to minutes or thereabouts.

By

aid of the Tables the

values can at once be converted into radian measure.


Arithmetic Solutions.

From

the figure before

him the

stu-

which is known. The determination of the angle he needs is merely a problem in the solution of a right triangle by the tables, and
dent
sees clearly a right triangle, one leg of

now

220

CALCULUS

he proceeds to carry this work through to the degree of accuracy which the tables permit. Equations (a) and (c) are treated in a similar manner. The
point of this method
is

that the student

is

trained to visualize

a figure, and not

to try to

remember a

table that looks like

sin^

is

and when the student


helpless.

For, such tables vanish ia a short time,

needs his trigonometry ia later work, he

In terms of the inverse functions,


for the value of the variable, x

this first

problem consists

in finding all the values of the multiple-valued function cos~i x

= .4322.

Second Oeometric Method. This method consists ia readiag from the graph the two values which lie between and 2 jr, and then adding to these arbitrary positive or negative multiples of 2 jr. The graph suggests, moreover, how to determine these values arithmetically by the aid of a table of sines or cosines of angles of the first quadrant. It also suggests a further refinement of the graphical method, of which the student will do well to namely, this. Let him make an accurate avail himself,
off

graph of the function ^

y sin X

on cm.-mm.-paper, taking 10 cm. as the unit and measuring


This half-arch to ir/2. the angle in radians, x ranging from supplements the four graphs of the functions sin x, cos x, sin~i x,
cos~ia;

and serves as a 3-place table for determining

their values

(with a possible error of two or three units in the third place).

To sum
unit-circle

up, then, there are two geometric methods

1) the

method

2) the graphs of the functions, the latter

beiag supplemented by the 10-cm. graph just described.


of the geometric

Either

methods suggests how to use the tables correctly and affords an altogether satisfactory check on the tables.

When

the accurately drawn graphs are not at hand, freeclearly

hand drawings indicate security and accuracy.

how

to use the

tables with

THE INVERSE TRIGONOMETRIC FUNCTIONS


EXERCISES

221

1. Determine both in degrees and radians all values of x which satisfy the above equations (a), (6), (c), using each time all of the geometric methods set forth, and also the tables.

2. Mnd the value of each of the following functions. It is understood that the principal value is meant. Use first the method of the graphs. Then determine from the tables.

Check by

unit-circle

and

protractor.
ii)

i)

sin-i(-.1643);
iii)

cos-i (.6417)

tan-i(- 2.8162).

3.

By means

of a free-hand

drawing of the graph estimate

the value of each of the following functions.

Remember

that

a curve recedes from


inflection.

its

tangent very slowly near a point of

a) sin-1.113;

6)
;

tan-i(- .214)
cot"!
(

c)

cos-i.l72;
;

d) tan-i (- 7.4)
g)

e)

- .162)

/) cos-i (- .998)
i)
Z)

sin-i(-.21);

h) sin-i.89;
fc)

tan-i6.2;
sin-i

j) cot-17.3;

cos-i(-.138);
;

(-

.138).

In what cases
5.

is

your error large

in what, small ?

Applications.

The

inverse trigonometric functions afford

a convenient means of solving the following problem in Optics. A ray of light is refracted in a
prism.
its original

Show that its deviation from direction is least when

the incident ray and the refracted ray make equal angles with the
faces of the prism.

The study

of this

problem has a

pj^ ^3

vivid interest for the student

who

has seen the laboratory experiment of admitting a ray of sunlight into a darkened room, allowing it to pass through

222

CALCULUS
it

a prism, thus being refracted, and throwing


persed, on a screen.

finally, dis-

Let prism
of
6'

AF
;

be the incident ray;

PQ,

its

path through the

and
is
;

the ray -which emerges.

Then the

deflection
is

PQ

obviously

6 ^

and the further


is

deflection of Q,B

<l>'

so that the total deflection, u,

(1)

= e-4, + e'-4>' = e+6'-(ct> +


sum
\

<i>').

On

the other hand, the


/

of the angles of the triangle


/

PDQ is
Hence
(2)

'=(i-*Hi-*')+^ + 0'=a.
can, therefore, write (1) in the

We
(3)

form

u
is

= e+e'-a.
desired to

This
6' are,

the quantity

it is

make a minimum.

and

however, connected by a relation which can be obtained as follows. We have by the law of refraction (cf. Chap. V, 7)
/A\ (')

sin 5

sin^-'^'

THE INVERSE TRIGONOMETRIC FUNCTIONS 223


Take 6
as the independent variable.

Then

^'
and the condition

M-^ +
=
-

W
-^ =
d0
1.

de

gives ^

Next, take the differential of each side of the second equar


tion (8)

d(vsing)

d(vsiD.6')

Vl y2 sin2 d
or
V cos

Vl
,

1/2

sia2 6'

6dd
giji2

V cos

ff

d6'

VI
Hence
(\n\

1/2

VI _ y2 sui2 01
.

cos g

cos^^

Vl - v2 sin2 6
But dO'/dO
/-.
-|

/'^^''\_o

Vl-i/2sin2 0'Uey

= 1.

Consequently
cos 5

cos

6'

VI v2 sin2 a

Vl - v^ sin2 e'

the solution deOne solution of this equation is 6 = 6', manded by the theorem. But conceivably there might be
it -would not be clear which one of them makes u a miaimum. We can readUy show, however, that equation (11) has no further solutions. Square each side

other solutions, and then

cos^g

cos^y
l-v-'Sia^e''

l-v2siii2e

Clear of fractions and express each cosine in terms of the sine


(1

- v2 sin2 0')(1 _
sin2 e

sin'

S)=(l-

v"-

sitf e)(l

- sia^ 6').
:

Multiply out and suppress equal terms on the two sides

& = - sin' ff v^ sin' B, if - 1) sin2 e = (v2 - 1) sin2 6'.


v"'

sin2

224
Hence
siii2

CALCULUS
e

siii2

fl',

sin 6

= sin 6',
first

and consequently the only angles of the


can satisfy (11) are equal angles, 6

quadrant which

=
<f>

6'.

From

(5)

and

(6) it follows that

<^'.

Hence, from

(2)

(^

= ^,
M

and so

$= sin-i( n sin^j,
\

= 2sin"i( Msin-

a.

That M

is

tory experiment.

a minimum, is clearly indicated by the laboraIt can be proven analytically as follows.

From

(9)

dW-

dfl2

Differentiate (10) as it stands


tion, set dd'/dd

then, after the differentiaIt is seen at once tha*-

= 1 and & = 6'.


hence

^'<0,
and u has a minimum.

^>

0,

EXERCISE
mural painting 4 ft. high is 12 ft. above How far back from the wall should he stand, in order that the angle subtended by the painting be
of a

The bottom

the eye of the observer.


as large as possible ?

Suggestion.

Take the

distance, x, of the observer

from the

wall as the independent variable, and express the angle of


elevation of the bottom and the top of the painting in terms
of
X.

Printed in the United States of America.

You might also like